Torts practice questions

¡Supera tus tareas y exámenes ahora con Quizwiz!

An employer owes a duty to all those who may foreseeably come into contact with his employee to exercise due care in the hiring, supervision, and retention of the employee, and the landlord's retention of the manager under these circumstances may be a breach of that duty. If there is a breach, what kind of liability?

direct liability for negligent hiring

The plaintiff in the McDonald's case was found to be 20% responsible for her injuries and she lived in New Mexico, a comparative negligence jurisdiction, so she may recover how much from MacDonald's?

80%

What is one of the effects of res ipsa loquitur? (A) No directed verdict may be given for the defendant. (B) The burden of proof switches to the defendant. (C) It requires the defendant to present evidence of due care in rebuttal. (D) It creates a presumption of negligence.

A

Which of the following best states who may bring a strict liability action against a defendant engaging in abnormally dangerous activities? A Anyone injured as a result of the dangerous propensity of the activity B Any foreseeable plaintiff injured as a result of the dangerous propensity of the activity C Anyone directly injured by the activity D Any foreseeable plaintiff as long as she was directly injured by the activity

A defendant engaging in an abnormally dangerous activity may be liable only to foreseeable plaintiffs injured as a result of the dangerous propensity of the activity. This is the best statement of the scope of the duty owed. In most states, a defendant will be liable only to those persons to whom a reasonable person would have foreseen a risk of harm under the circumstances. In general, strict liability is not imposed for injuries to a plaintiff to whom no reasonable person would have foreseen a danger. The defendant will not be strictly liable to all plaintiffs who were directly injured by the activity. Rather, the harm must result from the kind of danger to be anticipated from the abnormally dangerous activity; i.e., it must flow from the "normally dangerous propensity" of the activity involved. Conversely, a foreseeable plaintiff may recover even if she was not directly injured by the activity as long as the injury was from the dangerous propensity (e.g., injuries caused by fleeing the danger from the activity).

The elements of defamation are

A statement about a business' or person's reputation or honesty that is untrue, a statement about a discernible business, publication to a third party, ALL of the above

A female employee of an insurance company was diagnosed with multiple sclerosis(also known as MS), which causes neurological problems and often results in permanent disability. Information about the diagnosis was transmitted to her employer's insurer. When her employer discovered the diagnosis, the woman was fired from her job. This action of the medical provider constitute

A violation of HIPAA, invasion of privacy, BOTH

The concept of imputed contributory negligence allows a defendant to use the contributory negligence of a third party as a defense against the plaintiff, where the plaintiff and the third party stand in a particular relationship. Which of the following relationships will permit the imputation of contributory negligence? A. Employer and employee B. Parent and child C. Automobile owner and driver D. Husband and wife

A. Employer and employee

If a bailment is for the sole benefit of the bailor, at common law the bailee __________. A. Is liable for gross negligence only B. Is liable for ordinary negligence C. Cannot be liable for negligence

A. Is liable for gross negligence only

A psychiatrist admitted a patient with severe depression to a hospital's observation unit. The psychiatrist noted "suicidal ideation" in the patient's chart intending the customary suicide-prevention measures. The hospital put none of those measures in place, even though it was customary in the observation unit to do so for patients suffering with thoughts of suicide. As a result, the patient committed suicide shortly after admission to the observation unit. Hospital administrators consulted a lawyer regarding liability to the patient's estate. Which of the following statements is the best evaluation? A. Liability because breaches of customary measures resulted in the suicide. B. Liability because no one should die an unnatural death in the hospital. C. No liability because suicide is an intervening and superseding act. D. No liability because persons are entitled to do as they please with their own life.

A. Liability because breaches of customary measures resulted in the suicide.

Which of the following is correct as to lessors of realty? A. Liability for conditions on the leased premises hinges on who occupies and controls the premises B. A lessor cannot pass the burden of maintaining the premises onto the lessee C. A lessor leasing the entire premises ordinarily remains jointly liable with the lessee for dangerous conditions

A. Liability for conditions on the leased premises hinges on who occupies and controls the premises

Which of the following is true regarding proximate cause for negligence? A. Proximate cause is a limitation of liability B. Proximate cause creates liability for unforeseeable consequences of one's conduct C. A defendant can be liable without his conduct being the proximate cause of an injury

A. Proximate cause is a limitation of liability

Which of the following is correct when the extent or severity of harm to the plaintiff is unforeseeable? A. The defendant is liable for any aggravation of an existing physical or mental illness B. The defendant can be liable only if he was the direct cause of the harm C. The defendant is only liable for the damages that were foreseeable

A. The defendant is liable for any aggravation of an existing physical or mental illness

A plaintiff cannot employ the last clear chance doctrine if: A. The defendant's negligence occurred prior to the accident B. The plaintiff was contributorily negligent C. The plaintiff but himself in peril

A. The defendant's negligence occurred prior to the accident

false imprisonment

An intentional act or omission by the defendant that causes the plaintiff to be confined or restrained to a bounded area

A local liquor store posts the checks of customers that are returned from the bank. The store manager says the public display of the checks is a good way to deter other bad-check writers. The display of the checks is:

An invasion of privacy

On September 28, 2017, Scott Baldwin, a Welsh rugby player, was bitten by a lion during a trip to a South African wildlife reserve when he stuck his hand between the metal bars of the lion enclosure to pet the lion. If US law applied and he sued the owners of the reserve for his injuries, the reserve's best defense would be

Assumption of risk

Bill entered a car race knowing that there was a risk of injury or death from a crash. Bill was seriously injured in a crash. Bill sued the owner of the track alleging negligence. The track owner's best defense is:

Assumption of risk, Bill knew the risk and voluntarily accepted it

Section 1 of the Vehicle Code of a state makes it illegal to cross a street in a central business district other than at a designated crosswalk. Section 2 of the Code prohibits parking any motor vehicle so that it blocks any part of a designated crosswalk. A pedestrian wanted to cross Main Street in the central business district of a city, located in the state, but a truck parked by a trucker was blocking the designated crosswalk. The pedestrian stepped out into Main Street and carefully walked around the back of the truck. The pedestrian was struck by a motor vehicle negligently operated by a driver. If the pedestrian asserts a claim against the trucker, the most likely result is that the pedestrian will A: prevail, because the trucker's violation of a state statute makes him strictly liable for all injuries caused thereby. B: prevail, because the probable purpose of Section 2 of the Vehicle Code of State was to safeguard pedestrians in using the crosswalk. C: not prevail, because the pedestrian assumed the risk of injury when he crossed the street outside the crosswalk. D: not prevail, because the driver's conduct was the actual cause of the pedestrian's harm.

B

Which of the following is correct regarding proximate cause in negligence? response - correct (A) Proximate cause is required only when an intervening force contributes to the injury. (B) A defendant is not liable for the harmful results of his conduct that are unforeseeable. (C) A defendant is liable for all harmful results caused by his acts. (D) Proximate cause is not required for a defendant to be liable, but actual cause is required.

B

To show breach of duty, a plaintiff may rely on evidence of any of the following except: A. Violation of a statute B. A "Good Samaritan" law C. Custom or usage D. Res ipsa loquitur

B. A "Good Samaritan" law

If a jurisdiction treats the violation of an applicable statute as negligence per se, this means that there is _________. A. A conclusive presumption of duty and evidence of breach of duty B. A conclusive presumption of duty and breach of duty C. A rebuttable presumption of duty and breach of duty D. Prima facie evidence of negligence

B. A conclusive presumption of duty and breach of duty

A plaintiff will be barred from recovery if his negligence is greater than 50% in: A. Pure comparative negligence jurisdictions B. All partial comparative negligence jurisdictions C. All comparative negligence jurisdictions D. Some partial comparative negligence jurisdictions

B. All partial comparative negligence jurisdictions

A "reasonable person" has which of the following of the defendant's personal characteristics? A. Mental characteristics B. Physical characteristics C. Mental and physical characteristics

B. Physical characteristics

Which of the following does a plaintiff not need to establish in an attractive nuisance case? A. The expense of remedying the dangerous situation is slight compared with the magnitude of the risk B. There was a dangerous condition on the land of which the landowner was aware C. The landowner knew or should have known that young persons frequent the vicinity of he dangerous condition

B. There was a dangerous condition on the land of which the landowner was aware

The defendant negligently blocked a road, forcing the plaintiff to take an alternate road that was equally safe. Another driver negligently collided with the plaintiff on that road. The defendant is not liable to the plaintiff because the collision is ______________. A. a foreseeable result caused by an unforeseeable intervening force B. an unforeseeable result caused by an unforeseeable intervening force C. an independent result caused by a dependent intervening force D. an unforeseeable result caused by a foreseeable intervening

B. an unforeseeable result caused by an unforeseeable intervening force

When an injured party neglects to take available steps to treat personal injuries inflicted by the defendant, the defendant can raise the doctrine of ___________. A. assumption of risk B. avoidable consequences C. last clear chance D. contributory negligence

B. avoidable consequences

While on a hiking trip during the late fall, the plaintiff arrived, toward the end of the day, at a clearing where several similar cabins were located, none of which was occupied. One of the cabins belonged to the plaintiff's friend, who had given the plaintiff permission to use it. The plaintiff entered one of the cabins, which she thought was her friend's, and prepared to spend the night. In fact, the cabin was owned, not by her friend, but by the defendant. When the night turned cold, the plaintiff started a fire in the stove. Unknown to the plaintiff, there was a defect in the stove that allowed carbon monoxide to escape into the cabin. During the night the fumes caused serious injury to the plaintiff. If the plaintiff asserts a claim against the defendant for her injury, will she recover? A: Yes, because the defendant knew that the stove was defective. B: Yes, because the defendant could have discovered the defect in the stove by a reasonable inspection. C: No, because the defendant had no reason to anticipate the plaintiff's presence in the cabin. D: No, unless the plaintiff needed to use the cabin for her own protection.

C

Which of the following statements is true regarding the element of damages in a negligence action? A. Nominal damages may be awarded in the absence of proof of damages B. The plaintiff must show actual economic loss to recover damages C. The plaintiff may recover damages for future harm

C. The plaintiff may recover damages for future harm

__________________ owe a duty to its passengers to furnish assistance when they are subject to danger, especially when it is the result of conditions created by the common carrier that cause a foreseeable risk of harm

Common carriers

Which of the following defenses to negligence serves as a complete bar to recovery?

Contributory negligence

Which of the following best describes res ipsa loquitur? (A) The fact that a particular injury occurred establishes breach of duty as a matter of law (B) Proof that a defendant violated a statute establishes the existence of a duty owed and breach thereof (C) Custom or usage establishes the standard of care in a given case (D) The fact that a particular injury occurred tends to establish the breach of a duty owed

D

A developer constructed several small stores in a commercial district. She received a bid from a contractor to install awnings on the front windows of the stores. The developer had heard that the contractor did shoddy work, but the price was right and the contractor expressly assumed all of the risk of any liability. The developer subsequently sold one of the stores to a barber. A few months later, an awning collapsed without warning, injuring a customer who was about to enter the barbershop. An investigation by the building inspector revealed that the awning collapsed because the brackets used by the contractor were cheaper and weaker than the required brackets, although they looked the same. The developer and the contractor are now both bankrupt. If the customer sues the barber for his injuries, is the customer likely to prevail? (A) Yes, because the barber had a nondelegable duty to keep the premises safe for customers and those passing by. (B) Yes, because the developer did not exercise reasonable care in hiring the contractor to install the awnings. (C) No, because the contractor assumed all of the risks from his work. Correct (D) No, because the barber had no opportunity to oversee the contractor's actions.

D (There was nothing wrong with the brackets evident from a reasonable inspection, and the awning collapsed without warning)

A casino advertised lighted, secure parking. The secure parking was so limited that on any given night most customers had to use a dark and unguarded back lot. A casino customer parked her vehicle in the dark and unguarded back lot because the lighted lot was full one late night. On her way in to the casino, the customer was robbed and assaulted in the darkness of the lot. The customer sued the casino. The casino's lawyer moved to dismiss the case based on no duty. How should the court rule? A. Grant the motion based on the event happening outside of the casino. B. Grant the motion based on no duty to protect against criminal acts. C. Deny the motion based on the money the casino would make from the customer. D. Deny the motion based on the advertised and assumed duty.

D. Deny the motion based on the advertised and assumed duty.

Businesses are never liable for criminal activity on their premises.

FALSE

Businesses are never liable for criminal activity on their premises.

False

I have committed defamation if I accuse my neighbor of setting my house on fire when we are speaking alone.

False

In a negligence action, the plaintiff cannot recover: A Unforeseeable damages B Noneconomic damages C Presumed damages D Damages for lost future earning capacity

In a negligence action, the plaintiff cannot recover presumed damages. Damage is an essential element of a plaintiff's prima facie case for negligence. This means actual harm or injury. Unlike for some intentional torts, damage will not be presumed in negligence. A plaintiff is entitled to all damages that he can prove, even if the extent of the damages was unforeseeable. Permissible damages includes economic damages, such as medical expenses and lost earnings, and noneconomic damages, such as pain and suffering. The plaintiff is also entitled to damages for lost future earning capacity, discounted to present value to avoid an excess award; i.e., the plaintiff receives an amount that, if securely invested, would produce the income that the jury wishes him to have.

Invasion of privacy includes

Intrusion into plaintiff's private affairs, public disclosure of private facts, appropriation of another's name for commercial advantage, ALL of the above

How long does title insurance protect an owner?

It continues to protect the owner after she sells the property, if ever sued on a title covenant.

A bouncy playhouse painted air pumps bright colors, but posted signs & childproof gates to keep kids out. A child's father opened the gate for him and the child was injured by an air pump. Can he collect? On what basis?

No, because the playhouse exercised reasonable care by posting signs. Although the child is a trespasser, the brighly painted air pumps are an attractive nuisance, so reasonable care is still the applicable standard. Therefore, the reason child can't collect IS NOT b/c he exceeded the scope of his invitation.

Is contributory negligence a defense to strict liability?

No.

A man delivered & recorded a deed granting his winery to his son. He orally told his son that the grant was conditioned on the son's sobriety for 12 months. The son wasn't sober for 12 months. Can the man set aside the conveyance?

No. Delivery creates a presumption of intent to make a present transfer. Although parole evidence is permitted to show lack of intent, it is not permitted to show an oral condition on the transfer.

Can a dentist be liable under strict products liability for implanting a defectively manufactured tooth?

No. Doctors are treated as service providers, not commercial sellers. So he's not held to strict liability for defective products.

Is the discharge of a mortgagor/seller's liability upon the lender's release of the mortgage dependant upon whether the buyer took subject to the mortgage or assumed the mortgage?

No. The release of the mortgage eliminates the seller's ability to be subrogated by the buyer for paying the debt either way, so it doesn't matter.

Firefighter is injured falling down ill-maintained stairs while fighting a fire negligently caused by a tenant. Can he recover from the landlord or the tenant?

Not from the tenant, but yes from the landlord. Firefighter's Rule bars negligence recovery for injuries resulting from risks inherent in the job. The improperly maintained stairs was not a risk inherent to the job.

The defense of shopkeeper's privilege

States that shopkeepers may detain individuals for a reasonable time

Letters of reference are usually exempt from liability for defamation as long as the employer verifies the truth of the statements in the reference.

TRUE

Opinion commentary is not defamation

TRUE

Physical damages are not required for liability for false imprisonment

TRUE

The article about Oprah below, if untrue, constitutes defamation as long as it is published with malice or with reckless disregard for the truth

TRUE

The plaintiff must prove that both "but for" causation and "proximate cause" exists in a negligence case in order to recover.

TRUE

To avoid liability for negligence, all persons are expected to behave as ordinary and reasonably prudent people under the same or similar circumstances.

TRUE

To establish assumption of risk, the defendant must show that the plaintiff was aware of the risk and accepted it voluntarily.

TRUE

Truth is a complete defense to defamation

TRUE

If some users decided not to use their own names and they chose to use the names of celebrities instead, what would a celebrity need to prove to recover for defamation?

That the celebrity was not a user of the site, that the celebrity's name was released using malice or reckless disregard for the truth, BOTH

What's the measure of damages for intentional misrepresentation?

The benefit of the bargain: value as represented - actual value (said chainsaw was worth $1000 & it was worth $0 - damages = $1000)

If the plaintiff establishes res ipsa loquitur, it will have the following effect: A A directed verdict will not be given for the defendant. B A directed verdict will be given for the plaintiff. C The burden of proof is shifted to the defendant. D A presumption of negligence is created.

The circumstantial evidence doctrine of res ipsa loquitur deals with those situations where the fact that a particular injury occurred may itself establish or tend to establish a breach of duty owed. Where res ipsa loquitur has been proven, the plaintiff has made a prima facie case, and a directed verdict will not be given for the defendant. Application of the doctrine does not shift the burden of proof to the defendant, nor does it create a presumption of negligence. Furthermore, the doctrine does not result in a directed verdict for the plaintiff. The defendant may introduce evidence that due care was exercised, and the jury may reject the permissible inference that may be drawn from the res ipsa proof and find for the defendant.

A small cruise ship struck a whale swimming underwater, causing the ship to suddenly lurch sideways. A passenger on the ship who was walking down a corridor lost his balance and bumped his head on the edge of a doorway. Because of a previously existing medical condition that made him susceptible to bleeding on the brain, he suffered a cerebral hemorrhage and permanent mental impairment, despite prompt medical attention on the ship. The passenger brought suit against the cruise ship owner for his damages. At trial, the passenger presented evidence of how he was injured as he walked down the hallway, his previous medical condition, and his medical expenses and other damages. The cruise ship owner presented evidence that the cruise ship was following its approved route and that the whale could not have been detected before impact, and that the bump would not have injured someone in ordinary health. At the close of the evidence, the cruise ship owner moved for a directed verdict. How should the court rule? A Grant the motion, because there is no evidence that the crew operated the ship negligently. B Grant the motion, because the cruise ship owner introduced uncontroverted evidence that a person in normal health would not have been injured by the bump. C Deny the motion, because the jury could find that the cruise ship owner, as a common carrier and innkeeper, breached its high duty of care to the passenger. D Deny the motion, because the fact that the severity of the passenger's injuries was not foreseeable does not cut off the cruise ship owner's liability.

The court should grant the cruise ship owner's motion because the passenger has not established a prima facie case of negligence against the cruise ship. To establish a prima facie case for negligence, a plaintiff must show (i) a duty of care, (ii) breach of that duty, (iii) actual and proximate cause, and (iv) damages. As a common carrier and/or an innkeeper, the cruise ship owed its passengers a high duty of care, and therefore would be liable for slight negligence. However, the passenger has offered no evidence to establish that the cruise ship employees breached that duty, and res ipsa loquitur is not applicable here because the collision with the whale swimming underwater is not the type of event that would occur only as a result of negligence. Because the passenger failed to establish breach of duty, the court should grant the cruise ship owner a directed verdict. (B) is incorrect because the cruise ship owner does not need that evidence to prevail. While evidence that a person in normal health would not have been injured by the bump supports the cruise ship's other evidence that it exercised due care, it is not necessary because the passenger has failed to offer evidence that the cruise ship owner breached its duty. On the other hand, if the cruise ship owner had breached its duty of care to its passengers, the fact that a person in normal health would not have been injured by the bump on the head would not be a defense to liability. If a defendant's negligence causes an aggravation of a plaintiff's existing physical illness, the defendant is liable for the damages caused by the aggravation. (C) is incorrect because, as discussed above, the passenger has failed to present evidence that the cruise ship owner breached the high duty of care that it owed to its guests. (D) is incorrect even though it is a true statement of law, as discussed above. The reason the cruise ship owner prevails is because the passenger has failed to establish a prima facie case.

The defendant negligently blocked a road, forcing the plaintiff to take an alternate road that was equally safe. Another driver negligently collided with the plaintiff on that road. The defendant is not liable to the plaintiff because the collision is: A An unforeseeable result caused by a foreseeable intervening force. B An unforeseeable result caused by an unforeseeable intervening force. C A foreseeable result caused by an unforeseeable intervening force. D An independent result caused by a dependent intervening force.

The defendant is not liable because the collision is an unforeseeable result caused by an unforeseeable intervening force. As a general rule, intervening forces that produce unforeseeable results (i.e., results that were not within the increased risk created by defendant's negligence) will be deemed to be unforeseeable and superseding. A superseding force is one that serves to break the causal connection between defendant's initial negligent act and the ultimate injury, and itself becomes a direct, immediate cause of the injury. Thus, the defendant will be relieved of liability for the consequences of his antecedent conduct. Here, the defendant's negligence was an actual cause of the plaintiff's injury because it would not have happened but for the defendant's negligence, but it did not increase the risk that a completely unrelated collision would happen. The result was not caused by a foreseeable intervening force because the other driver's negligence was completely unrelated to any risk created by the original defendant's negligence. Similarly, the collision was not a foreseeable result because traveling on the equivalent alternate road did not make a collision foreseeable.The collision is not an independent result caused by a dependent intervening force because dependent forces are those that are normal responses or reactions to the situation created by the defendant's negligent act, and here the force was unrelated to the defendant's conduct.

A baseball fan sued ESPN, the network's announcers, Major League Baseball and the Yankees for $10 million for defamation after he was filmed sleeping in April 2014 during the fourth inning of Yankees game against the Boston Red Sox. The ESPN broadcasters described him using derogatory words, such as "stupor" and "stupid". What was a valid reason for the court to dismiss the case?

The fan was sleeping and the other statements were opinions

Comparative negligence reduces the plaintiff's recovery.

True

Intentional infliction of emotional distress imposes liability for conduct that exceeds all bounds of decency.

True

Opinion commentary is not defamation.

True

If a plaintiff is found to be 30% responsible in a lawsuit for her injuries in a comparative negligence jurisdiction, then she may recover a. 70% of her damages. b. 80% of her damages. c. nothing. d. 30% of her damages.

a. 70% of her damages.

A prima facie case in products liability based on strict tort liability consists of the following: (i) the defendant is a ___________; (ii) the defendant produced or sold a ____________; (iii) the product was the ________________ of the plaintiff's injury; and (iv) the plaintiff ____________.

commercial supplier; defective product; actual and proximate cause; suffered damages

intentional tort defenses: 1. 2. 3.

consent; defense of self, others, or property; necessity

An intentional act by the defendant that causes a serious interference with the plaintiff's right of possession in a chattel

conversion

Best practices when invoking the shopkeeper's privilege include a. Stopping shoplifters discreetly. b. Yelling, "stop thief" when a suspected thief leaves the store. c. Not using physical force unless it is in response to the shoplifter's physical force d. a and c only. e. None of the above

d. a and c only.

Negligence requires which of these elements: a. breach (failure) of duty of care b. injury c. causation d. all of the above

d. all of the above

Oh no! You used your drone to peek into your competitor's top-secret production facility. Unfortunately for you, your competitor was able to trap it and trace the drone back to you. Assuming that corporations can recover damages for this tort, then you are liable for a. negligence b. assault c. defamation d. invasion of privacy

d. invasion of privacy

(Fact Pattern) Why did I use the term "allegedly" when writing about the suspect, who was charged but not yet convicted for this awful crime? a. most states require people to use "allegedly" by statute. b. to avoid liability for assault. c. to avoid liability for invasion of privacy. d. to avoid liability for defamation.

d. to avoid liability for defamation.

strict liability of product liability only attaches to ________________

harm caused by the defect

In general, a principal will not be vicariously liable for tortious acts of an independent contractor. Two broad exceptions exist, however: (i) __________________ (ii) ___________________

independent contractor is engaged in inherently dangerous activity; the duty, because of public policy, is simply nondelegable (includes duty of business to keep premises safe)

An intervening force occurs after the defendant's negligent act and combines with it to cause the injury

indirect cause cases

in regards to nuisance, the balancing of competing interests is only done when ____________________ from the nuisance is sought

injunctive relief

Intentional extreme and outrageous conduct by the defendant that causes the plaintiff to suffer severe emotional distress

intentional infliction of emotional distress

battery is ____________, ______________, _______________ contact

intentional, unpermitted, offensive

In a strict liability question, where the manufacturing company is the one who rly ****ed up, the fact that the hardware store was not at fault and had no opportunity to inspect is ___________. It is____________ because it is a commercial supplier of a defective product and the neighbor is suing under a strict liability theory.

irrelevant; liable

Under _____________________ , each defendant found by the trier of fact to be at fault for an indivisible injury is liable to the plaintiff for the entire amount of damages incurred, not just a portion of it.

joint and several liability

although the principle that "danger invites rescue" exists, the "danger" that occurred has to have happened _____________

negligently

landowner duty to undiscovered trespasser

no duty

A plaintiff can recover in battery regardless of __________________________

whether she was harmed (she would get nominal damages)

if a rescuer is injured while rescuing somebody in danger who was acting with reasonable care, the rescuer _________ recover

will not (the rescuee had to have been acting negligently)

In order to prove negligence

"But for" causation must be proven, proximate cause must be proven, BOTH

How is "actual malice" defined for purposes of the constitutional law of defamation? A Knowledge that the statement was false, or reckless disregard by the defendant as to the statement's truth or falsity. B At least negligence by the defendant as to the statement's truth or falsity. C Ill will or spite towards the plaintiff by the defendant. D Knowledge that the statement will defame the plaintiff, or reckless disregard by the defendant as to the effect of the defamatory language.

"Actual malice," which must be established by a public official or public figure to recover for defamation, was defined by the Supreme Court as (i) knowledge that the statement was false, or (ii) reckless disregard by the defendant as to the statement's truth or falsity. Negligence by the defendant as to the statement's truth or falsity is the minimum standard imposed by the Court on a private figure when the defamation involves a matter of public concern; actual malice is a higher standard. Ill will or spite towards the plaintiff by the defendant is how malice was defined at common law, but is not used by the Supreme Court to define malice. Knowledge that the statement will defame the plaintiff or reckless disregard by the defendant as to the effect of the defamatory language is not how malice was defined by the Supreme Court. The Court used actual malice to describe the defendant's necessary state of mind as to the statement's truth or falsity, not its defamatory nature.

Which of the following intentional conduct by the defendant is LEAST likely to constitute a trespass to land? A Exploding a mine on his own land which causes concussion damage to the plaintiff's land. B Throwing a rock onto a plaintiff's driveway which causes no damage. C Flooding his own land which causes water damage to the plaintiff's land from the overflow. D Chasing someone from his own land onto the plaintiff's land which causes no damage.

(A) If a defendant explodes a mine on his land which causes only concussion damage to a plaintiff's land, this will likely not constitute a trespass to land. A trespass to land requires a physical invasion of a plaintiff's real property by a defendant. If no physical object enters onto the plaintiff's land due to the defendant's actions, courts will generally not treat the defendant's conduct as a trespass to land. Instead, this will constitute a case of nuisance or strict liability (if ultrahazardous activities are involved). If a defendant throws a rock onto the plaintiff's driveway, causes water to flow onto the plaintiff's land, or chases someone onto the plaintiff's land, this will likely constitute a trespass to land. Although in none of these scenarios does the defendant actually enter upon the plaintiff's land, a trespass to land does not require that the defendant personally come onto the land. The tort of trespass to land protects a plaintiff's exclusive possession of realty from physical invasion, and all that is required to satisfy this element is a physical invasion of the plaintiff's land. Furthermore, damages are not required for this tort. The defendant has committed a trespass even if the property was not damaged.

The "shopkeeper's privilege" allows a shopkeeper to avoid liability for false imprisonment when detaining a suspect that he reasonably believes has committed a theft. The shopkeeper also must: A Conduct the detention in a reasonable manner and detain the suspect for only a reasonable time. B Conduct the detention in a reasonable manner and notify the police in a reasonable amount of time. C Detain the suspect for only a reasonable time and notify the police in a reasonable amount of time. D Conduct the detention in a reasonable manner, detain the suspect for only a reasonable time, and notify the police in a reasonable amount of time.

(A) In addition to having a reasonable belief as to the fact of theft, a shopkeeper is required to conduct the detention in a reasonable manner and detain the suspect for a reasonable period of time for the privilege to apply. By statute in some states and case law in others, shopkeepers have been given a privilege to detain someone suspected of shoplifting and thus avoid liability for false imprisonment. The following conditions must be satisfied: (i) there must be a reasonable belief as to the fact of theft; (ii) the detention must be conducted in a reasonable manner and only nondeadly force can be used; and (iii) the detention must be only for a reasonable period of time and only for the purpose of making an investigation. A shopkeeper is not required to notify the police in a reasonable amount of time to avoid liability for false imprisonment when detaining a suspect for shoplifting.

An inexperienced worker who was instructed to clean the floors of a store mixed ammonia and chlorine bleach in a large pail. Both he and a customer who was standing nearby were overcome by fumes and suffered lung damage. The customer sued the worker, alleging negli- gence. In defense, the worker presented uncon- troverted evidence that he could not read the warning labels on the containers and that, while he knew he was mixing ammonia and bleach, he had never been made aware of the danger of mixing the two chemicals. Nevertheless, the jury found him liable for the customer's injuries. If the worker challenges the verdict on appeal, how should the appellate court rule? (A) Uphold the verdict, because it was a deter- mination that a reasonable person should have known of the danger. (B) Uphold the verdict, because it was a deter- mination that the worker's evidence was not believed. (C) Overrule the verdict, because it is inconsis- tent with the evidence. (D) Overruletheverdict,becausetheworker's lack of knowledge of the danger should have been taken into account.

(A) The appellate court should uphold the verdict because it is a determination by the jury that the worker did not act as a reasonable person would have acted. In a negligence action, the defendant's conduct is measured against the reasonable person, an objective standard. The defendant must act as would a person of average mental ability, and he is deemed to have knowledge of things known by an average member of the community; individual shortcomings or ignorance of that particular defendant are not considered. Here, the verdict of negligence was a determination by the jury that a reasonable person should have known of the danger, regardless of the fact that the defendant did not. Hence, the verdict should be upheld because there is no basis in the facts for overturning it. (B) is not as good a choice as (A) because nothing suggests that the jury did not believe the worker's evidence. The worker admittedly knew that he was mixing ammonia and bleach. Given that he was also injured by the fumes, his assertion that he was not aware of the danger was very believable. (C) is incorrect because the appellate court will not overrule a determination of negli- gence by the trier of fact unless no reasonable jury could have made that determination. Given that the standard of care requires the exercise of knowledge and ability of the average person, a reasonable jury could have found negligence here. (D) is incorrect because, as discussed above, the worker's lack of knowledge is not taken into account when determining the standard of care for negligence.

A pilot was flying her small airplane when she realized that she was rapidly losing fuel and would not make it to the nearest airport. Looking down, she could find no large open space on which to attempt a landing except for a highway off to her left and a nearby lake about a mile to her right. She considered ditching the plane in the water but decided against it under the circumstances. As the pilot maneuvered over the highway and saw a long section free of any overpass or obstruction, her engine sputtered and died. In a barely controlled glide, the pilot descended onto the highway, but her left wing sideswiped the median and her plane veered to the right, crashing into a car. The plane and car catapulted into a fence, severely injuring both the pilot and driver. The driver brought an action for personal injuries against the pilot. At trial, the above facts were established, and the parties stipulated that the sudden loss of fuel was due to a defect in the fuel system that could not have been discovered by the pilot. At the close of the evidence, both parties moved for a directed verdict. How should the court rule? (A) Deny both motions, because the jury could decide that the pilot's selection of the high- way rather than the lake was not a reason- able choice under the circumstances. (B) Grant the driver's motion, because the evidence establishes that his injuries were the result of a defect in the pilot's plane. (C) Grant the pilot's motion, because the parties stipulated that she was not negligent in failing to discover the defect in her fuel system. (D) Grant the pilot's motion, because she made the decision to land on the highway rather than the lake under emergency conditions.

(A) The court should deny both motions and submit the case to the jury. Through process of elimina- tion, this has to be a negligence action. Clearly the pilot did not commit an intentional tort, and the driver cannot sue the pilot, the plane's owner, in strict liability. The jury could determine that the pilot was negligent in selection of the landing site; hence, the court should deny the pilot's motion. The court should also deny the driver's motion because this is not a strict liability action; negligent conduct needs to be established. Hence, (B) is incorrect. (C) is incorrect because even though the pilot was not negligent in discovering the defect, she could have been negligent in selecting the landing site. (D) is incorrect because in an emergency, the pilot is held to the standard of care of a reasonable person in an emergency. It is up to the jury to determine whether she acted reasonably under the emergency circumstances.

A tenant invited a friend over for dinner. On his arrival, the friend stepped on a split board on the front steps and the board broke, causing him to lose his balance and break his ankle. If the friend sues the tenant for his injuries and does not prevail in a jurisdiction that applies the traditional rules for landowners and possessors of land, what is the most likely expla- nation? (A) In the lease, the landlord had undertaken the duty to discover and repair dangerous conditions on the premises. (B) The friend arrived an hour earlier than his invitation specified. (C) The friend should have noticed the dangerous condition himself. (D) The tenant had stayed beyond the lease term and she no longer had the legal right to occupy the premises.

(C) If the friend does not prevail, it will be because he should have noticed the dangerous condition himself. In jurisdictions applying the traditional rules for landowners and possessors of land, the nature of the duty owed by an owner or occupier of land to those on the premises for dangerous conditions on the land depends on the legal status of the plaintiff in regard to the property, i.e., trespasser, licensee, or invitee. A licensee is one who enters on the land with permission for his own purpose or business and includes social guests. The owner or occupier owes a licensee a duty to warn of or make safe a dangerous condition known to the owner or occupier that creates an unreasonable risk of harm to the licensee and that the licensee is unlikely to discover. The owner or occupier does not have a duty to inspect for defects or to repair known defects. Here, the friend had been invited for dinner, making him a licensee. The facts do not indicate whether the tenant knew of the split board and neglected to alert the friend or simply was not aware of it, but the duty to warn does not extend to dangerous conditions that the licensee should reasonably have discov- ered. Hence, (C) presents the best basis for the friend not prevailing. (A) is incorrect because that fact would not make a difference to the tenant's liability. The tenant remains liable to the friend for dangerous conditions on the premises as the occupier of the land, regardless of the landlord's obligation to inspect and repair. (B) is incorrect because the friend still qualifies as a licensee even though he arrived sooner than his invitation specified. It is true that a person may lose invitee status and become a licensee by being on the premises at a time outside the scope of his invitation. However, there is no similar principle applicable here. The fact that the friend arrived an hour early does not make him a trespasser rather than a licensee. Hence, the fact in (B) would not affect the tenant's liability. (D) is incorrect because it also is irrelevant. Even if the tenant had no legal right to occupy the land, she still would be the possessor of the land as to the friend, and she owed the friend the duties owed to a licensee.

At a little league game, a seven-year-old boy was called out on strikes. The boy's father was so infuriated with the umpire's decision that he shouted in a loud voice, "Kill the umpire." The boy, who was still holding his bat, swung the bat at the umpire. The umpire ducked and the bat flew out of the boy's hands and struck a spectator, who was seriously injured. In a tort action by the umpire against the boy which of the following statements is correct? (A) The umpire could recover only on an as- sault theory. (B) The umpire could recover either on an assault theory or a negligence theory. (C) The umpire could recover only on a negli- gence theory. (D) The umpire could not recover.

(A) The umpire could recover on an assault theory. Even though the boy did not hit the umpire, his action still constitutes an assault because it can be shown that he intended to cause harmful contact and actually created an apprehension of contact. Children are liable for their intentional torts when they are capable of forming the requisite intent. For assault, the intent required is to bring about the offensive or harmful contact; knowledge of its wrongfulness is not an element of that intent. The fact that he swung the bat at the umpire strongly indicates that he believed his bat would hit the umpire and that the boy's purpose in doing so was to bring about this result. There- fore, it can be assumed that the boy had the capacity to form an intent to hit and that he did form that intent, which is a required element of the tort of assault. Thus, (C) and (D) are incorrect. The umpire could not recover on a negligence theory because the facts do not suggest that the umpire suffered any physical harm as a result of the boy's act. Specifically, most jurisdictions hold that recovery for negligent infliction of mental distress can only be had when the plaintiff has actually sustained physical contact as a result of defendant's actions or has suffered actual physical harm. These facts do not indicate that the harm done to the umpire is the kind of harm compensable in negligence. Therefore, (B) and (C) are incorrect.

Which of the following is correct regarding self-defense? A Retaliation may be permissible. B Deadly force may be permissible. C Retreat is required. D Actual necessity is required.

(B) Deadly force may be permissible for self-defense. A person may use deadly force to prevent death or serious bodily injury to herself. Self-defense requires use of force that reasonably appears necessary to prevent harm. A person may use deadly force if she reasonably believes that she is in danger of serious bodily injury. Retaliation is not permissible for self-defense. Self-defense is only permitted to prevent the commission of a tort. A person cannot retaliate by using force when there is no longer a threat of injury. Retreat is not required for self-defense. A majority of courts hold that a person may stand her ground and need not attempt an escape. Actual necessity is not required for self-defense. A person need only have an apparent necessity to defend oneself, i.e., a reasonable belief that she is being, or is about to be, attacked.Thus, a reasonable mistake as to the need for self-defense does not eliminate this defense.

Which one of the following types of common law defamation requires pleading and proof of special damages? A Libel not falling into one of the four per se categories. B Slander not falling into one of the four per se categories. C Slander not defamatory on its face. D Libel not defamatory on its face.

(B) Slander not falling into one of the four per se categories requires pleading and proof of special damages (i.e., pecuniary losses). Injury to reputation is not presumed for spoken defamation unless it falls into one of the four categories of slander per se. Damages will be presumed if the defamation (i) disparages the plaintiff in the conduct of her business or occupation; (ii) asserts that the plaintiff is suffering from a loathsome and communicable disease; (iii) alleges that the plaintiff has committed a serious crime or crime of moral turpitude; or (iv) imputes unchastity to a female plaintiff. Libel does not require special damages, even libel not falling into one of the four per se categories. In most jurisdictions, general damages are presumed by law for all libel; i.e., special damages need not be established. The fact that slander is not defamatory on its face does not affect whether special damages are required; rather, whether the slander falls into the per se categories determines that requirement. Only a minority of courts require special damages for libel not defamatory on its face. As stated above, most states do not require special damages for libel.

A landowner owned several dozen acres of mountain land near a national forest. A plaintiff who was injured by a condition on the owner's land brought an action for personal injury against the landowner. In a jurisdiction that applies the traditional rules for landowners and possessors of land, which of the following plaintiffs is most likely to win? (A) A 10-year-old trespasser who was swept onto some rocks while attempting to cross a swiftly flowing river. (B) A five-year-old trespasser who fell into a mineshaft from which the owner had removed all warning signs, but the plain- tiff was not attracted onto the owner's land because of the mineshaft. (C) A five-year-old trespasser who inadver- tently stepped into a badger hole that was obscured in the undergrowth. (D) A 10-year-old niece visiting the landowner who stepped into a badger hole that the landowner did not know was present but that could have been discovered by inspec- tion.

(B) The child who fell into the mineshaft is most likely to win. Under the attractive nuisance doctrine, a child trespasser who is injured by a dangerous artificial condition need not have been attracted onto the property by the condition. (A) is wrong because generally there is no obligation for a landowner to warn trespassers, whether they are children or adults, of dangerous natural condi- tions. (C) is wrong for the same reason. (D) is wrong because, as a licensee, the plaintiff need only be warned of dangerous natural conditions of which the landowner is in fact aware and which are unknown to the licensee or unlikely to be discovered by her, and here the landowner did not know of the hole.

A famous comedian was asked by the host of a popular late night television talk show what brand of cigars he smoked. He responded, "I smoke only [the manufacturer's] cigars, because they're the best." Two weeks later the manufacturer of those cigars began a national advertising campaign featuring billboards, posters for use in retail stores, and full-page ads in high circulation magazines. The advertising featured a picture of the comedian with the manufacturer's cigar in his hand, and the copy quoted his statement from the show. The manufacturer had not received the comedian's permission to use either his picture or the statement that had been made during the interview. Will the comedian prevail in an action against the manufacturer for using his picture and state- ment? (A) Yes, because the comedian has been de- famed. (B) Yes, because the comedian's likeness was appropriated for a commercial purpose without his consent. (C) No, because the advertising accurately reflects what the comedian said publicly before millions of television viewers. (D) No, because the comedian's appearance on television created an implied consent to reasonable use of anything he might say.

(B) The comedian will prevail on invasion of privacy grounds. (B) is a correct statement of the law defining the invasion of the comedian's privacy by appropriating his likeness for a commercial purpose (i.e., using the comedian's picture to promote the sale of a commercial product without the comedian's consent). (A) is incorrect because the comedian lacks a prima facie case for defamation (there was no defamatory statement causing damage to the comedian's reputation) and, in any case, what the manufacturer printed was true. (C) is relevant to defamation, but is incorrect here, because accuracy is not a defense to privacy torts. (D) is incorrect because consent to commercial appropriation must be specific; it may not be implied from the comedian's state- ments on the television program.

A horse breeder owned a small but excep- tionally well-tended horse farm for many years. The county in which the farm was located had no zoning or land-use regulations, but that had never been a problem until a half-acre plot of land next to the farm was recently purchased by a salvage company. The company let the weeds grow high on the land and it became littered with smelly, unsightly garbage and rusting metal. The breeder complained to the company on several occasions but was ignored. In addition, business started to taper off at the breeding farm due to the noise, smells, and general disarray of the junkyard. If the breeder brings an action for nuisance against the company, how will the court rule? (A) For the breeder, because the breeder was a property owner in the area long before the company bought the lot and opened the business. (B) For the breeder, if he can show a substantial and unreasonable interference with the use and enjoyment of his land. (C) For the company, because it is using the land for legal purposes. (D) For the company, unless the breeder can objectively demonstrate that the value of the farm has declined.

(B) The court will rule for the breeder. A private nuisance is a substantial, unreasonable interference with another's use or enjoyment of his own property; hence, (B) is correct because it states an element that the breeder must prove. (A) is incorrect because the fact that the plaintiff owned the property first does not establish the nuisance case. (C) is also incorrect. The fact that the defen- dant is using his property for legal purposes is not a defense to a nuisance action. (D) is incorrect, because a decline in the property's value is not an element of the prima facie case of nuisance.

A driver on a 3,000-mile cross-country trip in his new car tried to drive the entire trip without stopping, but fell asleep at the wheel, causing the car to strike a bridge abutment and roll over. The driver was seriously injured by the rollover, and suffered additional injuries when the turn signal rod broke off and punctured his lung. The driver had purchased the car from his local auto dealer. The car was manufactured by a local manufacturer, and the turn signal rod was manufactured by a subcontractor whom the manufacturer had used for many years. Tests after the accident established that the turn signal rod was defective and that the defect was the reason it broke off. The defect was not discov- erable through reasonable inspection and the manufacturer had had no prior indication of any defects. The driver brought a strict liability action against the manufacturer in a jurisdiction that does not apply its comparative negligence rules to strict liability actions. What is the likely result of the driver's action? (A) The driver will be awarded damages for all injuries incurred as a result of the accident. (B) The driver will be awarded damages for injuries incurred because the turn signal rod was defective, but he will not recover for other injuries incurred in the accident. (C) The manufacturer will prevail, because the accident was caused by the driver's negli- gence. (D) The manufacturer will prevail, because the driver cannot show that the manufacturer knew or should have known that the turn signal rod was defective.

(B) The driver will be awarded damages for injuries from the turn signal rod. Even though the defec- tive turn signal rod was not the proximate cause of the accident, the driver can recover from the manufacturer for the additional injuries suffered due to the defective turn signal rod, because those are identifiable as caused by that defect. The manufacturer is strictly liable under a products liability theory for injuries caused by that defect, even though a subcontractor manufactured the rod. Thus, (B) is the correct answer. (D) can be eliminated because it deals with a negligence theory, and the suit is being filed under a strict liability theory. Because liability attached only for the damages actually caused by the defective turn signal, (A) is incorrect.

An infant was injured in an automobile accident when the vehicle, driven by the infant's mother, left the roadway and rolled over down an embankment. At the time of the accident, the infant was buckled into an infant carrier car seat. The carrier was designed to snap into a base that was secured in the back seat by the rear center seat belt. Prior to driving, the mother had snapped the car carrier onto the base and pulled up on the car carrier's handle to ensure that the carrier was indeed secured in the base. When the rollover occurred, however, the carrier came loose from the base and was thrown about the inside of the vehicle, causing injuries to the infant's neck and face. The mother brought a products liability action on behalf of the child against the manufacturer of the car carrier, alleging that the manufacturer was negligent in the design of the base and seat combination. If the mother establishes at trial that the force of the rollover was enough to knock the seat loose, and that a reasonable, economically feasible alternative design existed, which of the following, if true, would be most helpful to the manufacturer's defense? (A) The mother violated a statute by traveling too fast for conditions, which caused the rollover accident. (B) No one had reported a car carrier coming loose in a rollover prior to this accident. (C) The car seat conformed with federal labeling requirements. (D) The retailer who sold the car seat was negli- gent in failing to notice the defect.

(B) The most helpful fact is that no one had reported this type of problem previously. The mother is alleging that the manufacturer's negligence led to the supplying of a defective product. To estab- lish this, the plaintiff must show that those designing the product knew or should have known of enough facts to put a reasonable manufacturer on notice about the dangers of marketing the product as designed. Negligent design is not shown, however, if the danger of the product becomes apparent only after the product reaches the public. Hence, the absence of any previous complaints about this problem would be most helpful to the manufacturer. (C) is less helpful than (B). Although compliance with government safety standards, such as labeling, is evidence that the product is not defective, it is not conclusive evidence, and federal labeling requirements do not preempt state products liability laws on defective warnings. (A) will not support the manufac- turer's defense. The answer choice suggests that the mother was contributorily negligent; however, the contributory negligence of a parent is not imputed to the child. (D) does not help the manufac- turer. An intermediary's negligent failure to notice a defect does not relieve the liability of a manufacturer whose original negligence was the cause of the defect.

A tenant moving into a new apartment bought a spool of "10-pound test" fishing line, manufactured by a fishing tackle and accessories company, for the purpose of hanging pictures, all of which she knew weighed less than 10 pounds. The spool came with no guidelines or warnings about using it for hanging objects. She attached the fishing line to either end of the pictures and hung them on hooks on the wall. The next week, a friend visiting the tenant was hit and injured while sitting on the couch by a picture that fell when the fishing line broke. It is common knowledge in the sporting goods industry that "10-pound test" indicates that the line will stand a pull of 10 pounds, but is not intended to support a 10-pound weight over a period of time. However, it is also common knowledge in the industry that the public in general uses fishing line to support heavy hanging objects over a period of time. The friend brought a products liability action against the tackle company for damages caused by his injury. What is the most likely result? (A) The friend will win, because the line failed to support a weight of less than 10 pounds. (B) The friend will win, because the label did not warn the consumer against relying on the term "10-pound test" for purposes other than fishing. (C) The friend will lose, because the line was not being used for its intended purpose, fishing. (D) The friend will lose, because the line conformed to the accepted standard for "10-pound test line."

(B) The most likely result is that the friend will prevail on grounds of inadequate warnings. Where a product presents an unreasonable risk of injury to users and bystanders, the fact that there is no sufficient warning of the danger may be a dangerous defect that will invoke strict products liability. The facts establish that the fishing tackle company knew that purchasers of its fishing line commonly used that line to support heavy hanging objects, and that the line would not support the weight rating in that type of use. Thus, the fishing tackle company's failure to provide a warning may be a defect that will permit the friend to recover in strict liability. Given that the other choices are clearly incorrect, (B) is the best option. (A) is incorrect because the fact that the line would not support a hanging weight of less than 10 pounds is not alone sufficient to constitute a dangerous defect; it is the fact that the fishing tackle company knew of the danger represented and failed to provide a warning to such users that may make the injury to the friend actionable. (C) is incorrect because the foreseeable misuse of a dangerously defective product by a purchaser or user does not relieve the manufacturer of the product of liability for injuries that are caused by the defect. Here, the known use of fishing line to support heavy hanging weights is what prompts the need for a warning, the absence of which may make the product dangerously defective. (D) is incorrect because, as indicated above, the presence or absence of a dangerous defect, and the responsibility for injuries caused by the defect, is not controlled by whether a product meets standards of performance measured solely by the use intended by the manufacturer. The fact that the fishing line performed adequately under industry standards when used for fishing does not mean that it cannot be dangerously defective when used for another purpose, when such use is known or reasonably should have been known by the manufacturer.

The parents of an eight-year-old boy at a public school told the school principal that their son said he had been molested by his teacher. The next day, the principal confronted the teacher with the accusation. The teacher admitted sexually molesting the boy, but he pleaded with the principal for another chance, promising never to do it again. Although the principal believed him, he still insisted that the teacher must leave. The teacher then threatened to blackmail the principal with information he had about a former student of the principal who had an affair with him when he taught at the local high school. Fearful that he would lose his position if this accurate information ever came to light, the principal allowed the teacher to continue teaching, and told the family that the boy's charges were unfounded. Two weeks later, the teacher molested the boy again, and the teacher was arrested and charged with both incidents. In a suit against the principal on the boy's behalf, will the parents prevail? (A) Yes, because the principal is vicariously liable for the teacher's torts. (B) Yes, because the principal was aware that the teacher was a child molester and did nothing. (C) No, because the principal did not molest the boy. (D) No, because the principal believed the teacher when the teacher told the principal that he would not molest children again.

(B) The parents will win because the principal was aware of the teacher's conduct and did nothing. The principal and the teacher are in an employer/employee relationship. Normally, intentional torts of the employee are deemed not within the ambit of the employment relationship, except in jobs such as that of a bill collector or club bouncer. There is no evidence that intentional tortious conduct is part of this employment relationship. So the principal cannot be vicariously liable for the teacher's intentional torts and (A) is incorrect. However, there is a cause of action against the principal under a negligence theory. The principal knew that the teacher was a child molester. The principal initially indicated that the proper thing to do was for the teacher to leave. He backed down only under threats. Clearly action of some sort should have been taken, and the parents should have been told the truth about the teacher. The fact that the principal did nothing is clearly negligence on his part. (C) is incorrect because, as explained above, the principal can be liable for negligence even if he did not molest the boy. (D) is incorrect because, even though the principal believed the teacher, he had a duty to take action to protect the schoolchildren from the teacher.

Two members of a backgammon club owned identically sized, red backgammon boards. The first member's board was made of cheap material while the second member's board was quite expensive. One night, after a competitive tourna- ment, the two members met in the finals, playing on a borrowed board. The second member won and the first member, visibly upset, mistakenly grabbed the other's board and drove home. As was her custom, she left the board in the trunk of her car. Meanwhile, the owner of the board discovered the board switch and drove to the first member's apartment to make an exchange. The first member took the second to her parking place and saw that her car had been stolen. The police recovered the car days later, with no backgammon board in the trunk. The second member demanded a replacement board, but was refused. In an action to recover the board's value, will the second member recover? (A) Yes, because when the first member took the board she committed a trespass to chat- tel. (B) Yes, because when the board was stolen along with the car, the first member became liable for conversion. (C) No, because the first member believed in good faith that the board was hers when she took it from the club. (D) No, because the board was lost through no fault of the first member.

(B) The plaintiff will win because the defendant committed a conversion. A conversion occurs when the defendant intentionally causes serious interference with the chattel of the plaintiff. The intent involved refers to the physical act that results in the conversion, not to the defendant's desires regarding the ultimate disposition of the property. Therefore, the first member was guilty of conversion when she intentionally (i.e., volitionally) took the second's board, which resulted in its loss, even though the first member did not intend to lose it or even realize that she had taken the property of another. (A) is not the best answer because complete loss of a chattel is too serious an interference to be classified a mere trespass. (C) is wrong because the first member's good faith is irrelevant in a conversion action. (D) is wrong because the fact that the first member's car was stolen does not relieve her of liability.

A man and a woman who were fierce business competitors were both competing for a large job. The man submitted his bid and then went to the woman's office and told her, "If you leave this office, I'm going to get you!" The woman merely laughed and said, "I'm about finished with my bid and will be leaving in a few minutes." The man left the office but placed a large, heavy couch across the entrance to the woman's office, hoping to keep her from leaving. Meanwhile, the woman finished the bid and tried to leave her office, but found that she could not open the door. She pushed against the door as hard as she could and was eventually able to force it open, then ran all the way to the place where bids were being taken and got her bid in with one minute to spare. As usual, her bid was slightly lower than the man's, and she was awarded the contract. If the woman sues the man, what causes of action can she assert? (A) Assault, but not false imprisonment. (B) False imprisonment, but not assault. (C) Both assault and false imprisonment. (D) Neither assault nor false imprisonment.

(B) The woman has a cause of action for false imprisonment only. The woman has no cause of action for assault, because there was no act by the man that created a reasonable apprehension in the first person of immediate harmful or offensive contact. The man's words, unaccompanied by any act, constitute at most a threat of future contact. Such a threat is insufficient to create the requisite apprehension of immediate harmful or offensive contact. Because there is no assault, (A) and (C) are incorrect. The woman has a cause of action for false imprisonment because the man placed a physical barrier across the entrance to the woman's office, intending to confine the woman therein. The man's action caused the woman to be confined to the office. The fact that the woman apparently was confined for only a short time is immaterial to the false imprisonment action, as is the apparent absence of actual damages. Thus, (B) is correct and (A) and (D) are incorrect.

A teenager who was totally blind in one eye and had only 10% vision in the other could not obtain a driver's license. Nevertheless, on his 18th birthday, he borrowed his father's car and took his girlfriend for a ride. With his 10% vision in one eye, he was able to stay in the correct lane and avoid oncoming traffic, but he failed to see a jogger on the edge of the highway. The teenager's car hit the jogger, causing serious bodily injury. If the jogger brings a negligence suit against the teenager and the jury finds in the jogger's favor, what is the most likely reason? (A) The teenager failed to exercise ordinary and reasonable care under the circumstanc- es. (B) The teenager failed to exercise the amount of care that an 18-year-old of like educa- tion, intelligence, and experience would have exercised. (C) The teenager failed to exercise the ordinary and reasonable care that a person with the teenager's disability would have exercised. (D) Theteenagerviolatedthelawwhenhe drove without a license.

(C) If the jogger prevails, it will be based on failure to exercise ordinary care of someone with the teenager's vision problem. In a lawsuit based on negligence, the usual standard of care is ordinary and reasonable care under the circumstances. The standard changes when the defendant has a major physical disability such as blindness. In that situation, the standard becomes ordinary and reasonable care for a person with that disability. Therefore, (C) is a more accurate answer than (A). (B) is wrong because it states the standard to be applied to children, which would not be applied to an 18-year-old car driver. (D) is a true statement. However, the statutory violation of failure to obtain a license was not the cause of the accident.

A driver traveling the speed limit in the evening on a quiet country road rounded a curve and struck a bicyclist who was riding in the same lane. The driver stopped the car and inspected the bicyclist, who had a broken leg. The driver thought it best not to try to move the bicyclist, so he told him that he would go to get help. The driver drove away and left the bicyclist by the side of the road. After the driver had left the scene, he realized that he had forgotten his wife's birthday, so he stopped to buy a gift and hurried home. He did not remember the bicyclist until a few hours later, but assumed that by that time someone would have come along to render assis- tance. However, the bicyclist was not rescued until the following morning. By then, he had contracted pneumonia as a result of exposure. The bicyclist sued the driver to recover damages for his broken leg and the pneumonia. If the jury finds that the driver was not negli- gent in his operation of his automobile, for what harm will the bicyclist most likely recover? (A) Both the leg injury and the pneumonia. (B) The leg injury but not the pneumonia. (C) The pneumonia but not the leg injury. (D) Neither the leg injury nor the pneumonia

(C) The bicyclist will most likely recover for the pneumonia but not for the leg injury. The facts indicate that the driver was not driving negligently when the accident occurred. Therefore, he is not liable for the leg injury caused by the accident, and (A) and (B) are incorrect. However, where the defendant's actions have placed another person in peril or caused another's injury, the defendant has a duty to make reasonable efforts to rescue the imperiled person or render aid to his victim. The driver's neglect of the bicyclist after injuring him will make him liable for the resulting pneumonia. Therefore, (B) and (D) are incorrect.

A camper at a state park built a campfire within a fire ring on a calm day according to approved procedures. Just as a sudden strong wind arrived and blew some embers onto the grass, a large bear came out of the woods and charged at the camper. The camper ran to his car, which was some distance away, with the bear in close pursuit. By the time the bear left and the camper was able to exit the car and summon assistance, the embers in the grass had started a brush fire. The fire destroyed another camper's equipment and automobile at a nearby campsite before it could be extinguished. The other camper sued the camper who started the fire. At trial, the parties stipulated to the above facts. The plaintiff introduced into evidence a state statute that prohibited leaving any campfires unattended and required them to be extinguished immediately if any embers were blown out of the fire ring. At the conclusion of the proofs, both parties moved for a directed verdict. How should the court rule on the motions? (A) Grant the plaintiff's motion, because the statute was intended to prevent the type of harm that occurred, making the statutory standard applicable. (B) Grant the plaintiff's motion, because a brush fire caused by a campfire does not ordinarily happen in the absence of negli- gence by the camper. (C) Grant the defendant's motion, because the plaintiff has not established a prima facie case of negligence. (D) Deny both motions, because the jury should make the factual determination of whether the defendant was negligent.

(C) The court should grant the defendant's motion because the plaintiff has not offered sufficient evidence of negligence on the defendant's part to go to the jury. The standard of care in a negli- gence case may be established by proving that a statute imposing a specific duty applies instead of the more general common law duty of care. However, violation of the statute may be excused where compliance would cause more danger than violation or where compliance would be beyond the defendant's control. Here, the statute regulating campfires is applicable because (i) the plain- tiff, a fellow camper, is in the class intended to be protected by the statute, (ii) the statute was designed to prevent the escape of a campfire, which is what occurred here, and (iii) the statute clearly specifies what is required. However, even though the statute would apply to the defendant's conduct and the defendant violated the statute, the violation will be excused here because he was fleeing for his life from a bear and had to take refuge in his car. Hence, the defendant will not be held to the statutory standard of care here. Because the plaintiff has offered no other evidence that the defendant was negligent, the defendant's motion should be granted. (A) is incorrect because, as discussed above, even though the statute was intended to prevent the harm that occurred, the defendant's violation of the statute will be excused. (B) is incorrect because the plaintiff cannot rely on an inference of negligence here to establish breach of duty. Res ipsa loquitur permits the trier of fact to infer breach of duty where the facts strongly indicate that the plaintiff's injuries resulted from the defendant's negligence, but here the undisputed facts as to how the brush fire was caused are inconsistent with a finding of negligence. (D) is incorrect because there is neither a reasonable inference of negligence nor evidence of negligence, given that the violation of the statute will be excused here. Hence, the case should not be submitted to the jury because the plaintiff has not established a prima facie case.

A defendant intended to commit an assault on A, but his conduct only constituted a battery on B. Under the transferred intent doctrine, the defendant is liable for: A An assault of B. B An attempted assault of A and a battery of B. C A battery of B. D An attempted assault of A and an assault of B.

(C) The defendant has committed a battery of B when he acts with the intent to commit an assault on A, but his conduct only constitutes a battery on B. The transferred intent doctrine allows an intent to commit a tort against one person to be transferred to the committed tort or to the injured person. It applies to (i) assault, (ii) battery, (iii) false imprisonment, (iv) trespass to land, and (v) trespass to chattels. The defendant is not liable for an assault of B. The committed tort was a battery, and the intent transfers from the assault to the battery. Nor is the defendant liable for an attempted assault of A. There is no tort liability for an attempted assault standing alone. The defendant is liable only because of the transferred intent doctrine, and only to the person harmed. Thus, there is no liability to A here.

A hunter constructed an outfit which made him appear, from the waist up, to be a 16-point buck. Below the waist he wore camouflage pants and hiking boots. The day hunting season opened, the hunter stationed himself in a heavily populated deer area and awaited his prey. Meanwhile, a second hunter, who had had numerous alcoholic drinks before embarking on opening day hunting, caught sight of the first hunter in his deer costume and fired his shotgun. One of the bullets grazed the first hunter's head, blasting the deer mask off and knocking the hunter unconscious. The first hunter brings an action to recover damages for his personal injuries against the second hunter in a jurisdiction retaining tradi- tional contributory negligence rules. What result? (A) The second hunter will win, because he did not have the requisite intent to justify imposing liability for an intentional tort on the first hunter. (B) The second hunter will win, if he can prove that he did not have the last clear chance to avoid injuring the first hunter. (C) The first hunter will win, if he can estab- lish that the second hunter's act of hunting while intoxicated was reckless and wanton conduct. (D) The first hunter will win, if he can show that the second hunter was negligent in hunting while intoxicated.

(C) The first hunter will win if the second hunter's conduct was reckless or wanton. Because the first hunter was contributorily negligent in making himself appear to be a deer on opening day of deer hunting season, his recovery depends on the effect of his contributory negligence. Without comparative negligence principles, the first hunter's contributory negligence might bar his recovery unless he can show that the second hunter was reckless and wanton, as to which conduct contributory negligence is no defense. Thus, (C) is the correct pick. (A) is wrong because the second hunter's liability is not dependent on intentional tort theories. (B) is not the best answer because, even if the second hunter did not have the last clear chance (which overcomes contribu- tory negligence), he may still be liable if his actions are characterized as reckless and wanton. (D) is wrong because, absent the factors discussed above, the second hunter's negligence would be counterbalanced by the first hunter's contributory negligence.

What is the key difference between trespass to chattels and conversion? A The value of the chattels that are subject to the defendant's interference. B The nature of the intent on the part of the defendant. C The seriousness of the interference with the plaintiff's possession. D The means by which the defendant interferes with the plaintiff's possession.

(C) The key difference between trespass to chattels and conversion is the seriousness of the interference with the plaintiff's possession. An act by the defendant interfering with the plaintiff's right of possession in the chattel that is serious enough in nature or consequence to warrant that the defendant pay the full value of the chattel constitutes conversion. On the other hand, a less serious interference with the plaintiff's right of possession will constitute trespass to chattels (as long as the plaintiff suffered some damage). No specific rule can be stated for these situations; however, the longer the withholding period and the more extensive the use of the chattel during this time, the more likely it is that conversion has resulted. The value of the chattels that are subject to the defendant's interference is not determinative. The defendant may interfere with the plaintiff's possession of a valuable chattel but only briefly or in a minor way, and the defendant is liable only for trespass to chattels. The nature of the intent on the part of the defendant is essentially the same for both torts: the intent to perform the act that interferes with the plaintiff's right of possession. The defendant need not have intended to cause a serious interference to be liable for conversion. The means by which the defendant interferes with the plaintiff's possession need not be different for the two torts; it may be the consequences of the interference that make the difference. For example, a short joyride may amount to only trespass to chattels, but if the defendant accidentally totals the car while doing so, he will be liable for conversion.

After accounts of a confidential congres- sional hearing on a national security matter were published, the chief counsel at the hearing made a statement to a major newspaper accusing a popular network news anchorman of leaking the story and endangering national security. The network immediately fired the anchorman. When facts came to light a few weeks later showing that the allegation was not true, the anchorman was rehired and restored to his position. The anchorman sued the newspaper for defamation, claiming compensatory and punitive damages, and made allegations legally sufficient to sustain those damages if proved. No affirma- tive defenses were allowed. What is the newspaper's best defense? (A) It was not negligent in printing the chief counsel's remarks. (B) The anchorman was restored to his position within a few weeks. (C) The publication was not made with knowl- edge that it was false or with reckless disre- gard for the truth. (D) The statement was protected by the Speech and Debate Clause.

(C) The newspaper's best defense is that the publication was not made with actual malice. A public figure suing for damages in a defamation action must prove actual malice. Mere hostility or dislike of the plaintiff does not constitute actual malice. To establish actual malice, a plaintiff must show "that the utterance was false and that it was made with knowledge of its falsity or with reckless disregard of whether it was false or true." [New York Times v. Sullivan (1964)] Reckless disregard is not measured by whether a reasonable person would have investigated before publishing. There must be sufficient evidence to permit the conclusion that the defendant in fact entertained serious doubts as to the truth of the communication when it was published. (A) is incorrect because negligence is not the standard which will apply in this case to determine the defendant's fault. As the anchor of a popular news show, the anchorman is a public figure, and a public figure suing for damages in a defamation action must prove actual malice. (B) is incorrect because the fact that the anchorman was restored to his position quickly is relevant to the issue of damages, but it does not help the newspaper on the issue of liability. (D) is incorrect. The Speech and Debate Clause provides immunity from defamation, but only for communications in one of the houses of Congress. The chief counsel's statement outside of Congress would not be covered.

A pedestrian crossed the street at a crosswalk without looking for oncoming traffic. He was struck first by a car and then by a truck. The pedestrian sued both the driver of the car and the driver of the truck for negligence. The jury determined that the pedestrian was 60% at fault, the driver of the car 30%, and the truck driver 10%. The jury also determined that the pedes- trian suffered damages of $100,000. The driver of the car is insolvent. In a pure comparative negligence jurisdiction retaining traditional joint liability rules, how much can the pedestrian collect from the driver of the truck? (A) Nothing. (B) $10,000. (C) $40,000. (D) $100,000.

(C) The pedestrian can collect $40,000 from the driver. In a pure comparative negligence jurisdic- tion, the plaintiff can recover even if he was over 50% at fault. Thus, (A) is wrong. The recovery will be limited to the percentage of damage attributed to the defendant(s), in this case 40%. The jurisdiction retains the rule on joint and several liability. Therefore, each defendant is responsible for the combined liability of all defendants. (C) is therefore correct and (B) is wrong. (D) is wrong because, in a comparative negligence jurisdiction, the plaintiff's recovery is reduced due to his negligence.

While returning from transporting a group of children to summer camp, a bus driver and his assistant were caught in the leading edge of a forest fire raging down the high mountains. Hurrying ahead of the flames and smoke, the driver reached the last half-mile of a dirt road that ran to the main highway and safety, but he discovered that the road ahead was already blocked by fallen, burning foliage. Separating the driver's bus from the main highway, which angled off to the right, was the fenced property of a rancher. The bus driver drove across the property to reach the main highway, damaging some turf and a fence, and proceeded to the city. If the rancher asserts a claim against the bus driver to recover for the damage to his property, is the rancher likely to win? (A) No, because the bus driver was acting to protect the lives of himself and his assis- tant. (B) No, because the bus driver acted as would any reasonably prudent person under the circumstances. (C) Yes, because the bus driver damaged the rancher's property when he drove through the fence to get to the main highway. (D) Yes, because the bus driver intentionally drove across the property, knowing it would cause damage.

(C) The rancher will probably win. A person who intentionally intrudes upon land in the possession of another is guilty of the tort of trespass to land. The bus driver has clearly met all the elements of this tort. However, the bus driver has the defense of private necessity, because it was necessary to drive onto the rancher's land to avoid the forest fire. Although private necessity is a defense to trespass to lands, it does not relieve the bus driver of liability for damage done to the property. Hence, (A) and (B) are incorrect. Note that the call of the question merely asks what the outcome of the rancher's claim against the driver would be, not whether the driver has committed a trespass. (D) is not as good a choice as (C) because it focuses on the driver's intent in committing a trespass rather than on whether the land was damaged.

o assert the defense of property, a defendant using force against another may not: A Use force that may injure the other. B Make a mistake about the right to use force. C Use force without a request to desist. D Use force against one with a privilege to enter the property.

(D) A defendant cannot assert the defense of property if she uses force against one with a privilege to enter the property. Whenever an actor has a privilege to enter upon the land of another because of necessity, right of reentry, right to enter upon another's land to recapture chattels, etc., that privilege supersedes the privilege of the land possessor to defend her property. It is not true that the defendant may not use force that may injure the entrant. The force used must be reasonable and not likely to cause death or serious bodily injury. It is also incorrect to state that the defendant may not use force without a request to desist. A request to desist must usually precede the use of force, but if the circumstances make it clear that the request would be futile or dangerous, then a request to desist is not required. It is also incorrect to state that the defendant may not make a mistake about the right to use force. A reasonable mistake is allowed as to the property owner's right to use force in defense of property, where the mistake involves whether an intrusion has occurred or whether a request to desist is required.

The plaintiff was driving inattentively when she had to swerve to avoid two other negligently driven vehicles at a busy intersection, and her car struck a light pole. The plaintiff, who was the only driver injured, sued one of the other drivers to recover damages in a jurisdiction that has adopted pure comparative negligence. The jury determined that she suffered injuries of $100,000 and was 50% at fault. If the plaintiff is awarded a recovery of only $25,000 from the defendant, what will be the most likely reason? (A) The defendant's fault was less serious than that of the other tortfeasor. (B) The plaintiff's fault was as great as the total negligence of the other two drivers combined. (C) The jurisdiction applies contribution based on a pro rata approach rather than propor- tional fault. (D) The jurisdiction has abolished joint and several liability.

(D) If the plaintiff recovers only $25,000, it will be because the jurisdiction has abolished joint and several liability. Under joint and several liability, when two or more tortious acts combine to proximately cause an indivisible injury to a plaintiff, each tortfeasor is liable to the plaintiff for the entire damage incurred. Hence, the plaintiff could recover $50,000 from the defendant if joint and several liability applied (her total damages reduced by the amount of her own fault that contrib- uted to her injury). The facts do not state what percent of fault was assigned to the defendant, but given that the other choices are incorrect, the defendant's fault must have been determined to be 25%, so that $25,000 would be the plaintiff's recovery in the absence of joint and several liability. (A) is incorrect regardless of whether joint and several liability applies. If the defendant's fault were less serious than that of the other tortfeasor, he would be liable to the plaintiff for less than $25,000 in the absence of joint and several liability. If joint and several liability did apply, he would be liable to the plaintiff for $50,000—the full amount of her damages. (B) is incorrect because the fact that the plaintiff's fault equals the combined fault of the other two tortfeasors is relevant only in a partial comparative negligence jurisdiction. It is irrelevant in a pure comparative negligence jurisdiction. (C) is incorrect because the rule of contribution, regardless of whether it is based on a pro rata approach or a proportional fault approach, does not affect how much the plain- tiff receives from a defendant. Rather, it enables a defendant who has paid more than his share of damages to the plaintiff under joint and several liability to seek recovery against any other joint tortfeasor for the excess paid.

A college student was holding a loud party at her house. The next door neighbor was getting increasingly angry with the noise, and several complaints to the local police station brought no results. The neighbor called the student on the phone and told her that if she did not stop the noise, he would "come over there and cut your throat." The visibly shaken student told her guests what had just happened, and they all decided to leave immediately. The student was unable to sleep that night and thereafter purchased an alarm system for her house and a gun that she kept next to her bed. If the student brings an action for intentional infliction of emotional distress and succeeds, what is the most likely reason? (A) The neighbor had the apparent present abil- ity to make good on his threat. (B) The student suffered some physical harm as a result of her distress. (C) The student suffered pecuniary injury as a result of the neighbor's threat. (D) The neighbor's conduct was extreme and outrageous.

(D) If the student wins, it will be because the neighbor's conduct was extreme and outrageous. Inten- tional infliction of emotional distress requires proof that: (i) the conduct was extreme and outra- geous; (ii) it was also intentional, or at least reckless; and (iii) severe emotional distress resulted. If the threat is deemed to be extreme and outrageous, the student will prevail because the other elements are present. (A) is incorrect because an apparent present ability to make good on a threat is required for an action of assault, but it is not specifically required for an action for inten- tional infliction of emotional distress, the tort at issue here. (B) is incorrect because the require- ment of physical harm only applies in cases of negligent infliction of emotional distress. This is a case of intentional infliction of emotional distress. (C) is incorrect because pecuniary injury is not required in an action for intentional infliction of emotional distress. While severe emotional distress must be shown, an action will lie even if no actual monetary harm was caused.

Which of the following is NOT part of the prima facie case for intentional infliction of emotional distress? A Intent to cause the plaintiff to suffer severe emotional distress. B Evidence of severe emotional distress. C An act by the defendant amounting to extreme and outrageous conduct. D Physical symptoms caused by the emotional distress.

(D) Physical symptoms caused by the emotional distress are not required. A prima facie case for intentional infliction of emotional distress requires proof of: (i) an act by the defendant amounting to extreme and outrageous conduct; (ii) intent on the part of the defendant to cause the plaintiff to suffer severe emotional distress, or recklessness as to the effect of the defendant's conduct; (iii) causation; and (iv) damages—severe emotional distress. Intent to cause the plaintiff to suffer severe emotional distress establishes the intent element of the tort. Reckless disregard of a high probability that emotional distress will result also satisfies the intent element of the tort. Evidence of severe emotional distress must be shown; hurt feelings are not sufficient. An act by the defendant amounting to extreme and outrageous conduct is an element of the tort.

Two men stopped to have a drink together at a bar, where five other people were also gathered. The two men had a few drinks and began to argue, hurling insults at each other. After the first man threw a punch, a fight ensued, and the second man was struck in the back of the head with a heavy object from behind. He never saw who had struck the blow and he was knocked unconscious. He was hospitalized for a severe concussion and still suffers from severe headaches. The injured man sued the first man as well as the five other persons present in the bar at the time he was struck, seeking to recover damages for the blow to his head. He presented the evidence above. Will the injured man prevail? (A) Yes, because he was not the aggressor. (B) Yes, because the first man acted unreason- ably. (C) Yes, because of the doctrine of res ipsa loquitur. (D) No, because he cannot prove that there was concerted action against him.

(D) The injured man will not prevail. While concerted action would create joint and several liability among all parties sued by the injured man, he has presented no evidence of this and therefore cannot prevail. (A) and (B) are incorrect. The fact that the first man was the aggressor or acted unreasonably does not, by itself, impose liability on the other persons sued. (C) is incorrect because res ipsa loquitur does not apply when more than one party may have been in control of the instrumentality causing injury. It requires that the instrumentality causing the injury be in the defendant's sole control. Furthermore, res ipsa loquitur creates a prima facie case for negligence, and the striking of the second man involves an intentional tort.

A trucker owned and operated a small truck which he used commercially to haul dynamite to construction sites. Unbeknownst to the trucker, there was a hidden defect in the latch that held the rear panel of the truck. The trucker was hauling a load of dynamite one morning and exceeding the speed limit when his truck struck a bump in the road, the latch malfunctioned, and the rear panel of the truck flew open. One box of dynamite fell out of the truck and struck a pedes- trian, breaking her foot. If the pedestrian sues the trucker under strict liability for her injuries, will the pedestrian win? (A) Yes, because hauling dynamite is an abnor- mally dangerous activity. (B) Yes, because the trucker was speeding while driving with the dynamite. (C) No, because the defect in the latch was not discoverable upon reasonable inspection. (D) No, because the dynamite did not explode.

(D) The pedestrian will lose in this suit. The defendant is strictly liable for engaging in certain activities when the dangerous propensity of that activity injures the plaintiff. Although hauling dynamite is an unusually dangerous activity, it was not the dynamite's dangerous propensities that caused the accident. Instead, it was a defective latch in the truck. Thus, (A) is incorrect. (B) and (C) can be eliminated because they deal with negligence issues and the pedestrian is suing under a strict liability theory. (D) is the best answer. Note that if the dynamite had exploded, after falling out due to the defective latch, there would have been liability under a strict liability theory.

A physician performed scheduled surgery on her patient's right ear for a condition caused by prolonged and repeated infections in that ear. During the surgery, the physician determined that her patient had been particularly susceptible to this condition due to a previously unsuspected anatomical abnormality. The physician reason- ably believed that this same abnormality was likely to exist in the patient's left ear. Though the patient had not had many infections in the left ear, if a similar course of recurring infections were to transpire involving that ear, it would probably develop the same condition as the right and require surgery. The physician therefore decided to perform surgery on her patient's left ear, although she had received his consent only to operate on the right ear. The surgery was performed with due care and was successful. In an action by the patient against the physi- cian, what is the likely result? (A) The patient will not recover because the extension of the operation was successful. (B) The patient will not recover because the extension of the operation was carried out with due care. (C) The patient will recover at least nominal damages on a negligence theory. (D) The patient will recover at least nominal damages on a battery theory.

(D) Thepatientwillrecoveratleastnominaldamages.Thepatienthereneedonlyshowthattheexten- sion of the operation was an intentional, unpermitted, offensive contact in order to recover at least nominal damages in battery. The patient may recover in battery regardless of whether she was harmed. Battery is a tort where no physical harm need be shown, and no actual damage need be proven. (A) is incorrect because the patient here can show that the extension of the operation was an intentional, unpermitted, offensive contact. Therefore, the patient may recover in battery regard- less of whether or not she was harmed. Battery is a tort where no physical harm need be shown, and no actual damage need be proven. Actual harm is an element of the prima facie case of negli- gence, but is not required for most intentional torts. (B) is incorrect because the patient may recover in battery regardless of whether she was harmed and whether the physician's conduct was incompe- tent (i.e., negligent). Battery is a tort where no physical harm need be shown, and no actual damage need be proven. Also, no negligence or incompetence need be shown in battery. The patient here need only show that the extension of the operation was an intentional, unpermitted, offensive contact. (C) is incorrect because, in a negligence action, actual harm must be shown. The plaintiff must establish that she suffered damage as a result of the defendant's conduct. Damages will not be presumed and nominal damages cannot be awarded in a negligence action. Nominal damages are only appropriate in a suit based on an intentional tort where no actual harm can be shown.

Two lawyers work as partners in a small town that has only one other lawyer in it. The partners do a substantial amount of personal injury work. A client was severely and permanently injured in an automobile collision. The client employed the partners to represent her in obtaining damages from the motorist for her injuries. At the time she employed the partners, the statute of limitations on her claim had six weeks to run. The complaint was prepared but not filed. Each partner thought that the other partner would file the complaint. The statute of limitations ran on the client's claim against the motorist. The client has filed suit against the partners for negligence. That case is on trial with a jury in a court of general jurisdiction. In addition to proving that the partners were negligent, the client must establish, as a minimum, that she A: would have, but for her lawyers' negligence, recovered from the motorist. B: had a good faith claim against the motorist that was lost by her lawyers' negligence. C: was severely and permanently injured when struck by the motorist's automobile. D: did not negligently contribute to the failure to have the complaint filed.

A

A city ordinance makes it unlawful to park a motor vehicle on a city street within 10 feet of a fire hydrant. At 1:55 p.m. a man, realizing he must be in the bank before it closed at 2:00 p.m., and finding no other space available, parked his automobile in front of a fire hydrant on a city street. The man then hurried into the bank, leaving his aged neighbor as a passenger in the rear seat of the car. About five minutes later, and while the man was still in the bank, a driver was driving down the street. The driver swerved to avoid what he mistakenly thought was a hole in the street and sideswiped the man's car. The man's car was turned over on top of the hydrant, breaking the hydrant and causing a small flood of water. The man's car was severely damaged and the neighbor was badly injured. There is no applicable guest statute. If the man asserts a claim against the driver for damage to the man's automobile, the most likely result is that the man will A: recover, because the purpose of the ordinance is to provide access to the fire hydrant. B: recover, because the driver's negligence was later in time than the man's act of parking. C: not recover, because the man was contributorily negligent as a matter of law. D: not recover, because the man's action in parking unlawfully was a continuing wrong.

A

A man rented a car from a car rental agency. Unbeknownst to the rental agency, the car had a bomb hidden in it at the time of the rental. The bomb exploded an hour later, injuring the man. Immediately prior to renting the car to the man, the rental agency had carefully inspected the car to be sure it was in sound operating condition. The rental agency did not inspect for hidden explosive devices, but such an inspection would have revealed the bomb. There had been no previous incidents of persons hiding bombs in rental cars. In a negligence action by the man against the car rental agency, is the man likely to prevail? A: No, because the rental agency could not have reasonably foreseen the likelihood of someone placing a bomb in the car it was about to rent to the man. B: No, because the rental agency did not hide the bomb in the car. C: Yes, because an inspection for explosive devices would have revealed the bomb. D: Yes, because the bomb made the car abnormally dangerous.

A

A man was driving north on an interstate highway at about 50 miles per hour when a tractor-trailer rig, owned and driven by a driver, passed him. The tractor was pulling a refrigerated meat trailer fully loaded with beef carcasses hanging freely from the trailer ceiling. When the driver cut back in front of the man, the shifting weight of the beef caused the trailer to overturn. The man was unable to avoid a collision with the overturned trailer and was injured. The trailer had been manufactured by a trailer company. A number of truckers had complained to the trailer company that the design of the trailer, which allowed the load to swing freely, was dangerous. The driver knew of the dangerous propensity of the trailer. A restraining device that could be installed in the trailer would prevent the load from shifting and was available at nominal cost. The restraining device would have prevented the trailer from overturning. The driver knew of the restraining device but had not installed it. If the man asserts a claim for his injuries against the driver, he will A: prevail, because the use of a restraining device would have prevented the trailer from overturning. B: prevail, because the driver is strictly liable to the man for injuries resulting from defects in the trailer. C: not prevail because the driver was not driving in a negligent manner at the time the man was injured. D: not prevail, because the driver was not the manufacturer or seller of the trailer.

A

A truck driver was driving along a lonely road on a very cold night. He saw a man lying in a field by the side of the road and apparently injured. The truck driver stopped his truck, alighted, and, upon examination, discovered that the man was intoxicated and in danger of suffering from exposure to the cold. However, the truck driver returned to his truck and drove away without making any effort to help the man, who remained lying at the same place and was later injured when struck by a car driven by a traveler who, drowsy and inattentive, had veered off the road into the field and hit the man. The traveler did not see the man prior to hitting him. If the man asserts a claim against the traveler, will the man prevail? A: Yes, because the traveler was negligent in going off the road. B: Yes, because the man was in a helpless condition. C: No, because the traveler did not see the man before he was struck. D: No, because the man's intoxication was the cause in fact of his harm.

A

An eight-year-old child went to the grocery store with her mother. The child pushed the grocery cart while her mother put items into it. The child's mother remained near the child at all times. Another customer in the store noticed the child pushing the cart in a manner that caused the customer no concern. A short time later, the cart the child was pushing struck the customer in the knee, inflicting serious injury. If the customer brings an action, based on negligence, against the child, the child's best argument in defense would be that A: The child exercised care commensurate with her age, intelligence, and experience. B: The child is not subject to tort liability. C: The child was subject to parental supervision. D: The customer assumed the risk that the child might hit the customer with the cart.

A

In a civil action, a plaintiff sued a decedent's estate to recover damages for injuries she suffered in a collision between her car and one driven by the decedent. At trial, the plaintiff introduced undisputed evidence that the decedent's car had swerved across the centerline of the highway into oncoming traffic, where it had collided with the plaintiff's car. The decedent's estate introduced undisputed evidence that, before he swerved across the centerline, the decedent had suffered a fatal heart attack, which he had no reason to foresee, and that, just prior to the heart attack, the decedent had been driving at a reasonable speed and in a reasonable manner. A statute makes it a traffic offense to cross the median of a highway. In this case, which party is likely to prevail? A: The decedent's estate, because its rebuttal evidence is undisputed. B: The decedent's estate, because the plaintiff has not established a prima facie case of liability. C: The plaintiff, because the accident was of a type that does not ordinarily happen in the absence of negligence on the actor's part. D: The plaintiff, because the decedent crossed the median in violation of the statute.

A

A bystander who witnesses the defendant negligently injuring another can recover for negligent infliction of emotional distress in most states by showing: A A close relationship between the bystander and the person injured, the bystander's presence at the scene of the injury, and the bystander's observation or perception of the event. B A close relationship between the bystander and the person injured, the bystander's presence within the zone of danger from physical injury, and the bystander's observation or perception of the event. C The bystander's presence within the zone of danger from physical injury, and the bystander's observation or perception of the event. D A close relationship between the bystander and the person injured, and the bystander's observation or perception of the event.

A bystander must show a close relationship between the bystander and the person injured, the bystander's presence at the scene of the injury, and the bystander's observation or perception of the event to recover. Traditionally, a bystander outside the "zone of danger" of physical injury who sees the defendant negligently injuring another could not recover damages for her own distress. A majority of states now allow recovery in these cases as long as (i) the plaintiff and the person injured by the defendant are closely related; (ii) the plaintiff was present at the scene of the injury; and (iii) the plaintiff personally observed or perceived the event. The bystander's presence within the zone of danger from physical injury is no longer required for a bystander to recover for witnessing an injury to another.

Which of the following is true of the duty owed to a licensee by a landowner? A The landowner must inspect for dangerous conditions on the land. B The landowner owes a duty to warn of or make safe known dangerous conditions on the land of which the licensee is not aware. C The landowner owes no duty to protect the licensee from active operations on the land. D The landowner must repair known dangerous conditions on the land of which the licensee is not aware.

A landowner owes a duty to a licensee to warn of or make safe known dangerous conditions on the land of which the licensee is not aware. A licensee is one who enters on land with the landowner's permission, express or implied, for her own purpose or business, rather than for the landowner's benefit. The owner has a duty to warn of or make safe a dangerous condition known to the owner that creates an unreasonable risk of harm to the licensee and that the licensee is unlikely to discover. As to a licensee, the landowner does not need to inspect for dangerous conditions on the land. Similarly, the landowner need not repair known dangerous conditions on the land of which the licensee is not aware; a warning generally will suffice. The landowner does have a duty to protect the licensee from active operations on the land. The owner has a duty to exercise reasonable care in the conduct of active operations for the protection of a licensee whom he knows to be on the property.

A plaintiff who is 40% negligent can recover 60% of his damages in which of the following cases? A In either a pure or a partial comparative negligence jurisdiction, regardless of the number of defendants and their degree of fault. B In a pure comparative negligence jurisdiction only. C In a partial comparative negligence jurisdiction, but only against a defendant who is more at fault than the plaintiff. D In a partial comparative negligence jurisdiction, but only if there is only one defendant.

A plaintiff who is 40% at fault can recover in either a pure or a partial comparative negligence jurisdiction, regardless of the number of defendants and their degree of fault. In a pure comparative negligence state, the plaintiff can recover no matter how great her degree of fault. In a partial comparative negligence jurisdiction, the plaintiff's recovery will be barred if her negligence passes a threshold level, either 50% or 51%. Hence, the plaintiff here can recover in a pure comparative negligence jurisdiction and in any partial comparative negligence jurisdiction. For that reason, it is incorrect to state that the plaintiff can recover in a pure comparative negligence jurisdiction only. In a partial comparative negligence jurisdiction, if several defendants have contributed to a plaintiff's injury, a "combined comparison" approach is used to determine the threshold level; i.e., the plaintiff's negligence is compared with the total negligence of all of the defendants combined. Thus, a plaintiff who is 40% at fault can recover in a partial comparative negligence jurisdiction even if there is more than one defendant, and may recover against a defendant who is also less at fault than she; the plaintiff will still be less at fault than the combined fault of the defendants.

A principal will be vicariously liable for the tortious acts of her independent contractor: A If the independent contractor is engaged in inherently dangerous activities. B Under the doctrine of respondeat superior. C If the principal negligently selected the independent contractor. D If the principal negligently supervised the independent contractor.

A principal will be vicariously liable for the tortious acts of her independent contractor if the independent contractor is engaged in inherently dangerous activities. In general, a principal will not be vicariously liable for tortious acts of an independent contractor. Two broad exceptions exist, however: (i) the independent contractor is engaged in inherently dangerous activities, e.g., excavating next to a public sidewalk, blasting; or (ii) the duty, because of public policy considerations, is simply nondelegable, e.g., the duty of a business to keep its premises safe for customers. Respondeat superior is the doctrine that makes employers vicariously liable for the torts of employees; it does not apply to independent contractors. A principal may be liable for negligently selecting or supervising an independent contractor. However, that liability is for her own negligence; it is not vicarious liability.

A duty of care is generally NOT owed to: A A rescuer, unless the defendant negligently put herself or a third person in peril. B A viable fetus. C A third party for whose economic benefit a legal or business transaction is made. D A discovered trespasser.

A rescuer is a foreseeable plaintiff and is owed a duty of care as long as the rescue is not wanton, but only if the defendant negligently put herself or a third person in peril. However, firefighters and police officers may be barred by the "firefighter's rule" from recovering for injuries caused by the risks of a rescue. A duty of care is owed to a viable fetus; prenatal injuries are actionable. A third party for whose economic benefit a legal or business transaction is made is owed a duty of care if the defendant could reasonably foresee harm to that party if the transaction is done negligently. While a landowner owes no duty to an undiscovered trespasser, a trespasser whose presence on the property is known to the landowner is owed a duty to be warned about seriously dangerous artificial conditions on the property.

If the plaintiff establishes res ipsa loquitur, it will have the following effect: A. A directed verdict will not be given for the defendant B. The burden of proof is shifted to the defendant C. A presumption of negligence is created D. A directed verdict will be given for the plaintiff

A. A directed verdict will not be given for the defendant

Which of the following is correct for the defense of contributory negligence? A. A failure to mitigate damages is not contributory negligence B. The standard of care for contributory negligence is greater than that for ordinary negligence C. A plaintiff can never recover if he is found to be contributorily negligent

A. A failure to mitigate damages is not contributory negligence

Which of the following persons is considered to be an invitee of the landowner? A. A child accompanying a customer of the landowner B. A hiker hiking on the landowner's open land with permission C. A customer of the landowner who goes through a door marked "employees only" D. A firefighter fighting a fire on the landowner's property

A. A child accompanying a customer of the landowner

Which of the following is correct regarding proximate cause in negligence? A. A defendant is not liable for the harmful results of his conduct that are unforeseeable B. Proximate cause is not required for a defendant to be liable, but actual cause is required C. A defendant is liable for all harmful results caused by his acts D. Proximate cause is required only when an intervening force contributes to the injury

A. A defendant is not liable for the harmful results of his conduct that are unforeseeable

In determining actual cause in a negligence action, the burden of proof shifts to the defendant under the __________. A. Alternative causes approach B. Substantial factor test C. "But for" test D. Joint causes approach

A. Alternative causes approach

Which of the following is correct regarding a landowner's duty to an invitee under traditional landowner liability rules? A. An invitee may lose that status if she exceeds the scope of her invitation B. A firefighter engaged in fighting a fire on the landowner's premises is an invitee C. The landowner does not owe an invitee a duty to make inspections D. The landowner must repair dangerous conditions to satisfy his duty to invitees

A. An invitee may lose that status if she exceeds the scope of her invitation

For the last clear chance doctrine to operate, the defendant must have been able to avoid harming the plaintiff ____________. A. At the time of the accident B. Prior to the time of the accident C. At any point in time

A. At the time of the accident

Which of the following generally is recoverable as damages in a negligence action? A. Damages that were recouped from insurance or other sources B. Attorneys' fees C. Interest from the date of damage in a personal injury action D. Additional damages caused by the failure to mitigate damages

A. Damages that were recouped from insurance or other sources

A diabetic woman saw a physician regarding deep infection and circulation loss in her small toe. The woman consented to the physician's debridement of the deep infection to see if the toe could be saved from amputation. With the woman under general anesthetic, the physician attempted debridement but decided to amputate the toe. The woman was mortified when she discovered the amputation. She retained a lawyer to sue the physician. The lawyer contacted several physicians all of whom declined to testify as liability experts in the matter. Which of the following is the lawyer's best course? A. File suit without expert testimony on liability. B. Do not file suit until a liability expert is found. C. Terminate the representation for lack of a claim. D. Consult another lawyer who might find an expert.

A. File suit without expert testimony on liability.

An art museum hired a security company to protect its new building and grounds from vandalism. Vandals caused substantial damage to the museum's new building and grounds on a day that the security company's guard had failed to show up for work at the museum. The museum filed a negligence action against the security company for the damage. The security company's lawyer moved to dismiss the action on the basis of superseding cause. Which of the following is the best evaluation of the merits of that motion? A. It will be denied because the vandalism's foreseeability was what made the company negligent. B. It will be denied because the substantial nature of the damages due to vandalism warrants compensation. C. It will be granted because the vandalism was intentional and criminal, and therefore less likely foreseeable. D. It will be granted because security companies cannot prevent every act of vandalism and should not have to pay for the loss.

A. It will be denied because the vandalism's foreseeability was what made the company negligent.

An elderly woman had multiple minor health problems. She frequently sought help for those minor health problems from a neighbor physician, who would tell her where she could obtain medical assistance. On one occasion, the elderly woman called and left a telephone message asking the neighbor physician for help, but the neighbor physician did not respond. Family members later found the elderly woman comatose in her home, due to a condition the symptoms of which she had reported to the physician in her telephone message. The family members, who were distraught that the neighbor physician had not helped the elderly woman, consulted a lawyer regarding the neighbor physician's liability to her. Which of the following is the best evaluation? A. No liability. B. No liability unless the physician had the special knowledge and skills to have helped the elderly woman based on her telephone message to him. C. Liability unless the physician did not get the telephone message before the elderly woman lapsed into a coma. D. Liability.

A. No liability.

Under the last clear chance doctrine, a defendant will be liable despite the plaintiff's inattentive peril ________. A. Only if she had actual knowledge of the peril B. Under no circumstances C. If she had actual knowledge or should have known of the peril

A. Only if she had actual knowledge of the peril

What is the effect of comparative negligence on other doctrines in most states? A. The defense of express assumption of risk is retained B. A plaintiff's negligence is a defense to the defendant's intentional tort C. There is no effect on the last clear chance doctrine

A. The defense of express assumption of risk is retained

A young woman loved visiting with her neighbor while her neighbor worked in her garden. On one of those occasions, the young woman broke her ankle when a loose walkway plank in her neighbor's garden suddenly twisted. The neighbor knew, but the young woman did not know, that the plank was loose. The young woman asked a lawyer to contact the neighbor's homeowner's insurer regarding liability for the young woman's medical expense and wage loss due to her ankle injury. Which of the following best describes how the lawyer should state the neighbor's duty to the young woman, in a jurisdiction that follows the traditional premises-liability classifications? A. The neighbor owed the young woman the duty to warn of known hidden dangers creating unreasonable risk of harm. B. The neighbor owed the young woman a duty of reasonable care to keep the premises in reasonably safe condition. C. The neighbor owed the young woman a duty to make sure that planks in the walkway were safe on which to walk. D. The neighbor owed the young woman a duty not to allow her to come over and talk if the young woman was going to get hurt.

A. The neighbor owed the young woman the duty to warn of known hidden dangers creating unreasonable risk of harm.

If a statute providing for a criminal penalty is applicable to a common law negligence case, the statute's specific duty will replace the more general common law duty of care. Which of the following does a plaintiff not need to show to prove the availability of the statutory standard? A. The plaintiff suffered physical injury because of the defendant's violation of the statute. B. The standards set out in the statute are clearly defined. C. The statute was designed to prevent the type of harm that the plaintiff suffered D. The plaintiff is in the class intended to be protected by the statute

A. The plaintiff suffered physical injury because of the defendant's violation of the statute

Which of the following statements is not true under the rule that the tortfeasor takes the victim as he finds him? A. The rule applies to the victim's existing physical condition but not his mental condition B. The rule is also known as the "eggshell-skull plaintiff" rule C. The rule applies in both direct cause cases and indirect cause cases D. The unforeseeable severity of the plaintiff's harm does not relieve the defendant of liability

A. The rule applies to the victim's existing physical condition but not his mental condition

A crowd gathered outside of a store early in the morning on Black Friday, the day after Thanksgiving, the busiest shopping day of the year. When the store opened at 9 a.m., a man rushed in to be the first shopper to purchase sale merchandise. An employee had left an extension cord on the floor from last-minute vacuuming. In the crush of shoppers, the man tripped over the extension cord and injured himself. Which of the following allegations best describes how the man's lawyer should plead his negligence action in a jurisdiction following the traditional premises-liability classifications? A. The store breached its duty, owed an invitee, of reasonable care. B. The store breached its duty, owed a licensee, to cure hidden defects. C. The store breached its duty, owed a licensee, to warn of hidden defects. D. The store breached its duty, owed anyone, to warn of known hidden danger.

A. The store breached its duty, owed an invitee, of reasonable care.

For comparative negligence jurisdictions, which situation requires consideration of the reasonableness of the plaintiff's conduct? A. When the defendant's initial breach of duty to the plaintiff is superseded by the plaintiff's assumption of a risk B. When the plaintiff has expressly assumed the risk C. When the defendant has only a limited duty to the plaintiff because of the plaintiff's knowledge of the risks

A. When the defendant's initial breach of duty to the plaintiff is superseded by the plaintiff's assumption of a risk

There often is more than one cause for an injury The "but for" test for actual cause applies to ___________. A. concurrent causes B. superseding causes C. alternative causes D. joint causes

A. concurrent causes

Which of the following correctly states the duty of a landowner? A. The landowner's duty to warn of or make safe highly dangerous artificial conditions applies only to those trespassers he discovers on the land B. The landowner has a duty to inspect his land for the presence of trespassers C. The landowner owes the same duty to anticipated trespassers as he does to discovered trespassers

C. The landowner owes the same duty to anticipated trespassers as he does to discovered trespassers

A company operated an installation for distributing sand and gravel. The installation was adjacent to a residential area. On the company's grounds, there was a chute with polished metal sides for loading sand and gravel into trucks. The trucks being loaded stopped on the public street below the chute. After closing hours, a plywood screen was placed in the chute and the ladder used for inspection was removed to another section of the installation. For several months, however, a number of children, eight to 10 years of age, had been playing on the company's property and the adjoining street after closing hours and had discovered the chute could be used as a slide. The company knew of this activity. One evening, as children were playing in the chute, a commuter driving by the chute hit an eight-year-old boy who slid down in front of the automobile. The commuter applied her brakes, but they suddenly failed, and she hit and injured the child. The commuter saw the child in time to have avoided hitting him if her brakes had worked properly. Two days earlier, the commuter had taken her car to a mechanic to have her brakes inspected, and the mechanic had told her that the brakes were in perfect condition. Claims were asserted on behalf of the child by his proper legal representative against the company, the commuter, and the mechanic. With respect to the child's claim against the commuter, the commuter's best defense is that A: her conduct was not the cause in fact of the harm. B: she used reasonable care in the maintenance of her brakes. C: she could not reasonably foresee the child's presence in the street. D: she did not act willfully and wantonly.

B

A company operated an installation for distributing sand and gravel. The installation was adjacent to a residential area. On the company's grounds, there was a chute with polished metal sides for loading sand and gravel into trucks. The trucks being loaded stopped on the public street below the chute. After closing hours, a plywood screen was placed in the chute and the ladder used for inspection was removed to another section of the installation. For several months, however, a number of children, eight to 10 years of age, had been playing on the company's property and the adjoining street after closing hours. The children found the ladder and also discovered that they could remove the plywood screen from the chute and slide down to the street below. The company knew of this activity. One evening, as children were playing in the chute, a commuter driving by the chute hit an eight-year-old boy who slid down in front of the automobile. The commuter applied her brakes, but they suddenly failed, and she hit and injured the child. The commuter saw the child in time to have avoided hitting him if her brakes had worked properly. Two days earlier, the commuter had taken her car to a mechanic to have her brakes inspected, and after a negligent inspection, the mechanic had told her that the brakes were in perfect condition. Claims were asserted on behalf of the child by his proper legal representative against the company, the commuter, and the mechanic. On the child's claim against the mechanic, will the child prevail? A: Yes, because the mechanic is strictly liable in tort. B: Yes, because the mechanic was negligent in inspecting the commuter's brakes. C: No, because the child was in the legal category of a bystander. D: No, because the company's conduct was an independent and superseding cause.

B

A pedestrian started north across the street in a clearly marked north-south crosswalk with the green traffic light in her favor. The pedestrian was in a hurry, and so before reaching the north curb on the street, she cut to her left diagonally across the street to the east-west crosswalk and started across it. Just after reaching the east-west crosswalk, the traffic light turned green in her favor. She proceeded about five steps further across the street to the west in the crosswalk when she was struck by a car approaching from her right that she thought would stop, but did not. The car was driven by a driver, 81 years of age, who failed to stop his car after seeing that the traffic light was red against him. The pedestrian had a bone disease, resulting in very brittle bones, that is prevalent in only 0.02 percent of the population. As a result of the impact the pedestrian suffered a broken leg and the destruction of her family heirloom, a Picasso original painting that she was taking to her bank for safekeeping. The painting had been purchased by the pedestrian's grandmother for $750 but was valued at $500,000 at the time of the accident. A pedestrian has filed suit against the driver. The driver's attorney has alleged that the pedestrian violated a state statute requiring that pedestrians stay in crosswalks, and that if the pedestrian had not violated the statute she would have had to walk 25 feet more to reach the impact point and therefore would not have been at a place where she could have been hit by the driver. The pedestrian's attorney ascertains that there is a statute as alleged by the driver, that his measurements are correct, that there is a state statute requiring observance of traffic lights, and that the driver's license expired two years prior to the collision. The failure of the driver to have a valid driver's license has which of the following effects? A: It makes the driver liable to the pedestrian because the driver is a trespasser on the highway. B: It would not furnish a basis for liability. C: It proves that the driver is an unfit driver in this instance. D: It makes the driver absolutely liable for the pedestrian's injury.

B

A plaintiff suffered from a serious, though not immediately life-threatening impairment of his circulatory system. The plaintiff's cardiologist recommended a cardiac bypass operation and referred the plaintiff to a surgeon. The surgeon did not inform the plaintiff of the 2% risk of death associated with this operation. The surgeon defended his decision not to mention the risk statistics to the plaintiff because the plaintiff "was a worrier and it would significantly lessen his chance of survival to be worried about the nonsurvival rate." The surgeon successfully performed the bypass operation and the plaintiff made a good recovery. However, when the plaintiff learned of the 2% risk of death associated with the operation, he was furious that the surgeon had failed to disclose this information to him, saying that he would have refused the operation if he had known of the risk. If the plaintiff asserts a claim against the surgeon based on negligence, will the plaintiff prevail? A: No, because the surgeon used his best personal judgment in shielding the plaintiff from the risk statistic. B: No, because the operation was successful and the plaintiff suffered no harm. C: Yes, because the plaintiff would have refused the operation had he been informed of the risk. D: Yes, because a patient must be told the risk factors associated with a surgical procedure in order to give informed consent.

B

A water pipe burst in the basement of a grocery store, flooding the basement and damaging cases of canned goods on the floor. The plumbing contractor's workmen, in repairing the leak, knocked over several stacks of canned goods in cases, denting the cans. After settling its claims against the landlord for the water leak and against the plumbing contractor for the damage done by his workmen, the grocery store put the goods on special sale. Four weeks later, a customer was shopping in the grocery store. Several tables in the market were covered with assorted canned foods, all of which were dirty and dented. A sign on each of the tables read: "Damaged Cans - Half Price." The customer was having a guest over for dinner that evening and purchased two dented cans of tuna, packed by a canning company, from one of the tables displaying the damaged cans. Before the guest arrived, the customer prepared a tuna casserole which she and the guest later ate. Both became ill, and the medical testimony established that the illness was caused by the tuna's being unfit for consumption. The tuna consumed by the customer and the guest came from the case that was at the top of one of the stacks knocked over by the workmen. The tuna in undamaged cans from the same canning company's shipment was fit for consumption. If the guest asserts a claim against the grocery store, the most likely result is that the guest will A: recover on the theory of res ipsa loquitur. B: recover on the theory of strict liability. C: not recover, because the grocery store gave proper warning. D: not recover, because the guest was not the purchaser of the cans.

B

Section 1 of the Vehicle Code of a state makes it illegal to cross a street in a central business district other than at a designated crosswalk. Section 2 of the Code prohibits parking any motor vehicle so that it blocks any part of a designated crosswalk. A pedestrian wanted to cross Main Street in the central business district of a city, located in the state, but a truck parked by a trucker was blocking the designated crosswalk. The pedestrian stepped out into Main Street and carefully walked around the back of the truck. The pedestrian was struck by a motor vehicle negligently operated by a driver. If the pedestrian asserts a claim against the driver, the pedestrian's failure to be in the crosswalk will have which of the following effects? A: It is not relevant in determining the right of the pedestrian. B: It may be considered by the trier of fact on the issue of the driver's liability. C: It will be considered an express assumption of risk. D: It will bar the pedestrian's recovery as a matter of law.

B

Football player injures knee, waits to seek treatment, Doctor negligently conducts surgery, then football player doesn't follow post-op instructions. Player loses career & has emotional distress. Which will the court NOT consider: delay of treatment, failure to follow post-op instructions, loss of career, or emotional distress?

Delay of treatment. Future earnings & emotional distress are both recoverable b/c there's a physical injury. Delay of treatment isn't relevant b/c doctor agreed to fix the already aggravated injury.

A developer purchased farmland for residential development. The farmland had an old barn on the property that wind and weather had seriously damaged over the years and looked to be in the process of falling down. A strong wind carried a section of the barn onto an expensive motor home parked on a neighbor's property, nearly destroying the motor home. The neighbor sued the developer, who retained counsel to defend. Which of the following facts would be most helpful to the developer's defense? A. Expert investigation determined the barn to have been unsafe for occupation. B. Expert investigation determined the strong wind to have been of tornado force. C. Neighbors had previously asked the developer to take down the unsightly barn. D. The developer had scheduled the barn for demolition the day after the strong wind.

B. Expert investigation determined the strong wind to have been of tornado force.

A company leased land from its owner to operate a gas station. The company installed gasoline pumps and operated the station for half of the 30-year lease. A gasoline pump exploded, seriously injuring several customers at the station. Investigation established that the pump exploded due to poor maintenance. After the company went bankrupt, a customer who was injured in the explosion sued the landowner. The landowner's lawyer moved to dismiss the case. How should the court rule? A. Grant the motion based on no causation. B. Grant the motion based on no duty. C. Grant the motion based on no jurisdiction. D. Deny the motion.

B. Grant the motion based on no duty.

An internist performed abdominal surgery on a patient with clear indications of appendicitis. The internist used cottonoids to pack the surgical site and keep the operative field clear of blood during the surgery. The patient developed an infection and continued to have abdominal pain after the surgery. Follow-up radiographs of the area showed a cottonoid's radiopaque marker, indicating that the internist had mistakenly left the cottonoid in the patient when completing the surgery. The internist's mistake made a second surgery necessary. In a trial of the patient's malpractice case against the internist, the patient presented these proofs without expert testimony as to the customary practice to ensure removal of cottonoids. The internist's lawyer moved for directed verdict. Which of the following is the most appropriate ruling? A. Motion denied because jurors determine reasonableness under all circumstances. B. Motion denied because no expert testimony is required for obvious breaches. C. Motion granted because expert testimony is required to establish the custom. D. Motion granted because the internist did what was necessary in the second surgery.

B. Motion denied because no expert testimony is required for obvious breaches.

A feed mill stored fertilizer in a locked and secure shed. A worker negligently left the shed unlocked. Finding the shed conveniently unlocked, a thief stole the fertilizer and made a bomb out of it, destroying a business with the bomb. The business' owner consulted a lawyer over whether to sue the feed mill in negligence for leaving the shed unlocked. Which of the following is the best evaluation? A. Strong on breach, strong on cause in fact, strong on proximate cause. B. Strong on breach, strong on cause in fact, weak on proximate cause. C. Weak on breach, weak on cause in fact, strong on proximate cause. D. Weak on breach, weak on cause in fact, weak on proximate cause.

B. Strong on breach, strong on cause in fact, weak on proximate cause.

Which of the following is a duty owed to a trespasser by a landowner? a. To warn a discovered trespasser of highly dangerous natural conditions that are unknown to the trespasser B. To warn an anticipated trespasser of highly dangerous artificial conditions known to the landowner C. To make reasonable inspections to identify highly dangerous artificial conditions D. To ascertain whether trespassers are coming onto his property

B. To warn an anticipated trespasser of highly dangerous artificial conditions known to the landowner

A woman consented to a vein-stripping procedure by a physician. The woman suffered a stroke as a result of the procedure. The woman did not know that stroke is a common risk of vein stripping about which physicians ordinarily advise patients. The woman would not have consented to the procedure if she had known of that risk. The woman consulted a lawyer regarding potential claims against the physician. Which of the following is the best evaluation of the claim? A. Valid claim for negligently causing a stroke. B. Valid claim for failure to obtain informed consent. C. No claim because stroke is a common risk. D. No claim because the procedure worked.

B. Valid claim for failure to obtain informed consent.

The owner of a residence planted an ornamental tree to hide a stop sign from the view from his living-room window. The branches of the ornamental tree gradually grew to hide the stop sign from motorists using the lane on which the residence was located. A motorist touring the lane ran the stop sign because it was hidden by the tree. A collision ensued in which another person was injured. That person sued the motorist, who retained a lawyer to answer and defend. The lawyer asked her law clerk to draft a research memorandum on whether the motorist should third-party in the owner of the residence. What should the memorandum conclude? A. Yes because landowners usually have liability insurance that should cover a portion of the losses due to the motorist's own negligence. B. Yes because landowners may owe limited duties to protect passersby from artificial conditions on the premises creating unreasonable dangers. C. No because landowners owe duties only with respect to conditions on their own premises that injure individuals on their own premises. D. No because landowners have nothing to do with the safety of motorists driving by the lands and accidents that they may cause.

B. Yes because landowners may owe limited duties to protect passersby from artificial conditions on the premises creating unreasonable dangers.

A contributorily negligent plaintiff cannot rely on the last clear chance doctrine when she was in a position of ________ peril and the defendant _____________ of her predicament. A. inattentive; knew B. inattentive; should have known C. helpless; knew D. helpless; should have known

B. inattentive; should have known

A car owner washed her car while it was parked on a public street, in violation of a local ordinance that prohibits the washing of vehicles on public streets during specified hours. The statute was enacted only to expedite the flow of automobile traffic. Due to sudden and unexpected cold weather, the car owner's waste water formed a puddle that froze in a crosswalk. A pedestrian slipped on the frozen puddle and broke her leg. The pedestrian sued the car owner to recover for her injury. At trial, the only evidence the pedestrian offered as to negligence was the car owner's admission that she had violated the ordinance. At the conclusion of the evidence, both parties moved for a directed verdict. How should the trial judge proceed? A: Deny both motions and submit the case to the jury, because, on the facts, the jury may infer that the car owner was negligent. B: Deny both motions and submit the case to the jury, because the jury may consider the ordinance violation as evidence that the car owner was negligent. C: Grant the car owner's motion, because the pedestrian has failed to offer adequate evidence that the car owner was negligent. D: Grant the pedestrian's motion, because of the car owner's admitted ordinance violation.

C

A city ordinance makes it unlawful to park a motor vehicle on a city street within 10 feet of a fire hydrant. At 1:55 p.m. a man, realizing he must be in the bank before it closed at 2:00 p.m., and finding no other space available, parked his automobile in front of a fire hydrant on a city street. The man then hurried into the bank, leaving his aged neighbor as a passenger in the rear seat of the car. About five minutes later, and while the man was still in the bank, a driver was driving down the street. The driver swerved to avoid what he mistakenly thought was a hole in the street and sideswiped the man's car. The man's car was turned over on top of the hydrant, breaking the hydrant and causing a small flood of water. The man's car was severely damaged and the neighbor was badly injured. There is no applicable guest statute. If the neighbor asserts a claim against the man, the most likely result is that the neighbor will A: recover, because the man's action was negligence per se. B: recover, because the man's action was a continuing wrong which contributed to the neighbor's injuries. C: not recover, because a reasonably prudent person could not foresee injury to the neighbor as a result of the man's action. D: not recover, because a violation of a city ordinance does not give rise to a civil cause of action.

C

A doctor resided in her own home. The street in front of the home had a gradual slope. The doctor's garage was on the street level, with a driveway entrance from the street. At two in the morning, the doctor received an emergency call. She dressed and went to the garage to get her car and found a car parked in front of her driveway. That car was occupied by a man, who, while intoxicated, had driven to that place and now was in a drunken stupor in the front seat. Unable to rouse the man, the doctor pushed him into the passenger's side of the front seat and got in on the driver's side. The doctor released the brake and coasted the car down the street, planning to pull into a parking space that was open. When the doctor attempted to stop the car, the brakes failed to work, and the car crashed into the wall of a homeowner's home, damaging the home and the man's car and injuring the doctor and the man. Subsequent examination of the car disclosed that the brake linings were badly worn. A state statute prohibits the operation of a motor vehicle unless the brakes are capable of stopping the vehicle within specified distances at specified speeds. The brakes on the man's car were incapable of stopping the vehicle within the limits required by the statute. Another state statute makes it a criminal offense to be intoxicated while driving a motor vehicle. If the man asserts a claim against the doctor for his injuries, the man will probably A: recover, because the doctor was negligent as a matter of law. B: recover, because the doctor had no right to move the car. C: not recover, because his brakes were defective. D: not recover, because he was in a drunken stupor when injured.

C

A mother rushed her eight-year-old daughter to the emergency room at a local hospital after the child fell off her bicycle and hit her head on a sharp rock. The wound caused by the fall was extensive and bloody. The mother was permitted to remain in the treatment room, and held the child's hand while the emergency room physician cleaned and sutured the wound. During the procedure, the mother said that she was feeling faint and stood up to leave the room. While leaving the room, the mother fainted and, in falling, struck her head on a metal fixture that protruded from the emergency room wall. She sustained a serious injury as a consequence. If the mother sues the hospital to recover damages for her injury, will she prevail? A: Yes, because the mother was a public invitee of the hospital's. B: Yes, because the fixture was not an obvious, commonly used, and essential part of the hospital's equipment. C: No, because there is no evidence that the hospital's personnel failed to take reasonable steps to anticipate and prevent the mother's injury. D: No, because the hospital's personnel owed the mother no affirmative duty of care.

C

A patient had been under the care of a cardiologist for three years prior to submitting to an elective operation that was performed by a surgeon. Two days thereafter, the patient suffered a stroke, resulting in a coma, caused by a blood clot which formed after the operation. When it appeared that she had entered a permanent vegetative state, with no hope of recovery, the artificial life-support system that had been provided was withdrawn, and she died a few hours later. The withdrawal of artificial life support had been requested by her family, and duly approved by a court. The surgeon was not involved in that decision, or in its execution. The administrator of the patient's estate thereafter filed a wrongful death action against the surgeon, claiming that the surgeon was negligent in having failed to consult a cardiologist prior to the operation. At the trial the plaintiff offered evidence that accepted medical practice would require examination of the patient by a cardiologist prior to the type of operation that the surgeon performed. In this action, the plaintiff should A: prevail, because the surgeon was negligent in failing to have the patient examined by a cardiologist prior to the operation. B: prevail, because the blood clot that caused the patient's death was caused by the operation which the surgeon performed. C: not prevail, because there is no evidence that a cardiologist would have provided advice that would have changed the outcome if one had examined the patient before the operation. D: not prevail, because the surgeon had nothing to do with the withdrawal of artificial life support, which was the cause of the patient's death.

C

A patient who had suffered a severe fracture of her leg was treated by an orthopedist, who set the patient's leg and put it in a cast. When the leg continued to bother the patient six months later, she consulted a second orthopedist in the same town. The second orthopedist surgically inserted a pin to facilitate healing. The patient brought a malpractice action against the first orthopedist, claiming that he should have surgically inserted a pin at the time of initial treatment. The only evidence that the patient offered in support of her malpractice claim was the testimony of the second orthopedist, as follows: In response to the question "Would you have inserted a pin initially?" the second orthopedist testified, "I personally would not have been satisfied that the leg would heal properly without a pin." At the close of the patient's evidence, the first orthopedist moved for judgment as a matter of law. Should the motion be granted? A: No, because the patient has introduced evidence that the first orthopedist failed to give the care that the second orthopedist would have provided. B: No, because the second orthopedist practices in the same town and field of specialty as the first orthopedist. C: Yes, because the patient has failed to introduce evidence that the first orthopedist's care fell below the professional standard of care. D: Yes, because the second orthopedist also treated the patient and is thus not sufficiently objective to offer expert testimony.

C

For the doctrine of res ipsa loquitur to apply, the plaintiff must establish that _________. (A) The defendant's breach of duty was the sole cause of the plaintiff's injury (B) The defendant possessed the instrumentality that caused the injury (C) The accident would not normally occur unless someone was negligent (D) The defendant violated a statute establishing a standard of care

C

Two lawyers work as partners in a small town that has only one other lawyer in it. The partners do a substantial amount of personal injury work. A client was severely and permanently injured in an automobile collision. The client employed the partners to represent her in obtaining damages from the motorist for her injuries. At the time she employed the partners, the statute of limitations on her claim had six weeks to run. The complaint was prepared but not filed. Each partner thought the other partner would file the complaint. The statute of limitations ran on the client's claim against the motorist. The client has filed suit against the partners for negligence. That case is on trial with a jury in a court of general jurisdiction. In order to establish a breach of the standard of care owed to her by the partners, the client A: must have a legal expert from the same locality testify that defendants' conduct was a breach. B: must have a legal expert from the same state testify that defendants' conduct was a breach. C: can rely on the application of the jurors' common knowledge as to whether there was a breach. D: can rely on the judge, as an expert in the law, to advise the jury whether there was a breach.

C

When a mother visited a bowling alley to participate in the weekly bowling league competition held there, she brought her two-year-old son along and left him in a nursery provided by the bowling alley for the convenience of his customers. The children in the nursery were normally supervised by three attendants, but at this particular time, as the mother knew, there was only one attendant present to care for about 20 children of assorted ages. About 30 minutes later, while the attendant was looking the other way, the son suddenly started to cry. The attendant found him lying on his back, picked him up, and called his mother. It was later discovered that the son had suffered a skull fracture. If a claim is asserted against the bowling alley on the son's behalf, will the son prevail? A: Yes, because the bowling alley owed the child the highest degree of care. B: Yes, because a 2-year old is incapable of contributory negligence. C: No, because the bowling alley and its employees exercised reasonable care to assure the son's safety. D: No, because the mother assumed the risk by leaving her son in the nursery.

C

While approaching an intersection with the red light against him, a motorist suffered a heart attack that rendered him unconscious. The motorist's car struck a child, who was crossing the street with the green light in her favor. Under the state motor vehicle code, it is an offense to drive through a red traffic light. The child sued the motorist to recover for her injuries. At trial it was stipulated that: (1) immediately prior to suffering the heart attack, the motorist had been driving within the speed limit, had seen the red light, and had begun to slow his car; (2) the motorist had no history of heart disease and no warning of this attack; (3) while the motorist was unconscious, his car ran the red light. On cross motions for directed verdicts on the issue of liability at the conclusion of the proofs, the court should A: grant the child's motion, because the motorist ran a red light in violation of the motor vehicle code. B: grant the child's motion, because, in the circumstances, reasonable persons would infer that the motorist was negligent. C: grant the motorist's motion, because he had no history of heart disease or warning of the heart attack. D: deny both motions and submit the case to the jury, to determine whether, in the circumstances, the motorist's conduct was that of a reasonably prudent person.

C

Romeo considers himself irresistible. He is accustomed to flirting with the girls at will. He comes up to Ophelia, a new student, on her first day in the school and, by way of introduction, gives her a hug. She sues him for battery. Is he liable?

Obviously, Romeo is of the opinion that no woman in her right mind would object to his attentions. However, the question is not whether Romeo finds his conduct offensive. It is not even whether Romeo thinks that Ophelia will. As the introduction points out, the question Romeo must ponder before his dalliance with Ophelia is whether the reasonable person in Ophelia's circumstances would find it offensive. The answer to that question is almost certainly "yes." Most teenagers don't like being hugged by strangers, even attractive strangers. So, offensiveness is determined by an objective test—whether the contact would be offensive to the reasonable person in the victim's circumstances. But isn't it true, even if Romeo's hug is "offensive" under this definition, that Romeo must intend an offensive contact, not just cause one? And, if Romeo genuinely believed that the new girl in school would welcome his attention, how can he be said to have intended an offensive contact? Very likely Romeo will be held liable, even if he is too conceited to realize that this contact is offensive under the Restatement definition; the law will attribute to him an understanding of what the reasonable person finds offensive. Otherwise, he could avoid liability based on his testimony that he didn't think it would be offensive. Such a test would allow social boors to escape liability simply because they have poor judgment—or lie about what they understood—even though they inflict unwanted contacts on others. There is some authority suggesting that an actor commits battery by intentionally causing a contact that turns out to be harmful or offensive, even if the actor did not intend it to be either. This "single intent" theory (that is, that the only intent needed is the intent to make the contact) is approved in White v. University of Idaho, 797 P.2d 108 (Idaho 1990); but see White v. Munoz, 999 P.2d 814 (CO. 2014) (en banc) (rejecting the single intent theory). See generally K. Simons, A Restatement (Third) of Intentional Torts? 48 Ariz. L. Rev. 1061, 1070 (2006). The draft Third Restatement of Torts explores this "single intent" approach in great detail, and comes down in support of it. Restatement of the Law Third: Intentional Torts to Persons (Tentative Draft No. 1) §102 cmt. b and Reporter's Note. Your Humble Author is troubled by the "single intent" approach, because it would impose liability on an actor who made a contact that was not intended to cause either harm or offense. The Third Restatement draft acknowledges that this imposes "a modest degree of strict liability." Id. at §102, Reporters' Note §b. Romeo would be liable in this example under either a "single intent" or "dual intent" approach.

Which of the following is true as to intervening forces in a proximate cause analysis? A. Subsequent medical malpractice is usually not foreseeable B. Rescuers are generally not foreseeable intervening forces C. Acts of God may be foreseeable Intervening forces

C. Acts of God may be foreseeable Intervening forces

In most states, when a medical provider was negligent before or during a patient's pregnancy, damages for ______________ may be recovered by the _________ in a wrongful ______________ case. A. Pain and suffering; child; life B. Child-rearing expenses; parents; pregnancy C. Additional medical expenses; parents; birth

C. Additional medical expenses; parents; birth

Under the alternative causes approach to actual causation in negligence: A. It may be applied to one defendant with alternative theories of liability B. Each defendant initially must show he was not responsible before the burden shifts to the plaintiff C. The burden of proof shifts to the defendants after the plaintiff establishes that one of them caused the harm

C. The burden of proof shifts to the defendants after the plaintiff establishes that one of them caused the harm

A bailee's slight negligence may give rise to liability only in which type of bailment? A. A bailment for the sole benefit of the bailor B. A mutual benefit bailment C. A bailment for the sole benefit of the bailee

C. A bailment for the sole benefit of the bailee

Which of the following is correct regarding a dangerous condition for the attractive nuisance doctrine? A. A dangerous condition is a hazard to adults and children B. Only a natural condition may constitute a dangerous condition C. A body of water generally is not a dangerous condition

C. A body of water generally is not a dangerous condition

Several boys dug small caves at the base of a cliff cut into the wall of an unused portion of a gravel pit. Workers from another part of the gravel pit occasionally chased the boys out of their caves, but the boys would return after a few days. The boys never realized that the cliff under which they were digging could come down at any moment. A portion of the cliff collapsed, killing one of the boys. The boy's parents made a demand on the owner of the gravel pit, who consulted a lawyer regarding liability. Which of the following best describes the duty the gravel pit owner owed the boys? A. No duty because the boys were trespassers and the workers chased them off. B. No duty because a cliff is a natural condition and who knew it would fall? C. A duty with respect to concealed artificial conditions and discovered peril. D. A duty with respect to dangerous instrumentalities under the owner's control.

C. A duty with respect to concealed artificial conditions and discovered peril.

Which of the following statements of a lessor's duty as to leased premises is true? A. A lessor who voluntarily makes repairs and does so negligently is liable only if his negligent repairs make the condition worse B. A lessor's duty to repair unreasonably dangerous conditions relieves the tenant of liability for injuries to third persons from the dangerous condition C. A lessor owes no duty to a lessee as to unreasonably dangerous conditions of which the lessee is or should be aware D. A lessor's duty to members of the public to repair unreasonably dangerous conditions ends when the premises is transferred to the lessee

C. A lessor owes no duty to a lessee as to unreasonably dangerous conditions of which the lessee is or should be aware

In which of the following are both of the intervening forces almost always foreseeable in a proximate cause analysis? A. A subsequent accident and an intentional tort of a third person B. Efforts to protect person or property and acts of God C. A subsequent disease and negligence of rescuers D. Subsequent medical malpractice and criminal acts of third persons

C. A subsequent disease and negligence of rescuers

Which of the following is a permitted cause of action against a medical provider for negligence? A. A suit by a child for wrongful birth B. A suit by a child for wrongful life C. A suit by a parent for wrongful pregnancy

C. A suit by a parent for wrongful pregnancy

A lawyer prepared a will for an elderly client that bequeathed the client's estate to a charity. The lawyer had his daughter, a high-school student who was shadowing his secretary, sign the client's will along with the secretary. After the client subsequently died, heirs at law proved that the will was invalid because the lawyer's daughter was too young to act as witness. The charity sought legal advice regarding the lawyer's malpractice liability to it for losing the bequest. Which of the following statements is most accurate with respect to the lawyer's liability? A. The lawyer is not liable to the charity because the only duties the lawyer owed were to the elderly client, not the charity. B. The lawyer is not liable to the charity because the lawyer did his best to carry out the elderly client's wishes, and the elderly client is deceased. C. Depending on the law of the specific jurisdiction, the lawyer may be liable to the charity as the direct beneficiary of the lawyer's services. D. Depending on the law of the specific jurisdiction, the lawyer may be liable to the charity but only if the charity paid for the lawyer's services.

C. Depending on the law of the specific jurisdiction, the lawyer may be liable to the charity as the direct beneficiary of the lawyer's services.

If a condition on a landowner's property creates a danger to those off the premises: A. He does not owe a duty to anyone B. He owes a duty to invitees to take precautions C. He owes a duty to passersby to take due precautions

C. He owes a duty to passersby to take due precautions

Which of the following is an effect of res ipsa loquitur? A. It shifts the burden of proof to the defendant B. It creates a presumption of negligence on the part of the defendant C. It establishes a prima facie case for the plaintiff

C. It establishes a prima facie case for the plaintiff

A patient struggling with relationship problems visited a therapist. The patient described for the therapist problems that the patient had with his girlfriend. "I bought a gun and plan to use it on her," the patient calmly told the therapist. The therapist noted the statement in the patient's records. Two weeks later, the patient shot and killed his girlfriend. Police located the therapist's record and confirmed with the therapist that the patient had threatened the girlfriend. A lawyer for the girlfriend's estate learned of the threat. Which of the following statements best describes the therapist's liability to the girlfriend's estate? A. No liability because the therapist had no way to stop the patient from killing. B. No liability because a duty of confidentiality bound the therapist not to disclose. C. Liability to an identifiable victim of a particularized threat of imminent violence. D. Liability to anyone the patient harmed within two months of treatment.

C. Liability to an identifiable victim of a particularized threat of imminent violence.

A woman consulted a cosmetic surgeon to improve the appearance of her nose. The surgeon drew pictures of her anticipated new nose. The woman agreed to the surgery. An office manager then met with the woman to have her sign a consent form saying that nothing was guaranteed as to the final appearance of her nose. The surgeon then performed the surgery. The result was nothing like the pictures the surgeon had drawn. A second cosmetic surgeon told the woman that she got a bad result because the first surgeon had not done the surgery according to the custom. The woman consulted a lawyer who assigned an associate to draft a complaint against the first surgeon. What claims should the associate include? A. Breach of contract. B. Breach of contract and malpractice. C. Malpractice. D. Malpractice and battery.

C. Malpractice.

A patient consented to surgery on her right foot. With the patient under a general anesthetic, the surgeon mistakenly performed the surgery on the patient's left foot and then, realizing the mistake, also completed the right-foot surgery. The surgeries benefited both of the patient's feet, although the patient had consented to surgery on only one foot. The patient consulted a lawyer about claims against the surgeon. The lawyer negligently ignored the patient's matter until after the limitations period had expired for the patient's battery claim but not the medical-malpractice claim. Which of the following is the best evaluation of the patient's legal-malpractice claim against the lawyer? A. Substantial based on the lawyer's reprehensible conduct. B. Substantial based on the medical-malpractice claim. C. Nominal based on the battery claim. D. Non-existent because the lawyer did not hurt the patient.

C. Nominal based on the battery claim.

A man asked a store clerk if the farm-supply store the man was visiting had a public restroom. The store clerk showed the man to an employee area, saying that he was welcome to use the restroom there. The man cut himself badly on a broken piece of metal corner-beading when he tripped over a cat sleeping in the darkened employee area on the way to the restroom, and reached to catch himself. The man consulted a lawyer regarding whether he had a claim against the store for his injury. What additional evidence would most help the man in his claim, in a jurisdiction following traditional premises-liability rules? A. The store clerk was unaware that the cat was sleeping in the darkened backroom. B. The restroom was ordinarily reserved only for employees, not customers. C. The man had been inspecting riding mowers at the store for a possible purchase. D. The man had been waiting to find a restroom since he left home an hour earlier.

C. The man had been inspecting riding mowers at the store for a possible purchase.

A customer approached the entrance to a store during a light rain in cold weather. The customer slipped and fell on a thick layer of ice that had formed in front of the store's entrance as rain ran off the roof onto the concrete at the store entrance. The customer was seriously enough injured in the fall that she required hospitalization. The customer retained a lawyer to sue to get the store's insurer to pay for her medical bills. Which of the following best describes the standard of care the lawyer should allege in the complaint? A. The store owed the customer the duty to reasonably inspect the premises so as to warn the customer against weather conditions. B. The store owed the customer the duty to reasonably design and construct the premises so that rain did not freeze from run-off. C. The store owed the customer the duty to reasonably design, construct, inspect, and maintain the premises, and to reasonably warn and protect the customer. D. The store owed the customer the duty to keep all rain, ice, and snow off of the store entrance under any weather conditions.

C. The store owed the customer the duty to reasonably design, construct, inspect, and maintain the premises, and to reasonably warn and protect the customer.

A homebuyer contracted with surveyors for a survey of property that the homebuyer was considering purchasing. The surveyors had surveyed the property just the previous year. Instead of performing a complete survey, the surveyors provided the homebuyer with a copy of the previous survey. Unknown to the homebuyer or surveyors, a neighbor had recently built a driveway and carport on the property. The homebuyer discovered the neighbor's encroachment only after purchasing the property because the survey did not disclose it. The homebuyer made a demand on the surveyors for damages and expenses relating to the undisclosed encroachment. Which of the following facts would most support the surveyors' liability? A. The homebuyer paid the surveyors' charge at the time of the closing on the sale of property to the homebuyer. B. The neighbor did not have a survey performed before having the encroaching driveway and carport constructed by a builder. C. The surveyors usually did a visual inspection of the site to avoid these kinds of costly errors but had neglected in this instance to do so. D. The surveyors usually charged less for a survey based on a recent previous survey but had neglected in this instance to do so.

C. The surveyors usually did a visual inspection of the site to avoid these kinds of costly errors but had neglected in this instance to do so.

For a trespasser discovered on the landowner's property, what is the landowner's duty as to conditions on the land of which he is aware and which the trespasser is unlikely to discover? A. No duty as to conditions on the land B. To warn of or make safe dangerous natural or artificial conditions C. To warn of or make safe highly dangerous artificial conditions D. To warn of or make safe dangerous artificial conditions

C. To warn of or make safe highly dangerous artificial conditions

A 13-year-old girl was operating a high-speed motorboat. The boat was towing a 9-year-old boy in an inner tube tied to the rear of the motorboat by a rope. The rope became tangled around the boy's foot, causing him to suffer severe injuries. In a suit brought on the boy's behalf against the girl, the boy has introduced uncontroverted evidence that the girl drove carelessly in such a way as to entangle the boy in the rope. Is the boy likely to prevail? A: No, because the boy assumed the risk. B: No, because the girl was too young to be expected to appreciate and avoid the risk she exposed the boy to. C: Yes, because children of the girl's age should have the capacity to operate motorboats. D: Yes, because the girl will be held to an adult standard of care.

D

A fire that started in the defendant's warehouse spread to the plaintiff's adjacent warehouse. The defendant did not intentionally start the fire, and the plaintiff can produce no evidence as to how the fire started. However, the defendant had failed to install a sprinkler system, which was required by a criminal statute. The plaintiff can produce evidence that had the sprinkler system been installed, it could have extinguished the fire before it spread. In an action by the plaintiff against the defendant to recover for the fire damage, is it possible for the plaintiff to prevail? A: No, because the statute provides only for criminal penalties. B: No, because there is no evidence that the defendant negligently caused the fire to start. C: Yes, because a landowner is strictly liable for harm to others caused by the spread of fire from his premises under the doctrine of Rylands v. Fletcher. D: Yes, because the plaintiff was harmed as a result of the defendant's violation of a statute that was meant to protect against this type of occurrence.

D

A pedestrian started north across the street in a clearly marked north-south crosswalk with the green traffic light in her favor. The pedestrian was in a hurry, and so before reaching the north curb on the street, she cut to her left diagonally across the street to the east-west crosswalk and started across it. Just after reaching the east-west crosswalk, the traffic light turned green in her favor. She proceeded about five steps further across the street to the west in the crosswalk when she was struck by a car approaching from her right that she thought would stop, but did not. The car was driven by a driver, 81 years of age, who failed to stop his car after seeing that the traffic light was red against him. The pedestrian had a bone disease, resulting in very brittle bones, that is prevalent in only 0.02 percent of the population. As a result of the impact the pedestrian suffered a broken leg and the destruction of her family heirloom, a Picasso original painting that she was taking to her bank for safekeeping. The painting had been purchased by the pedestrian's grandmother for $750 but was valued at $500,000 at the time of the accident. The pedestrian has filed suit against the driver. The driver's attorney has alleged that the pedestrian violated a state statute requiring that pedestrians stay in crosswalks, and that if the pedestrian had not violated the statute she would have had to walk 25 feet more to reach the impact point and therefore would not have been at a place where she could have been hit by the driver. The pedestrian's attorney ascertains that there is a statute as alleged by the driver, that his measurements are correct, that there is a state statute requiring observance of traffic lights, and that the driver's license expired two years prior to the collision. The violation of the crosswalk statute by the pedestrian should not defeat her cause of action against the driver because A: the driver violated the traffic light statute at a later point in time than the pedestrian's violation. B: pedestrians are entitled to assume that automobile drivers will obey the law. C: the pedestrian was hit while in the crosswalk. D: the risks that the statute was designed to protect against probably did not include an earlier arrival at another point.

D

A pedestrian started north across the street in a clearly marked north-south crosswalk with the green traffic light in her favor. The pedestrian was in a hurry, and so before reaching the north curb on the street, she cut to her left diagonally across the street to the east-west crosswalk and started across it. Just after reaching the east-west crosswalk, the traffic light turned green in her favor. She proceeded about five steps further across the street to the west in the crosswalk when she was struck by a car approaching from her right that she thought would stop, but did not. The car was driven by a driver, 81 years of age, who failed to stop his car after seeing that the traffic light was red against him. The pedestrian had a bone disease, resulting in very brittle bones, that is prevalent in only 0.02 percent of the population. As a result of the impact, the pedestrian suffered a broken leg and the destruction of her family heirloom, a Picasso original painting that she was taking to her bank for safekeeping. The painting had been purchased by the pedestrian's grandmother for $750 but was valued at $500,000 at the time of the accident. The pedestrian has filed suit against the driver. The driver's attorney has alleged that the pedestrian violated a state statute requiring that pedestrians stay in crosswalks, and that if the pedestrian had not violated the statute, she would have had to walk 25 feet more to reach the impact point and therefore would not have been at a place where she could have been hit by the driver. The pedestrian's attorney ascertains that there is a statute as alleged by the driver, that his measurements are correct, that there is a state statute requiring observance of traffic lights, and that the driver's license expired two years prior to the collision. The pedestrian's violation of the crosswalk statute should not be considered by the jury because A: there is no dispute in the evidence about factual cause. B: as a matter of law the violation of the statute results in liability for all resulting harm. C: as a matter of law the driver's conduct was an independent intervening cause. D: as a matter of law the injury to the pedestrian was not the result of a risk the statute was designed to protect against.

D

The defendant parked her car in violation of a city ordinance that prohibits parking within 10 feet of a fire hydrant. The city ordinance was enacted solely to allow firefighters quick access to the hydrants in the case of an emergency. Because a man was driving negligently, his car sideswiped the defendant's parked car. The plaintiff, a passenger in the man's car, was injured in the collision. The plaintiff would not have been injured if the defendant was not parked by the fire hydrant. If the plaintiff asserts a claim against the defendant to recover damages for his injuries, basing his claim on the defendant's violation of the parking ordinance, will the plaintiff prevail? A: Yes, because the defendant was guilty of negligence per se. B: Yes, because the plaintiff would not have been injured had the defendant's car not been parked where it was. C: No, because the defendant's parked car was not an active or efficient cause of the plaintiff's injury. D: No, because the prevention of traffic accidents was not a purpose of the ordinance.

D

What type of injuries are subject to strict liability regarding wild animals?

Injuries caused directly by the animal, and also injuries caused by P's fearful reaction to the sight of an unrestrained wild animal.

Which of the following situations involves a common intervening force that courts almost always find foreseeable? A. A roofer negligently leaves a hammer on the plaintiff's roof, and a strong wind blows the hammer off the roof, where it strikes the plaintiff B. A defendant negligently blocks a sidewalk, forcing the plaintiff to walk in the roadway, where he is struck by a negligently driven car C. A parking lot attendant negligently leaves the keys to the plaintiff's car inside it with the doors unlocked, and a thief steals the car D. A defendant negligently causes a plaintiff to break her leg, and while walking on her crutches, the plaintiff loses her balance and injures herself

D. A defendant negligently causes a plaintiff to break her leg, and while walking on her crutches, the plaintiff loses her balance and injures herself

To which of the following plaintiffs may a landowner be liable? A. A neighbor who trips and falls over a rock that fell off the landowner's rock wall and landed on the neighbor's property B. A neighbor who slips and falls in his own driveway from berries that dropped from the landowner's tree overhanging the driveway C. A pedestrian walking on the public sidewalk in front of the landowner's property who slips and falls on a patch of ice after a freezing rain D. A pedestrian walking on the edge of the landowner's property adjacent to a sidewalk who falls into an unprotected excavation

D. A pedestrian walking on the edge of the landowner's property adjacent to a sidewalk who falls into an unprotected excavation

An affirmative duty to act arises for all of the following defendants except: A. A motorist who begins to assist an injured traveler B. A defendant who non-negligently placed another in peril C. An employer whose employee is injured in the workplace D. A physician in the presence of a bystander who needs medical attention

D. A physician in the presence of a bystander who needs medical attention

A farmer looked over used farm equipment on a sales lot. The farmer inspected a combine he thought he might purchase. The farmer knew from signs and prior conversations with sales agents that the sales lot's strict policy was that customers should stay out of equipment unless accompanied and authorized by a sales agent. The farmer nonetheless climbed into the combine to inspect the cab. Climbing out of the combine, the farmer fell and injured himself due to the combine's broken handrail. The farmer sued the sales lot for his injury. The sales lot's insurance company assigned the defense to a lawyer for answer. The lawyer's research disclosed that the jurisdiction would treat the farmer under the traditional classifications for premises liability. Which of the following best describes how the lawyer should treat the farmer's status at the moment of his injury, in the sales lot's answer? A. As a customer owed a duty of reasonable care. B. As an invitee owed a duty of reasonable care. C. As a licensee owed a duty to warn of known hidden dangers. D. As a trespasser owed no duty.

D. As a trespasser owed no duty.

Under the traditional implied assumption of risk defense, a plaintiff in the stands could not recover against a defendant for being struck by a foul ball during the defendant's baseball game. In most comparative negligence jurisdictions, a court in this case likely would rule that the plaintiff: A. Can recover reduced damages based on the shared risk of the activity B. Cannot recover damages if her unreasonable conduct contributed to her injury C. Can recover reduced damages based on a comparison of fault D. Cannot recover damages because the defendant has not breached his limited duty of care

D. Cannot recover damages because the defendant has not breached his limited duty of care

Which of the following is NOT a type of intervening force that is almost always foreseeable in a proximate cause analysis? A. Negligence of rescuers B. A subsequent disease C. Efforts to protect person or property D. Criminal acts of third persons

D. Criminal acts of third persons

An exclusive development constructed a shallow pond to advertise as a lake. The development constructed a dock in the pond to make the lake look more authentic. Late one night some visiting young boys snuck out to dive into the pond from the dock. The first boy to attempt the dive injured his neck in the shallow water. His parents sued the development, whose lawyer moved to dismiss the action. Which of the following best describes the court's most likely ruling on the motion? A. Grant the motion on the obviousness of the risk to boys on escapades like this one. B. Grant the motion on foreseeability, tender years, and attractive nuisance. C. Deny the motion on the exclusive nature of the development and affordability. D. Deny the motion on foreseeability, tender years, and attractive nuisance.

D. Deny the motion on foreseeability, tender years, and attractive nuisance.

What is one of the effects of res ipsa loquitur? A. It creates a presumption of negligence B. It requires the defendant to present evidence of due care in rebuttal C. The burden of proof switches to the defendant D. No directed verdict may be given for the defendant

D. No directed verdict may be given for the defendant

A host had several friends over to his home for an evening. The host offered beer and wine to his friends. Near the end of the evening, the host urged a departing guest, who was already slurring her words and acting tipsy, to have one last beer, which the guest did. The guest left in her car an hour later but only made it a few blocks before running a stop sign, striking and seriously injuring a pedestrian who was in the crosswalk. In a state with no dramshop act, what is the most likely evaluation of the pedestrian's negligence claim against the host? A. Liability due to the host negligently offering more beer to a drunken departing guest. B. No liability due to no negligence offering more beer to a drunken departing guest. C. No liability due to no cause in fact between the drinking and running the stop sign. D. No liability due to no proximate cause with respect to the guest voluntary imbibing.

D. No liability due to no proximate cause with respect to the guest voluntary imbibing.

A motorist ran her vehicle through a red light, hitting a pedestrian who was in the intersection's crosswalk. The pedestrian suffered fractures to both wrists when she put both hands forward to brace herself at the last instant before the collision. The pedestrian was taken to a hospital where an emergency physician overlooked the pedestrian's wrist fractures while treating other serious injuries. The physician's malpractice increased the pedestrian's wrist pain and disability resulting from the motor-vehicle accident. In the pedestrian's negligence claim against the motorist, the motorist's lawyer requested a jury instruction prohibiting the jury from including the increased wrist pain and disability from the physician's malpractice. Which of the following is the best evaluation of whether the jury instruction should be given? A. Yes, because malpractice is not foreseeable and is a superseding cause as to prior negligence. B. Yes, because malpractice is not foreseeable, although if it were, it would not be a superseding cause. C. Yes, because although malpractice is foreseeable, it is a superseding cause as to prior negligence. D. No, because malpractice is foreseeable and not a superseding cause as to prior negligence.

D. No, because malpractice is foreseeable and not a superseding cause as to prior negligence.

A board-certified neurologist misdiagnosed a patient's neurological condition. The misdiagnosis resulted in delayed treatment and a serious worsening of the patient's condition. The patient consulted a lawyer about a malpractice claim against the neurologist. The lawyer hired another board-certified neurologist expert from a distant city to testify to the standard of care and breach. At trial, defense counsel sought to prove the expert's distant residence to establish that the expert did not know the neurologist's standard of care in the locale where the misdiagnosis occurred. The lawyer objected to the questioning. Which of the following is the most accurate ruling on the objection? A. Objection overruled because distant experts are less credible. B. Objection overruled because the local standard is what matters. C. Objection sustained because the expert may know the local standard. D. Objection sustained on the basis of a national standard.

D. Objection sustained on the basis of a national standard.

A court likely will not permit a defendant to raise an assumption of risk defense when: A. Risk has been expressly assumed by a clause in a contract B. The plaintiff's injury was caused by a foul ball hit into the stands during a baseball game C. The plaintiff's injury was caused by the defendant's wanton or reckless conduct D. Risk has been assumed by a disclaimer from a common carrier

D. Risk has been assumed by a disclaimer from a common carrier

A physician sent a patient to a lab for special testing. The physician advised the patient that the testing could worsen the patient's condition but was advisable to diagnose the patient's condition. The patient's condition worsened as a result of the testing. The patient made a demand that the physician compensate the patient for the worsened condition. The physician consulted a lawyer regarding the physician's potential liability to the patient. Which of the following facts, none of which the patient knew at the time of consenting to the testing, would the lawyer most want to know in order to properly advise the physician? A. That the patient was suffering from another disease that required treatment. B. That the lab had an average incidence of success and failure with the testing. C. That the physician owned substantial assets to protect from liability in these cases. D. That the physician owned the lab to which he had sent the patient for the testing.

D. That the physician owned the lab to which he had sent the patient for the testing.

Which of the following is true under a comparative negligence system? A. When multiple defendants are involved, each defendant will be liable to the plaintiff only for the amount of damages that equates to his share of the fault B. The plaintiff's negligence will not be compared if the defendant's conduct was reckless C. Express assumption of risk is not recognized as a defense D. The last clear chance doctrine Is not applied

D. The last clear chance doctrine Is not applied

A novice climber negligently trapped himself on a precipitous ledge in severe weather through lack of skill, preparation, and proper equipment. An expert climber quickly reached the novice climber leading to the novice's heroic rescue. The expert climber was injured in the rescue process through no fault of her own. The expert climber missed several months of work in her job in the construction trades and had many thousands of dollars in lost income as a consequence. The expert climber consulted a lawyer regarding whether the expert climber had any way of recovering her lost income. Which of the following is the best evaluation? A. The expert climber has no recourse against anyone for the lost income. B. The expert climber has recourse against her construction trades employer. C. The novice climber is liable only if the novice climber has insurance. D. The novice climber is liable to the expert climber for the lost income.

D. The novice climber is liable to the expert climber for the lost income.

An earthquake destroyed a building. Investigation showed that when the building was built, an architect had failed to specify structural steel as was the custom. The building was built with ordinary steel. The building owner consulted a lawyer about suing the architect for the building's collapse. What further opinion does the lawyer need from an architect expert to sustain a malpractice action against the architect who failed to specify structural steel? A. What was the custom with respect to specifying structural steel in these circumstances. B. What were the architect's qualifications to design a building in these circumstances. C. Whether quakes were common enough that this building was sure to collapse. D. What would have happened in the quake if the architect had specified the structural steel.

D. What would have happened in the quake if the architect had specified the structural steel.

A hunter using lands with the owner's permission found an old tree stand. The hunter climbed into the tree stand to hunt. The tree stand broke, seriously injuring the hunter in a fall. The hunter sued the landowner for his injury. The landowner's insurer assigned the case to defense counsel, who undertook to research affirmative defenses. Which of the following should defense counsel research? A. Whether tree stands are reasonably safe for hunting. B. Whether hunters are ordinarily reasonably prudent. C. Whether there is a rescue-doctrine in the jurisdiction. D. Whether there is a recreational-use statute in the jurisdiction.

D. Whether there is a recreational-use statute in the jurisdiction.

Father tells mother & new husband that son was in a car accident & in a coma. Both suffer severe emotional distress but no physical symptoms. Father moves to dismiss new husband's claim. What result?

Deny motion, father is liable. D can be liable to a 3rd party for IIED if he intentionally / recklessly caused severe emotional distress to a member of P's immediate family, & P contemporaneously perceives D's conduct.

Which of the following describes only dependent intervening forces in a proximate cause analysis? A Efforts to protect person or property and acts of God. B Subsequent medical malpractice and criminal acts of third persons. C A subsequent disease and negligence of rescuers. D A subsequent accident and an intentional tort of a third person.

Dependent intervening forces are normal responses or reactions to the situation created by the defendant's negligent act. Dependent intervening forces are almost always foreseeable. A subsequent disease is a common dependent intervening force. The original tortfeasor is usually liable for diseases caused in part by the weakened condition in which the defendant has placed the plaintiff by negligently injuring her. Also, negligence of rescuers is a common dependent intervening force. Generally rescuers are viewed as foreseeable intervening forces, so the original tortfeasor usually is liable for their negligence. Efforts to protect person or property are common dependent intervening forces. A defendant is usually liable for negligent efforts on the part of persons to protect the life or property of themselves or third persons endangered by the defendant's negligence. Subsequent medical malpractice is also a common dependent intervening force. The defendant is usually liable for the aggravation of the plaintiff's condition caused by the malpractice of the treating physician. A subsequent accident may also be a dependent intervening force if the original injury was a substantial factor in causing the second accident. However, acts of God and intentional torts and criminal acts of third persons are independent intervening forces. Independent intervening forces operate on the situation created by the defendant's negligence, but they are independent actions rather than natural responses or reactions to the situation. (Note that the defendant may or may not be liable for independent intervening forces. It depends on whether they are foreseeable.)

For assumption of risk to be available as a defense, the plaintiff must have: A Ignored the risk and negligently assumed it. B Known of the risk and voluntarily assumed it. C Known of the risk and expressly assumed it. D Ignored the risk and impliedly assumed it.

For assumption of risk to be available as a defense, the plaintiff must have known of the risk and voluntarily assumed it. A plaintiff who ignored a risk or negligently assumed it would be subject to the defense of contributory negligence rather than assumption of risk. For assumption of risk, it is irrelevant that the plaintiff's choice is unreasonable. While the plaintiff must have known of the risk, it need not be expressly assumed; it may be impliedly assumed. Different rules may apply, but both types of assumption of risk may be raised as a defense.

What must be shown for a qualified privilege to apply for defamatory statements? A There must be a common interest between the publisher and the recipient B The statement must be reasonably relevant to the interest being protected C The statement must not have been made with ill will D The statement must be in response to a request by the recipient

For a qualified privilege to apply, the statement must be reasonably relevant to the interest being protected. The privilege does not encompass the publication of irrelevant defamatory matter unconnected with the public or private interest entitled to protection. However, there need not be a common interest between the publisher and the recipient; statements made in the interest of one party but not the other may fall within a qualified privilege. While a qualified privilege does not apply if the defendant acted with actual malice, that term refers to knowledge of falsity or reckless disregard of truth rather than ill will; hence, a defendant acting with ill will may still assert a qualified privilege. Finally, the statement need not be in response to a request by the recipient; if the publisher has a relationship with the recipient, volunteered statements may fall within the privilege.

Punitive damage awards against corporations and individuals cannot be limited

FALSE, The BMW v. Gore case decreased or eliminated punitive damages in many cases where punitive damage awards were much higher than the actual damages suffered. You should understand the factors on page 322 discussing when punitive damages will be assessed and how much is appropriate

Tort reform is designed to help people that are injured by increasing the amount of compensation they receive from the person that committed the tort.

FALSE, designed to help people that are injured by DECREASING

"Car dealers are crooked" is an example of actionable defamation.

False

A consumer purchased a grass trimmer from a hardware store. He took it out of the box and assembled it according to the instructions. He noticed that there were bolts and screws left over and some joints that could have accepted additional fasteners, but he just discarded the extra hardware. As he was using the trimmer, the housing came apart and a hard piece of plastic flew off. His neighbor, who was standing nearby, was struck in the eye by the piece of plastic and suffered permanent injuries. The neighbor sued the hardware store and the manufacturer of the trimmer in a strict liability action. Through discovery, it was determined that the instructions omitted a critical step in the assembly process that would have used the extra hardware, which is why the housing came apart, and that the manufacturer had received some complaints about the instructions previously. The hardware store had no knowledge of any complaints regarding any of the manufacturer's products. T/F? As to the hardware store, the neighbor will: not recover, because the hardware store had no opportunity to inspect the product and no reason to anticipate that the instructions were faulty.

False

In the AMC TV drama, Better Call Saul, a client tells Attorney Saul Goodman while they are speaking alone, "I thought all lawyers were idiots." Goodman replied, "Only half of us are idiots. The other half are crooks!" Since I am a lawyer, I can sue him for defamation.

False

Punitive damage awards against corporations and individuals can not be limited.

False

Tort reform is designed to help people that are injured by increasing the amount of compensation they receive from the person that committed the tort.

False

For a design defect in a products liability action, the plaintiff usually must show that: A A less dangerous modification or alternative for the product was economically feasible. B The product emerged from production different from other products and more dangerous than if it had been made the way it should have been. C The product was dangerous because it departed from its intended design. D The product does not comply with government safety standards.

For design defect cases, the plaintiff usually must show a reasonable alternative design, i.e., that a less dangerous modification or alternative for the product was economically feasible. Proof that the product emerged from production different from other products and more dangerous than if it had been made the way it should have been, or that the product was dangerous because of a departure from its intended design, establishes a manufacturing defect rather than a design defect. It is not necessary to show that the product does not comply with government safety standards to establish a design defect. A product may comply with the standards and still be found defective.

Which of the following constitutes a defamatory statement (assuming the statements are untrue)?

He was dismissed for embezzlement from his last job

Comparative negligence means

If any portion of the damages suffered by the plaintiff were caused by the plaintiff, the plaintiff's damages will be reduced by the amount of damages he caused

Romeo is sitting on a wall in front of the school. He sees Thibault wandering across the lawn, with his nose in a book, toward a trench recently excavated for some utility work. Cheering silently, he watches as Thibault ambles absentmindedly toward disaster. To his delight, Thibault walks right into the trench, suffering minor injuries and considerable humiliation. Thibault sues Romeo for battery. What result?

If desire can make a battery, Romeo has surely committed one, since he fervently hoped Thibault would fall in, and was delighted when he did. And we know that Romeo need not directly touch Thibault to batter him: Contact with the trench suffices to meet the contact requirement. And certainly Thibault found the contact both harmful and offensive. But Romeo is still not liable to Thibault. He has not done anything to cause the contact. To incur liability, he must act; he must inflict the contact, not simply hope for it. This contact results from the acts of others, not Romeo. No Offense Intended?

While practicing their target shooting at the firing range, a man and woman got into an argument that almost erupted into physical combat, except that they were restrained and separated by bystanders. Later, in the parking lot of the range, the man shot the woman in the shoulder. Bystanders who rushed to the scene immediately after hearing the man's shot found the woman on the pavement with a black metal flashlight in her hand. The woman's pistol was in her locker at the firing range. At the trial of the woman's civil action for battery against the man, the woman established that the man intentionally shot her. In defense, the man testified that the woman approached him, saying, "We'll settle this once and for all, right now," and raised an object toward the man. He testified that he feared that the woman was about to shoot him with a pistol, so he fired in self-defense. Assuming that the jury decides that the man is telling the truth, what must the jury also need to find for him to prevail? A No additional facts. B That a reasonable person in the same circumstances would have believed that the woman was about to shoot. C That the woman was at fault in raising a black object toward the man while threatening him. D That the woman was the original aggressor.

If the man prevails, it will be because the jury determined that he acted reasonably under the circumstances. One may act in self-defense not only where there is real danger but also where there is a reasonable appearance of danger. An honest but mistaken belief that the woman was about to shoot would justify the use of deadly force by the man if a reasonable person would have acted similarly under those circumstances. The test is an objective one—an honest belief alone is not sufficient. Thus, (A) is incorrect. (C) is incorrect because the woman's fault is not the determining factor—the reasonableness of the man's belief governs for self-defense. (D) is incorrect because it does not resolve whether the man had the right to use deadly force. Even if the man started the altercation at the range, he would have the right to use deadly force if the woman escalated the fight with deadly force.

In a products liability case based in strict liablity, a plaintiff may recover: A Only damages for economic losses B Only personal injury damages C Personal injury damages and property damages D Personal injury damages, property damages, and damages for economic losses

In a products liability case based on strict liability, a plaintiff may recover both personal injury damages and property damages for the supplying of a defective product. If the plaintiff's complaint is only that the product does not work as well as expected or requires repairs (i.e., no personal injury or property damages), most courts do not permit recovery of damages for economic losses under either a strict liability or a negligence theory; the plaintiff must bring an action for breach of warranty.

Bethune is driving a backhoe during the construction of a storm drain. She negligently backs up without looking or sounding her beeper and hits Maltby, a passing pedestrian, breaking her leg and knocking her over next to the excavation. DeWolfe, a construction worker down in the pit, negligently throws a large stone up out of the excavation without looking. The stone hits Maltby on the head, causing a severe concussion. She sues them both for her injuries. Are they jointly and severally liable to Maltby? Suppose that this case goes to trial against both Bethune and DeWolfe. The jury finds them both negligent, and that Maltby's total damages are $60,000. What judgment should the judge enter against each defendant?

In this example, Bethune and DeWolfe are joint tortfeasors as to part of Maltby's injuries, but not as to all of them. The negligence of both Bethune and DeWolfe caused the concussion. Maltby would not have been hit by the stone if Bethune had not knocked her into harm's way, or if DeWolfe had not negligently thrown the stone out of the pit. It is a case of successive negligent acts that together cause the harm. So long as DeWolfe's subsequent negligence is foreseeable, it does not insulate Bethune from liability. As to Maltby's concussion, Bethune and DeWolfe are joint tortfeasors. However, DeWolfe is not a joint tortfeasor with Bethune in causing Maltby's broken leg. DeWolfe's negligence had no part in causing the broken leg, which happened before DeWolfe entered the picture. Thus, Bethune would be solely liable for this injury, but both would be liable for Maltby's concussion damages. The problem here is that the jury's general verdict does not separate the leg damages from the concussion damages. As to Bethune, it doesn't matter, since she is liable for both. The judge should enter a judgment against her for $60,000. But DeWolfe should only be held liable for the concussion damages, and the jury's verdict doesn't indicate how much they are. The court should avoid this problem by using a special verdict form that asks the jury to specify the amount of damages the plaintiff has suffered from the leg injury and the amount attributable to the concussion. Then the judge can enter judgment against Bethune for the total damages and against DeWolfe for the separate amount attributed to the concussion.

Salon.com reported that a well-known reality tv star used the photo of a well-known DJ to find dates on his Ashley Madison account. The DJ claims that the use of his photo amounted to "extreme and outrageous" conduct and it caused, "prolonged pain and suffering, anxiety, depression, embarrassment, humiliation, loss of self-esteem and extreme and severe mental anguish and emotional distressincluding but not limited to frequent panic attacks, disruption of sleep and insomnia, mood changes, fits of anger, a loss of interest in pursuing both professional and personal endeavors, and a complete loss of self-confidence."If the DJ can prove his claims, the celebrity is liable for

Intentional infliction of emotional distress

A number of FBI agents wiretapped the telephone lines of suspects under the USA Patriot Act, which did not require warrants based on probable cause, so the warrants were later deemed to be invalid. What torts are these agents liable for when they listened to telephone conversations without valid warrants?

Invasion of privacy

Which tort did the hackers commit by releasing the names of the users of the site to the public?

Invasion of privacy

What does an anti-ademption statute do?

It gives the proceeds of real property to the devisee of the property whose interest was adeemed. At common law, the proceeds of adeemed real property were personal property. Under anti-ademption statutes, though, the proceeds go to the devisee of the adeemed real property.

You practice your twerking moves on the dance floor so you can impersonate your favorite pop singer, Miley Cyrus. Suddenly, you lose your balance and then you slip and fall, crashing into another patron on the dance floor and as a result, the patron has broken her wrist. You are liable for this tort

Negligence

Would the users of the website whose personal information was released by the hackers be able to win a lawsuit against the hackers for defamation?

No, as long as they were actual users of the website

To play a joke on his friend Ethel Rosenberg, Max disguises himself as an FBI agent, comes to Ethel's door, and tells her that her husband Julius has just been arrested for spying and is about to be executed. In fact, Max knows that Julius has been out fishing all day, and that Ethel has been scared sick worrying about him. She screams and faints, and is extremely anguished for months afterwards. When she recovers, she sues Max for intentional infliction of emotional distress. Max defends on the grounds that Ethel's distress has not led to any physical illness or injury (a factually correct statement). Will Max's defense succeed? _______

No. So long as the defendant's conduct has produced serious emotional distress, the fact that that distress is not manifested by physical symptoms (e.g., sleeplessness, nausea, or ulcers) is not fatal to the claim. (Obviously, the presence of physical symptoms makes the distress easier to prove, but physical harm is not actually required.) On the other hand, the distress must be severe; mere unhappiness, humiliation, or a couple of sleepless nights won't suffice. In general, the more objectively outrageous the conduct, the less proof of great distress is required. Max's conduct here is so completely outrageous that Ethel probably won't need very detailed proof of her distress.

The Minnow, a tiny ship, sets sail from a tropical port for a three-hour tour. Shortly thereafter a fearsome storm kicks up, the tiny ship is tossed, and the crew seeks refuge at a private dock belonging to Snively. If Snively sues for trespass, will he win? _______

No. The crew has a private necessity defense, because it seemed necessary to invade Snively's dock to avoid death or serious harm, and the invasion they committed was substantially less serious than the injury they faced. NOTE: Private necessity is analogous to self-defense, but there the plaintiff is the source of the threat. NOTE: The privilege of necessity means the landowner cannot take even what would otherwise be lawfulaction against the entrant. So if Snively turned the boat out to sea, and it was destroyed, Snively would be liable for conversion. NOTE: The privilege only lasts until the danger has passed. Any excess = trespass. RELATED ISSUE: Any loss caused to the landowner must be compensated; the private necessity privilege is limited.

Dorothy is out for a drive with her dog, Toto. A howling storm kicks up, and the visibility decreases to almost nothing. Dorothy is terrified, and says, "Toto, I don't think we're in Kansas anymore." She pulls off the road, into the driveway of the Wicked Witch of the West. She grabs Toto and runs into the garage. Wicked Witch sees Dorothy coming and reasonably believes she's trying to steal Witch's magic brooms, which she keeps in the garage. Witch runs out and beats her up. When Dorothy sues Wicked Witch for battery, will Wicked Witch have a valid "defense of property" privilege?

No. The focus here is on the how the defendant's mistake impacts his assertion of the "defense of property" privilege. The answer depends on what it is the defendant's mistaken about. Mistake negates the privilege if the mistake consists of a false (even if reasonable) belief that the intruder is not privileged to enter the land. That's the case here; Dorothy entered Witch's land out of necessity, and that's a privilege. Witch was mistaken about Dorothy's privilege to enter, and that mistake negates Witch's defense of property privilege. That means she'll be liable to Dorothy. RELATED ISSUE: But a reasonable mistake as to whether force is necessary will leave the privilege intact. For instance, let's say Dorothy really was trying to steal the brooms, but she didn't have any weapons and force wouldn't have been necessary to subdue her. If Witch mistakenly believed force was necessary and it wasn't, and that mistake was reasonable, she'll be able to rely on "defense of property" as a defense.

Pilot is flying her two-engine private jet from New York to Boston. Suddenly, one engine stops working, and Pilot is unable to restart it. Pilot knows that there is a good, but not 100%, chance that she will be able to continue on just the other engine until Boston. However, she decides that it would be more prudent to make an emergency landing sooner. There are no commercial airfields around, so she lands in a meadow owned by Farmer. There is no measurable economic harm done to the meadow. Farmer sues Pilot for trespass. May Farmer recover anything?

No. Under the doctrine of "private necessity," a person has a privilege to enter another's property if this is necessary to protect herself (or another) from serious harm. This privilege constitutes a complete defense to Farmer's trespass action. See Rest. 2d, §197, Illustr. 3.

Sindell sues nine DES makers in California. At trial, she introduces some proof that the drug was that of Acme Drug Company. In the alternative, she relies on the market share theory. If the jury concludes that Acme made the drug her mother took, what should it do?

Obviously, the jury should find Acme liable for her full damages (assuming, of course, that they find Acme was negligent) and dismiss the claims against the other manufacturers. The market share theory provides a back-up alternative where the plaintiff cannot prove which maker caused the harm. If the jury finds that one defendant caused it, that defendant should pay, and the others should not.

Regan and Goneril, two teenagers, decide to wile away the afternoon standing on a bridge over the interstate, watching the traffic. Regan takes a mirror from her pocket and starts to shine it in the eyes of oncoming drivers. Cordelia, driving under the bridge, is temporarily blinded, swerves out of control, and hits the bridge. Is Regan liable for battery?

On first glance, this seems to be a battery because the plaintiff crashed into the bridge abutment. That's a contact if there ever was one. However, Regan did not have the necessary intent to cause that contact. She did not either desire or know to a substantial certainty that Cordelia would crash into the wall. Presumably, she merely meant to annoy drivers, not to cause a crash, and it is far from substantially certain—though clearly possible—that this brief distraction would cause Cordelia to crash. Regan did, on the other hand, intend to flash the mirror in Cordelia's eyes: She desired to cause that result, even if she was not substantially certain that her aim would be good enough to hit such a small moving target. But, of course, for battery, there has to be a contact, and that means the court would have to conclude that the light rays constitute "contact." One reader sent me the following e-mail, arguing that the light here satisfies the contact requirement. The physics of light is divided into particle theory and wave theory. Although it is certain that light behaves as a wave, physicists more often describe it using particle theory. A particle of light is a photon imparting energy to a surface that it strikes. Under this theory, light is analogous to the smoke of example [12f]. Lasers are highly focused beams of photons having extremely high energy. The difference in a laser and reflected sunlight is a matter of degree, not nature. This may be good science, but I'm not sure that it would persuade a court that calling light a contact would be good law. Accepting the argument opens the door to slippery-slope arguments in the horn honking and other cases. If Regan had used a high-powered laser that burns through steel, the court would doubtless find a contact. On these facts, however, the court might conclude that there was no contact, and therefore no intentional tort. But see Adams v. Commonwealth, 534 S.E.2d 347, 351 (Va. Ct. App. 2000) (contact through laser held a battery where it "result[ed] in some manifestation of a physical consequence or corporeal hurt"). If Regan's act was not a battery here, it surely was recklessness or gross negligence, which, of course, gives rise to tort liability as well.

What type of damages are available for intentional misrepresentation?

Only actual economic loss. Personal injury damages aren't available.

What's the Restatement rule for duty to trespassers?

Owner owes a duty of reasonable care to all, including trespassers, unless they're a flagrant trespasser.

Corbett, a four-foot-nine-inch gymnast, approaches Press, a 250-pound shot-putter who has just beaten out the American favorite in the shot put competition. Shaking her fist at her, she growls, "Press, wipe that smile off your face or I'll wipe it off for you!" Press sues Corbett for assault. Is she liable?

Perhaps your first response here is that Press would have to be dreaming to be at all intimidated by Corbett's threat. Given the disparity in their size, the reasonable shot-putter would not have feared a threat from Corbett, so arguably no assault occurred. However, while Press doubtless was not frightened by Corbett's threat, her conduct does cause Press to apprehend an unwelcome contact from her—surely Corbett has the present ability to attack Press, even if she was unlikely to cause her any substantial harm. The action for assault protects victims from the anticipation of unwelcome contacts, even where they are confident that they can adequately defend themselves. Put another way, assault turns on the acts and intent of the defendant, not on how good the victim is at self-defense. The strong have a right to physical autonomy as well as the weak, and should not be placed in the position of having to defend themselves from such invasions. Corbett is liable for assault.

In an NHL playoff hockey game, Wayne Greatsky gets around the player defending him, Mario Lemeow, and scores. Lemeow, enraged at being scored upon, skates up behind Greatsky and hacks at him with his stick. Greatsky falls, strikes his head on the ice, and suffers a career-ending injury. Greatsky sues Leme-ouw for battery. Lemeow raises the affirmative defense of implied consent. Will this defense succeed? _______

Probably not. Where people participate in an organized sport, each will be held to have impliedly consented to those harmful or offensive bodily contacts that are an ordinary part of the give-and-take of the game , even if the contact violates the rules of the game. But a player will generally not be held to have impliedly consented to actions by fellow competitors that are "unrelated to the normal method of playing the game," and that are done without any competitive purpose. Here, if Lemeow had, say, tripped Greatsky in an attempt to stop him from scoring, the court would probably hold that Greatsky impliedly consented to that conduct, even though it constituted a foul, because the contact was part of the ordinary give-and-take of professional hockey, and was done for competitive purposes. But when Lemeow struck Greatsky out of frustration, and not until the play was over, a court would likely hold that Lemeow's action was "unrelated to the normal method of playing the game," was not done with a competitive purpose, and was therefore not covered by the consent that Greatsky will be found to have impliedly given by entering the league. Cf. Hackbart v. Cincinnati Bengals, Inc. (where one football player strikes another out of anger after the play was over, this was not a contact to which the person struck impliedly consented).

Romeo likes to drive his souped-up Trans Am around the high school parking lot, racing the motor, accelerating rapidly, and stopping on a dime. He arrives at school one winter morning, speeds across the parking lot, and screeches to a halt in a parking space, hoping to impress the ladies with his hotshot driving. Unfortunately, the parking lot is icy; the rear end of the car skids out of control, jumps the curb sideways, and knocks Thibault to the ground. Has Romeo battered him?

Romeo has done a dumb thing, a clearly negligent thing, but he has not committed a battery. A battery requires an intent to cause a harmful or offensive touching. While Romeo certainly did cause a harmful contact, he didn't intend to under the Restatement definition. He did act intentionally in the sense that he deliberately drove his car across the lot. However, while this act was voluntary, he did not act with the purpose of hitting Thibault or with knowledge that he was substantially certain to do so; he was headed in another direction entirely. Nor, the facts suggest, was he trying to frighten Thibault or another student, which might support an argument for transferred intent. He was just showing off. Thus, his act was not intentional in the limited sense in which courts use that term for defining intentional torts. In analyzing battery cases, always distinguish the intent to act from the intent to cause a harmful or offensive contact. Battery requires more than a deliberate act. It requires a deliberate act done for the purpose of causing a harmful or offensive contact, or which the actor knows to a substantial certainty will cause such a contact. If only a deliberate act were required, battery would encompass many cases where the actor intended no harmful or offensive contact. For example, a driver would commit a battery if she looked away from the road and got in an accident, even though she did not intend to hit the plaintiff. Or, a joker would be liable for battery for throwing a snowball at a tree, if a pedestrian unexpectedly stepped into the snowball's path. In both of these cases, the actor did a voluntary act. But these acts—like Romeo's in the example—were not done with the state of mind necessary to commit an intentional tort: either purpose or substantial certainty that a harmful or offensive contact would result. They may be negligent acts, if the actor failed to exercise due care, but they are not intentional torts.

a defendant (the person who needs rescuing) is _______ if he negligently puts himself or a third party in peril and the plaintiff (the rescuer) is injured in attempting a rescue

liable

Best practices when invoking the shopkeeper's privilege include

Stopping shoplifters discreetly, not using physical force unless it is in response to the shoplifter's physical force, BOTH

A ______________________ is one where the facts present an uninterrupted chain of events from the time of the defendant's negligent act to the time of plaintiff's injury.

direct cause case

Asking a buyer to break its existing contracts with another supplier is contract interference.

TRUE

Comparative negligence reduces the plaintiff's recovery

TRUE

False imprisonment occurs when a person is detained for any period of time against his or her will

TRUE

If a plaintiff is found to be 15% responsible for her injuries in a comparative negligence jurisdiction, the defendant will have to pay 85% of the damages to the plaintiff.

TRUE

In most states, employers are not liable for defamation if the information that they disclose about a former employee is provided in good faith.

TRUE

Intentional infliction of emotional distress imposes liability for conduct that exceeds all bounds of decency

TRUE

There often is more than one cause for an injury. The "but for" test for actual cause applies to: A Joint causes. B Alternative causes. C Superseding causes. D Concurrent causes.

The "but for" test for actual cause applies to concurrent causes. An act or omission to act is the cause in fact of an injury when the injury would not have occurred but for the act. This test applies in concurrent cause cases, where several acts combine to cause the injury, but none of the acts standing alone would have been sufficient. But for any of the acts, the injury would not have occurred. The "substantial factor" test is used for joint causes, where several causes commingle and bring about an injury, but any one alone would have been sufficient to cause the injury. In that case, it is sufficient if defendant's conduct was a substantial factor in causing the injury. An alternative causes situation arises when two or more persons have been negligent, but uncertainty exists as to which one caused the plaintiff's injury. Under this approach, the plaintiff must prove that harm has been caused to him by one of them (with uncertainty as to which one). The burden of proof then shifts to the defendants, and each must show that his negligence is not the actual cause. Superseding causes arise in the context of proximate cause rather than actual cause. In addition to being an actual cause, the defendant's conduct must also be a proximate cause of the injury. Causes that arise after the defendant's conduct that contribute to the injury may be so unforeseeable as to be superseding causes, which cut off the defendant's liability for his original negligent act.

A bicyclist was riding his bicycle in the street when a negligently driven car struck the bike, knocking the bicyclist off the bike and breaking his right ankle. The driver of the car immediately stopped and went to his assistance. She got him to his feet and was slowly moving him toward the curb when a negligently driven taxicab struck him in the left leg. The bicyclist required surgery on both his right ankle and his left leg. If the bicyclist sues the driver and the cabbie, which of the following best states his right to recover? A He can recover from either the driver or the cabbie for all of his injuries because the driver and the cabbie are jointly and severally liable. B He can recover from the driver only for the injury to his right ankle and recover from the cabbie only for the injury to his left leg. C He can recover from either the driver or the cabbie for the injury to his left leg and recover from the driver only for the injury to his right ankle. D He cannot recover against the driver for the injury to his left leg unless the jury determines that the driver acted negligently when she came to his aid.

The bicyclist can recover from either party for the left leg injury but only from the driver for the right ankle injury. When two or more tortious acts combine to proximately cause an indivisible injury to a plaintiff, each tortfeasor is jointly and severally liable to the plaintiff for the entire damage incurred. Joint and several liability applies even though each tortfeasor acted entirely independently. However, if the actions are independent, plaintiff's injury is divisible, and it is possible to identify the portion of injuries caused by each defendant, then each will be liable only for the identifiable portion. Here, the cabbie would not be liable for the injury to the right ankle, because the cabbie did not cause the injury. (A) is therefore incorrect. With regard to the left leg, the cabbie was not the only cause of that injury. The original tortfeasor is liable for harm caused by the negligence of third persons when such negligence was a foreseeable risk created by the original tortfeasor's conduct. Here, as a result of the driver's original negligence, the bicyclist was in a position of danger while he was still in the street. The negligence of the cabbie in striking the bicyclist was a foreseeable risk while the bicyclist was in the street; it is therefore a foreseeable intervening force that will not cut off the driver's liability. Hence, both the driver and the cabbie will be jointly and severally liable for that injury. (B) is therefore incorrect. (D) is incorrect because the driver remains responsible for the foreseeable consequences of her original negligence in striking the bicyclist, regardless of whether she acted with due care when she came to his aid.

Is proof of damages required for trespass to chattels?

Yes. P must prove either actual damages or loss of use.

A chicken has a tendency to peck at Owner's hands to protect eggs. Chicken gets out one day & pecks mailman's hand. Can mailman sue, and what standard will apply?

Yes. Strict liability. This is a domestic animal with a known dangerous propensity.

A housecleaning agency was given a key to a customer's house so that the agency could have its employees clean while the homeowner was away. After a maid sent by the agency had finished and left the homeowner's house, she went back because she had forgotten her cigarettes. She neglected to lock the door when she left the second time because she was already late for the next job. When the homeowner returned after a few days away, she discovered that her house had been ransacked and several items of jewelry stolen. The front door was open, and there were no signs of forced entry. If the homeowner brings an action against the agency that employed the maid, what is the likely result? A She will not prevail, because she is limited to claims for breach of contract based on her agreement with the agency. B She will not prevail, because the act of the burglar was an independent supervening cause of the homeowner's loss. C She will prevail, because the maid's failure to lock the door created the risk that someone might enter and take the homeowner's valuables. D She will prevail, because when the maid returned after having completed her work, she was technically a trespasser, making the agency vicariously liable for any damage she caused to the premises.

The homeowner will prevail because the maid's negligence increased the risk of criminal conduct by a third party. Criminal acts and intentional torts of third persons are foreseeable independent intervening forces if the defendant's negligence created a foreseeable risk that they would occur. Here, the maid's failure to lock the door was negligent because it created a risk of burglary; hence, the burglary does not cut off the agency's liability for the maid's negligence. As the maid's employer, the agency is vicariously liable under respondeat superior. (A) is wrong because there is nothing in the facts to indicate that the homeowner waived her right to bring tort claims against the agency; having a contractual relationship with a party does not automatically preclude bringing a tort action against the party. (B) is wrong because the burglary was not a superseding cause of the loss; it was within the increased risk caused by the maid's negligence. (D) is wrong because she reentered to retrieve a personal item that she had brought with her when she went to the job; her return just to get the item was within the scope of her employment and would not make her a trespasser.

A new homeowner had two dogs that frequently barked at birds and squirrels in the yard, especially during the day while the homeowner was at work. A neighbor who worked nights was aggravated by the barking, which disturbed his sleep, and decided to let the homeowner know how he felt. One evening, upon learning that the homeowner was entertaining her boss and several clients, the neighbor came to her front door with a boombox and started playing a recording of the dogs barking, putting it at full volume. When the homeowner came to the door, he began yelling at her and berating her in front of her guests for having no consideration for her neighbors, while continuing to play the recording. The homeowner was very upset, especially because her guests decided that they had better leave, and she ended up losing a bonus that her boss was going to give her at the end of the evening. If the homeowner asserts a claim based on intentional infliction of emotional distress against the neighbor, what will be the probable result? A The homeowner will prevail because the neighbor's conduct was extreme and outrageous. B The homeowner will prevail because she suffered pecuniary harm from the neighbor's conduct. C The neighbor will prevail because the homeowner suffered no physical harm. D The neighbor will prevail if the barking from the homeowner's dogs is judged to constitute a nuisance.

The homeowner will probably prevail on a claim for intentional infliction of emotional distress because the neighbor's conduct was sufficiently extreme and outrageous and the other elements of the tort are present. Intentional infliction of emotional distress requires: (i) an act by defendant amounting to extreme and outrageous conduct; (ii) intent to cause severe emotional distress or recklessness as to the effect of defendant's conduct; (iii) causation; and (iv) damages. "Outrageous conduct" is extreme conduct that transcends all bounds of decency. The neighbor's use of the recording and his insults against the homeowner for the benefit of her guests would probably qualify as extreme and outrageous conduct, particularly because there is no evidence that he had previously tried to resolve the problem with the homeowner in a more civilized manner. The neighbor had the requisite intent (either he intended to cause emotional distress or he was reckless as to its effect), there was causation, and the homeowner suffered damages (i.e., she was severely distressed) as a result of the neighbor's actions. (B) is wrong because pecuniary harm is not required for purposes of this tort—all that is required is severe emotional distress. (C) is wrong because, in contrast to negligent infliction of distress, intentional infliction of distress does not require proof of physical harm to recover. (D) is wrong because the fact that the barking constituted a nuisance would not be a defense to conduct amounting to intentional infliction of distress; abatement of a private nuisance by self-help must be preceded by notice to the other party and must be conducted in a reasonable manner.

For purposes of private nuisance, the interference with the plaintiff's use of the land is unreasonable only if: A The interference is offensive or annoying to an average person in the community. B The remedy of damages is unavailable or inadequate. C The nuisance is a "nuisance per se." D The severity of the plaintiff's inflicted injury outweighs the utility of the defendant's conduct.

The interference with the plaintiff's use of the land will be considered unreasonable under nuisance law when the severity of the plaintiff's inflicted injury outweighs the utility of the defendant's conduct. In balancing these respective interests, courts take into account that every person is entitled to use his own land in a reasonable way, considering the neighborhood, land values, and existence of any alternative courses of conduct open to the defendant. Whether the interference is offensive or annoying to an average person in the community is the test for whether the interference is substantial, which is a separate requirement for establishing a nuisance. Whether the remedy of damages is unavailable or inadequate determines only whether the plaintiff may be able to obtain an injunction remedy. A nuisance is sometimes called a "nuisance per se" when it is based on strict liability (e.g., a nuisance arising from an abnormally dangerous activity).

Which of the following invasion of privacy branches require the plaintiff to show "publicity"? A Intrusion on plaintiff's affairs or seclusion and public disclosure of private facts about plaintiff. B Intrusion on plaintiff's affairs or seclusion and publication of facts placing plaintiff in a false light. C Publication of facts placing plaintiff in a false light and public disclosure of private facts about plaintiff. D Intrusion on plaintiff's affairs or seclusion, publication of facts placing plaintiff in a false light, and public disclosure of private facts about plaintiff.

The invasion of privacy branches based on publication of facts placing the plaintiff in a false light and public disclosure of private facts about the plaintiff require "publicity" concerning the false light or private facts—i.e., widespread dissemination of the facts. Mere publication to a third person is not sufficient for liability. In contrast, invasion of privacy based on intrusion on the plaintiff's affairs or seclusion requires neither publication nor publicity - just the act of intruding.

Bell is driving home late at night when her car dies. She pulls into the breakdown lane, but fails to put on her hazard lights or put out flares. Marconi negligently fails to see her, drifts into the breakdown lane and hits Bell's car, injuring Thatcher, a passenger in Bell's car. Thatcher sues Marconi for damages. If the jury concludes that Bell and Marconi were both negligent, and assesses Thatcher's damages at $60,000, what verdict should it render against Marconi, assuming it has been properly instructed on the principles of joint and several liability (and understood them!)?

The jury should render a verdict against Marconi for $60,000. His negligent driving was a "but for" cause of the harm. Consequently, he is a tortfeasor. Under joint and several liability he is liable to Thatcher for her full damages. It is true that Bell is a tortfeasor too, since her negligence was another cause of the accident. Under principles of joint and several liability, Thatcher was free to sue either of the tortfeasors and recover her full damages. She could, of course, have sued them both. But she didn't have to. She could choose . . . and it was not uncommon for a plaintiff like Thatcher to choose not to sue the driver of the car she was in.

A columnist for a major metropolitan newspaper had a very antagonistic relationship with the city's mayor. When a restaurant owned by the columnist's family was shut down by city health inspectors, the columnist responded with a column publicizing the shutdown and asserting that it was in retaliation for his prior columns in which he had criticized the mayor. In fact, the mayor had nothing to do with the action by the city health inspectors. While the columnist had no evidence of the mayor's involvement, he believed that there was a connection because "that's how the city works." Can the mayor recover against the columnist for defamation? A No, because the columnist did not act with actual malice. B No, because the columnist had a qualified privilege to explain why he believed his family's business was shut down. C Yes, because the columnist's hostility toward the mayor establishes malice so as to overcome any qualified privilege the columnist had. D Yes, because the columnist should have investigated the accuracy of his claims before publishing the column.

The mayor cannot recover against the columnist because he did not act with actual malice. A public official, such as a mayor, may not recover for defamatory words relating to his official conduct unless there is clear and convincing proof that the statement was made with actual malice, which is defined as knowledge that the statement was false or reckless disregard as to truth or falsity. Reckless conduct is not measured by whether a reasonable person would have investigated before publishing; rather, there must be a showing that the defendant in fact (subjectively) entertained serious doubts as to the truthfulness of his publication. Here, while the columnist had no evidence of the mayor's involvement with the action of the health inspectors, he believed that there was a connection based on his belief as to how the city operates. Hence, he has not acted with actual malice and is not liable to the mayor for defamation. (B) is incorrect because the columnist's qualified privilege applies only to statements made to defend his own actions, property, or reputation. Even if it were to apply to his explanation of why his family's restaurant was shut down, his statements in the column were beyond the scope of the privilege, which does not extend to making a statement to a mass audience whose reading of the statement would not reasonably further his interest in defending himself. Here, the publication in his newspaper column of his explanation as to why the restaurant was shut down was beyond the scope of any privilege he may have had. (C) is incorrect because malice that will result in the loss of a qualified privilege is defined by most courts as knowledge of falsity or reckless disregard as to truth or falsity, rather than hostility or ill-will. As long as the defendant is using a proper occasion for a qualified privilege in a proper way, he will not lose this privilege simply because he bears ill-will toward the plaintiff. (D) is incorrect because the fact that the columnist should have investigated the accuracy of his assertions and did not only establishes negligence on his part. As discussed above, the mayor, as a public official, must show at least reckless disregard as to truth or falsity to recover in a defamation action.

A motorcyclist was injured in a collision and suffered $100,000 worth of injuries, including $20,000 in hospital and physician's bills. The motorcyclist's medical insurance company paid her $20,000 to cover hospital and medical expenses. Later, she filed suit against the driver of the car that struck her motorcycle. When the case came to trial, the jury agreed with the motorcyclist's contention that her injuries were worth $100,000. The jury also determined that the motorcyclist was 30% negligent and that the driver was 70% negligent. How much should the motorcyclist recover from the driver? A $100,000. B $70,000. C $56,000. D $50,000.

The motorcyclist should recover $70,000 from the driver. Under a comparative negligence system, a contributorily negligent plaintiff is allowed to recover a percentage of her damages. The plaintiff's damages are reduced according to her proportionate share of the fault. Thus, the motorcyclist can recover 70% of her total of $100,000 in damages because she was 30% at fault, leaving her with a recovery of $70,000. As a general rule, damages are not reduced or mitigated by reason of benefits received by the plaintiff from other sources, such as health insurance. Therefore, the $20,000 paid by the motorcyclist's insurance company will not reduce the $70,000 in damages to which she is entitled. (A) is incorrect because it fails to reflect the reduction in damages required under comparative negligence. Because the motorcyclist was 30% negligent, she cannot recover the entire $100,000. (C) is incorrect because it is derived from an initial reduction of damages by the amount of the insurance payments ($100,000 minus $20,000, leaving $80,000). This $80,000 figure is then reduced by the 30% negligence of the motorcyclist, leaving an amount of $56,000. However, as noted above, the insurance payments are not allowed to reduce damages. Thus, the 30% reduction is made from the figure of $100,000, not from $80,000. Similarly, (D) is incorrect because it is derived from a reduction of the $70,000 proportionate recovery by the $20,000 insurance payment.

Which of the following statements is NOT true under the rule that the tortfeasor takes the victim as he finds him? A The rule applies to the victim's existing physical condition but not his mental condition. B The rule applies in both direct cause cases and indirect cause cases. C The unforeseeable severity of the plaintiff's harm does not relieve the defendant of liability. D The rule is also known as the "eggshell-skull plaintiff" rule.

The rule that the tortfeasor takes the victim as he finds him applies to both the victim's existing physical condition and his mental condition. In both direct cause cases and indirect cause cases, the fact that the extent or severity of the harm was not foreseeable does not relieve defendant of liability; in other words, the unforeseeable severity of the plaintiff's harm, or its extent, is irrelevant under this rule. This rule is also known as the "eggshell-skull plaintiff" rule.

A landowner had a swimming pool and a dressing cabana constructed in her spacious backyard. The pool was entirely within the confines of the landowner's property. However, one corner of the cabana extended a few inches onto a far corner of her neighbor's land. At the time of the construction, neither the neighbor nor the landowner was aware that the cabana extended onto the neighbor's property. Does the neighbor have a cause of action for trespass? A Yes, because the cabana extends onto the neighbor's land. B Yes, because the presence of the cabana on the neighbor's land has caused damage to his property. C No, because the landowner did not actually enter the neighbor's property. D No, because the landowner did not intend to have the cabana encroach on the neighbor's property.

The neighbor will prevail because the cabana extends onto the neighbor's land. The tort of trespass to land requires: (i) an act of physical invasion of the plaintiff's real property by the defendant, (ii) intent by the defendant to bring about a physical invasion of the property, and (iii) causation. The intent required is the intent to enter on a particular piece of land, rather than intent to trespass. Also, it is not necessary that the defendant personally enter the land. It is sufficient if the defendant's act or something set in motion thereby causes a physical invasion of the property. By having the cabana constructed, the landowner acted so as to bring about the physical invasion of the neighbor's land. (C) is incorrect because it makes no difference that the landowner herself did not enter the property that was being violated. (D) is incorrect because the landowner's intent to have the cabana built on its current site suffices for purposes of trespass liability. As noted above, the defendant need not have intended to commit a trespass. (B) is incorrect because actual injury to the violated property is not a prerequisite to sustain this cause of action. Damage is presumed.

Man prepares a deed to transfer land to his niece and records it. He doesn't tell the niece and doesn't deliver it, though. They then have a falling out. When the man died, niece found the deed and sold the land to a good faith buyer. Man devised the same land to his nephew. A notice recording statute applies. Who owns the land?

The nephew. The presumption of delivery by recording is rebuttable. The falling out rebuts the presumption of delivery. So Man owned the land at death. Since the niece never owned the land, her transfer to a 3rd party was invalid, and the recording statute didn't protect the buyer.

A man working at a clothing store discovered that his girlfriend and coworker had been taking money from the cash register. Not wanting to be a party to the situation, he ended the relationship and found another job. Not long after this, the man's new boss, who knew why the man had quit, came into the clothing store. He asked the girlfriend if she missed her boyfriend working with her at the store. She replied, "Yes, but when we found that he was stealing from the cash register, we had no choice but to let him go." If the man sues his former girlfriend for defamation, the fact that the new boss knew the truth of why the man had left his job at the store will have what result? A It will act as a complete defense to an action for defamation. B It will establish that the man has not suffered any actual injury. C It may diminish the damages that the man would be entitled to recover. D It proves that the girlfriend had no reasonable ground for believing that the man was fired for dishonesty.

The new boss's knowledge of the true circumstances behind the man's departure from the store may diminish the man's recovery. The girlfriend is liable for defamation because she made a defamatory statement about the man to a third person. As long as it is understood in its defamatory sense, an accusation need not be believed to be actionable. Because the statement that he was stealing at his job constituted slander per se, damages are presumed. Hence, (A) is wrong. (B) is wrong because actual injury encompasses not only damage to reputation but also humiliation and mental distress, for which the man could recover even though the new boss did not believe the girlfriend's statement. (D) is wrong because the fact that the new boss did not believe the statement does not prove lack of basis for the girlfriend to have made it.

What rights does one have in surface water?

The owner of the property can impound water and make reasonable use of it. The owner cannot prevent water from entering her property OR alter the flow of water off her property, though.

The tort of contract interference occurs when

There is a valid contract, the third party knew of contract, the plaintiff is injured, the third party induces one of the parties to breach the original contract, ALL of the above

A patient troubled by an irritating skin rash consulted a dermatologist for treatment. The dermatologist diagnosed the rash as a genetic condition that had no cure and would ultimately spread and lead to disfigurement. The patient was shocked and distressed by the diagnosis. On the advice of her family, a week later the patient consulted another doctor. That doctor immediately diagnosed the skin rash as a common bacterial infection and prescribed an ointment that cleared up the condition in a few days. Because the doctor was a friend of the family, the patient was not charged for that visit. Can the patient recover from the dermatologist for the emotional distress caused by his erroneous diagnosis? A Yes, but only if the patient's distress caused her some physical injury. B Yes, because the misdiagnosis by the dermatologist caused the patient actual harm. C No, because the patient did not have to pay for the second doctor visit. D No, because the dermatologist's conduct did not create a foreseeable risk of physical injury to the patient.

The patient's distress is a recoverable element of damages caused by the dermatologist's breach of duty to her. A medical specialist such as the dermatologist owes a duty to possess and exercise the degree of knowledge and skill that dermatologists across the nation exercise. He breached that duty by misdiagnosing a common skin infection that another doctor was able to diagnose immediately. His failure to properly diagnose the condition was the actual and proximate cause of injury to the patient; but for the misdiagnosis, she would not have had to continue suffering from the rash until the other doctor properly treated it. The continuation of the rash and any pain and suffering from it are compensable damages that she can recover from the dermatologist. Also compensable is the emotional distress that she suffered because of the misdiagnosis. While recovery for negligent infliction of emotional distress is not always available in many jurisdictions when there is no other injury caused by the breach, these restrictions do not apply when plaintiff is the victim of another tort that causes physical injury. Plaintiff can recover damages for emotional distress that arise from the tortious conduct. Hence, (A) is incorrect. (C) is incorrect because the patient has suffered compensable injury regardless of whether she had to pay for the second doctor visit. The continuation of the skin rash until she saw the other doctor suffices as the damage element of the prima facie case. (D) is incorrect because, given the patient's physical condition, a failure to make a proper diagnosis did create a foreseeable risk that she would continue to suffer from a painful condition that could otherwise have been alleviated. Thus, the dermatologist's conduct constituted a breach of duty.

Which of the following need NOT be shown by the plaintiff under the attractive nuisance doctrine? A The owner was or should have been aware of the dangerous condition. B The child was lured onto the property by the attractive nuisance. C The condition was likely to cause injury because of the child's inability to appreciate the risk. D The expense of remedying the situation is slight compared with the magnitude of the risk.

The plaintiff does not need to show that the child was lured onto the property by the attractive nuisance. The plaintiff does need to show that the owner was or should have been aware of the dangerous condition, that it was likely to cause injury because of the child's inability to appreciate the risk, and that the expense of eliminating the danger is slight compared with the magnitude of the risk. Under the attractive nuisance doctrine, a landowner has a duty to exercise ordinary care to avoid reasonably foreseeable risk of harm to children caused by artificial conditions on the property. To recover under this doctrine, the plaintiff must show that (i) there is a dangerous condition present on the land of which the owner is or should be aware, (ii) the owner knows or should know that young persons frequent the vicinity of this dangerous condition, (iii) the condition is likely to cause injury, i.e., is dangerous, because of the child's inability to appreciate the risk, and (iv) the expense of remedying the situation is slight compared with the magnitude of the risk.

f a statute providing for a criminal penalty is applicable to a common law negligence case, the statute's specific duty will replace the more general common law duty of care. Which of the following does a plaintiff NOT need to show to prove the availability of the statutory standard? A The plaintiff is in the class intended to be protected by the statute. B The statute was designed to prevent the type of harm that the plaintiff suffered. C The plaintiff suffered physical injury because of the defendant's violation of the statute. D The standards set out in the statute are clearly defined.

The plaintiff need not suffer physical injury from the defendant's violation of the statute. While damages is an element of the prima facie case for negligence, any type of damages, including property damages, will suffice. To prove the availability of the statutory standard, a plaintiff must show that the standards set out in the statute are clearly defined. For the statute to apply, (i) the plaintiff must be in the class intended to be protected by the statute, and (ii) the statute must have been designed to prevent the type of injury that he suffered.

The right of contribution among tortfeasors: A Imposes contribution based on equal shares of the overall liability. B Provides for apportionment of damages in the absence of joint and several liability. C Does not apply against a tortfeasor who is immune from liability. D Applies to intentional torts.

The right of contribution among tortfeasors is a device whereby responsibility is apportioned among those who are at fault. However, it does not apply against a tortfeasor who is immune from liability. If the contributing tortfeasor has a defense that would bar liability, such as intra-family tort immunity, she is not liable for contribution. In most states, contribution is based on relative fault of the various tortfeasors rather than on equal shares of the overall liability. Contribution does not provide for apportionment of damages in the absence of joint and several liability; rather, it can only operate in response to joint and several liability, because it allows any tortfeasor required to pay more than his share of damages under joint and several liability rules to have a claim against the other jointly liable parties for the excess. Contribution does not apply to intentional torts in most states.

A college student borrowed his roommate's notebook computer without permission because he needed to write a term paper that was due the next day. While the computer was sitting open on the student's desk overnight, a water pipe in the ceiling began leaking and water dripped down on the computer, rendering it inoperable. A computer repair service estimated that it would cost $500 to repair all the damaged components. At the time it was damaged, the computer was worth $700. If the roommate sues the student for the damage caused to the computer, what will be the extent of his recovery? A Nothing, because the damage occurred through no fault of the student. B Loss of use damages for the time it was in the student's possession. C $500 in damages. D $700 in damages.

The roommate can recover $700 in damages from the student for conversion. To establish a prima facie case of conversion, the following elements must be proved: (i) an act by the defendant interfering with the plaintiff's right of possession in the chattel that is serious enough in nature or consequence to warrant that the defendant pay the full value of the chattel; (ii) intent to perform the act bringing about the interference with the plaintiff's right of possession, and (iii) causation. Even if the conduct is wholly innocent, liability may attach where the interference is serious in nature. Accordingly, accidentally causing damage to another's chattel may constitute a conversion when the damage occurred while the defendant was using the chattel without permission. Here, the student interfered with the roommate's right of possession in the computer by taking it without permission, and it sustained damages of over 70% of its value while in the student's possession. Hence, the student has committed a conversion. The plaintiff in a conversion case is entitled to damages for the fair market value of the chattel at the time and place of the conversion, which in this case was $700. (A) is incorrect because even though the student was not at fault in the water pipe leaking, the damage occurred while the computer was wrongfully in his possession. (B) is incorrect. Had the computer not been damaged, the roommate's recovery would be limited to loss of use damages under a trespass to chattels theory. However, the serious damage that occurred while the computer was in the wrongful possession of the student warrants a recovery for conversion. (C) is incorrect because the damages remedy for conversion is the fair market value; in effect, there is a forced sale of the item. The student may keep the computer but he is liable to the roommate for the entire value of the computer rather than just the cost of repairs.

To establish assumption of risk, the defendant must show that the plaintiff was aware of the risk and accepted it voluntarily.

True

Truth is a complete defense to defamation.

True

A salesman in a highly competitive field went to the police station to post bond for his son, who had been arrested for possession of a small quantity of narcotics. A photographer for the local newspaper who was at the police station took a picture of the salesman flanked by two bulky police officers. The photo, which looked like the pictures of alleged criminals being taken into custody, ran on a quarter of the front page because it had been a slow news day. The photo was accompanied by a very small caption giving the salesman's name and stating that his son had been arrested for possession of narcotics. The salesman's boss was hypersensitive about the reputation of his company and fired the salesman after he saw the picture in the newspaper. If the salesman brings an invasion of privacy action against the newspaper, what is the most likely basis? A Intrusion upon seclusion B False light publicity C Public disclosure of private facts D Appropriation of plaintiff's picture for commercial purposes

The salesman's basis for an invasion of privacy action will be on the basis that the newspaper published facts about the salesman that placed him in a false light. To establish a prima facie case for invasion of privacy based on publication by defendant of facts placing plaintiff in a false light, the following elements must be proved: (i) publication of facts about plaintiff by defendant placing plaintiff in a false light in the public eye; and (ii) the "false light" must be something that would be highly offensive to a reasonable person under the circumstances. The large picture of the salesman flanked by two bulky police officers could suggest that the salesman committed a crime because it looked like pictures that newspapers often print of alleged criminals being taken into custody.This "false light" would be highly offensive to a reasonable person under the circumstances. (A) is incorrect. This branch of invasion of right to privacy, intrusion upon plaintiff's affairs or seclusion, requires (i) an act of prying or intruding on the affairs or seclusion of plaintiff by defendant; (ii) the intrusion must be something that would be highly offensive to a reasonable person; and (iii) the thing to which there is an intrusion or prying must be "private." Here, the photograph of the salesman was taken at the police station, which is a public place. Hence, the intrusion was not into anything of the salesman's private domain and is not actionable under this branch of invasion of privacy. (C) is similarly incorrect. Public disclosure of private facts requires (i) publication or public disclosure of private information about the plaintiff, and (ii) the matter made public is such that its disclosure would be highly offensive to a reasonable person. Here, the presence of the salesman outside the police station was not a private fact. (D) is incorrect because appropriation of a plaintiff's picture or name for commercial purposes must be for the promotion or advertisement of a product or service; the fact that the defendant is using the picture in a newspaper that it is selling is not sufficient.

Perrone, a landlord, rents rooms in an apartment building to tenants. Despite repeated citations from city authorities for failure to install a required fire escape at the back of the building, he doesn't install one. Late one night a first floor tenant, Dwyer, falls asleep in bed while smoking, causing a fire that destroys the building. Swensson, a tenant with a room on the front of the third floor of the building, is found dead in his bed of smoke inhalation. Swensson's family sues Perrone for wrongful death. What causation problem do you anticipate in proving Perrone's liability for Swensson's death? Suppose Swensson's body was found half way down the hall? How would that affect the causation issue?

There seems to be little question that Perrone was negligent in failing to provide adequate fire escapes for the building. Evidently a fire escape was required by law, so Perrone was likely negligent per se. However, proving Perrone negligent is not enough to establish his liability; Swensson's family will have to establish that Perrone's negligence caused Swensson's death. Yet it appears that Swensson died in his bed from smoke inhalation, without ever trying to escape from the building. If that is true, a dozen fire escapes would have made no difference in the outcome. We cannot say that, but for Perrone's failure to install a fire escape, Swensson would be alive today. It appears that Swensson would have died just the same if Perrone had not been negligent. If that is true, it will be impossible to establish that Perrone's negligence in failing to install a fire escape was a factual cause of his death. It should be easy, of course, to establish that Dwyer's negligence in falling asleep while smoking caused Swensson's death. But for that act, there would have been no fire. But the other tenant will not likely have as deep a pocket as the landlord. If Swensson is found in the middle of the hall, his family can argue that the lack of a fire escape did lead to the result, that it did "change history." If he was found in the hall, perhaps he made it to the back window, was unable to get out because there was no fire escape, and ran back toward the front. That would show that he died because of the lack of a fire escape, that "but for" the negligence of the landlord, Swensson would be alive today. That is the plaintiff's burden in establishing causation. Of course, this would still be a tough case on actual causation. The landlord will argue that Swensson never made it to the back window, so the lack of a fire escape did not cause the death, was mere "negligence in the air." It will be very difficult for the family to establish causation here without more evidence that the lack of a fire escape made a difference but it is not necessarily impossible. For example, evidence that the back window was open, or that someone saw Swensson at the window (wouldn't the family's lawyer love to have that evidence!) could help to establish causation.

Corbusier, driving down a country road, comes around a curve and encounters a deep puddle due to recent heavy rains. He tries to stop, but his brakes have been so poorly maintained that he cannot control the car. The car swerves to the left and hits Saarinen driving in the opposite direction, causing serious injuries. Is Corbusier liable under the "but for" test?

This accident is caused by both Corbusier's negligent maintenance of his car and the puddle that created the dangerous condition in the road. Although only one of these causes is due to Corbusier's negligence, that negligence is still a "but for" cause of the harm. The accident would not have happened if Corbusier had not been negligent. True, it would not have happened, even if Corbusier was negligent, if the puddle had not been there, but that does not exonerate him. It is not a defense for him that other circumstances, whether natural or negligent, also contributed to the accident, if his negligence was one of those causes.

Dobbs is driving carefully to the store for a quart of milk when Fletcher pulls out of a side street without looking and crashes into Dobbs, who swerves into Schwartz on the sidewalk. Which driver's conduct is a "but for" cause of Schwartz's injuries?

This example makes a very basic point. Clearly, both Dobbs's and Fletcher's driving are "but for" causes of the accident. If Dobbs hadn't gone for milk, the accident wouldn't have happened; if Fletcher hadn't pulled out of the side street, the accident wouldn't have happened. However, the fact that they both caused the accident doesn't mean that they will both be liable to Schwartz. Dobbs caused the accident by nonnegligent conduct, while Fletcher was a negligent cause. In a tort case, the plaintiff must show that the defendant's tortious conduct caused the harm, not just that his act caused it. In analyzing cause in fact, then, be careful to ask the right question—whether the defendant's negligence caused the harm, not just whether his conduct caused it.

Why should you use the term "allegedly" when discussing about a criminal suspect, who was charged but not yet convicted for a crime?

To avoid liability for defamation

To prove breach of duty in a products liability action based on negligence, the plaintiff must show: A Res ipsa loquitur. B The conduct involved was below the level of care generally exercised by the defendant. C The product was dangerous because it departed from its intended design. D The defendant supplied a defective product.

To prove breach of duty in a products liability action, the plaintiff must show (i) negligent conduct by the defendant leading to (ii) the supplying of a defective product by the defendant. Negligent conduct is demonstrated by showing that the defendant's conduct fell below the standard of care expected of a reasonable person under like circumstances, not the level of care generally exercised by the defendant. To show negligence in a manufacturing defect case, the plaintiff may invoke res ipsa loquitur, but it is not required that the plaintiff prove res ipsa loquitur in establishing breach of duty. A plaintiff may show that a product was dangerous because it departed from its intended design to establish a manufacturing defect, but may instead show that the design itself is deficient (to establish a design defect).

On September 29, 2008, Citigroup announced publicly that it agreed to buy all of Wachovia's banking assets. The companies never signed a definitive merger agreement, but they did sign a two-page term sheet and an "exclusivity agreement" stating that Wachovia can't negotiate a deal with anyone else. [Note: The exclusivity agreement was legally binding.] Citi provided emergency liquidity and market support to Wachovia since the time the agreement was announced. If Citi did not provide this financing, Wachovia would have collapsed. Wells Fargo allegedly knew about the Citigroup-Wachovia agreement and then made an offer to buy Wachovia, which Wachovia accepted. Which tort, if any, was committed by Wells Fargo?

Tortious Interference with Contract

On September 29, 2008, Citigroup announced publicly that it agreed to buy all of Wachovia's banking assets. The companies never signed a definitive merger agreement, but they did sign a two-page term sheet and an "exclusivity agreement" stating that Wachovia can't negotiate a deal with anyone else. [Note: The exclusivity agreement was legally binding.] Citi provided emergency liquidity and market support to Wachovia since the time the agreement was announced. If Citi did not provide this financing, Wachovia would have collapsed. Wells Fargo then made an offer to buy Wachovia, which Wachovia accepted. Which tort, if any, may have been committed by Wells Fargo?

Tortious interference with contract

Asking a buyer to break its existing contracts with another supplier is contract interference.

True

False imprisonment occurs when a person is detained for any period of time against his or her will.

True

If a plaintiff is found to be 15% responsible for her injuries in a comparative negligence jurisdiction, the defendant will have to pay 85% of the damages to the plaintiff.

True

In Burnett v. National Enquirer, Inc., the story in the Enquirer depicting Ms. Burnett as drunk was libel, even though a retraction was printed. The standard of malice was applied, and the evidence clearly established that the article was published with knowledge that part or all of it was not true.

True

Letters of reference are usually exempt from liability for defamation as long as the employer verifies the truth of the statements in the reference.

True

Physical damages are not required for liability for false imprisonment.

True

The article about Oprah below, if untrue, constitutes defamation as long as it is published with malice or with reckless disregard for the truth.

True

To avoid liability for negligence, all persons are expected to behave as ordinary and reasonably prudent people under the same or similar circumstances.

True

In a complaint dated October 6, 2015, musical artists known as Beyoncé, Jay Z, Kanye West, Pharrell Williams and Rihanna filed suit against French clothing manufacturer ElevenParis for

Unauthorized appropriation

Sindell sues four DES makers in California. At trial, she establishes that all defendants were negligent for marketing DES, and that their individual market shares were as follows: Dl 10 percent, D2 20 percent, D3 30 percent. D4 proves that it sold no DES in the relevant market at the time. The jury finds Sindell's damages to be $100,000. Under the Sindell approach, how much should each defendant pay?

Under Sindell, where the plaintiff sues defendants representing "a substantial share" of the market, they are liable in proportion to their market shares, unless a defendant proves that it did not sell DES in the relevant market. Since D4 has made that showing, it is not liable. The other three, however, are liable for their market shares. So what do they pay? There are two possibilities: They could each pay an appropriate proportion of $100,000, the total damages. Under this approach, Dl would pay 10/60ths of $100,000, D2 would pay 20/60ths, and D3 would pay 30/60ths. Sindell would recover fully, though she only sued four makers, and there are many more out there who could have supplied the drug. Alternatively, Sindell may be interpreted to require each defendant to pay its market share percentage times the total damages. Here are the numbers: D1 Pays 10% of 100,000, D2 pays 20% of 100,000, and D3 pays 30% of 100,000. It was unclear after Sindell which of these methods the California Supreme Court intended. In Brown v. Superior Court of San Francisco, 751 P.2d 470 (Cal. 1988), the court opted for the second. Under this approach, plaintiff does not recover her full damages, because the defendants found liable do not absorb the market shares of absent defendants.

In contrast to products liability cases based on negligence, those based on strict liability do not: A Require an injured bystander to be foreseeable. B Require that suppliers have an opportunity to inspect. C Prohibit recovery of solely economic losses. D Impose liability when an intermediary negligently failed to discover the defect.

Unlike with products liability cases based on negligence, those based on strict liability do not require that suppliers have an opportunity to inspect. Thus, for a case based on the sale of a defective product, a retailer in a strict liability action may be liable for a manufacturing or design defect simply for being a commercial supplier of that defective product, even if it had no opportunity to inspect the manufacturer's product before selling it. In a negligence action, the supplier's negligence must be proved. Products liability cases based on negligence and those based on strict liability both require that an injured bystander be foreseeable. While privity is not required in these cases, and bystanders are protected and may bring a claim under either theory, they must be foreseeable plaintiffs. Liability under these theories applies only to foreseeable plaintiffs. Products liability cases based on negligence and those based on strict liability both prohibit recovery of solely economic losses. The types of damages recoverable under both theories are the same: personal injury and property damages. Economic loss cannot be the sole damage claim. As under claims based on negligence, those based on strict liability will impose liability even though an intermediary negligently failed to discover the defect. The same concepts of proximate cause govern negligence and strict liability actions. The negligent failure of an intermediary to discover a defect is not a superseding cause and does not cut off the supplier's strict liability. However, if the intermediary's conduct becomes something more than ordinary foreseeable negligence, then it does become a superseding cause.

A child is required to conform to a higher standard of care than that of a child of like age, education, intelligence, and experience when the child: A Engages in an activity that is dangerous. B Is trespassing at the location of an attractive nuisance. C Has an affirmative duty to act. D Engages in an activity that is normally one in which only adults engage.

When a child engages in an activity that is normally one that only adults engage in, he is required to conform to the same standard of care as an adult in such an activity. The general rule that a child must conform to the standard of care of a child of like age, education, intelligence, and experience does not apply. It is not enough that the child engages in an activity that is dangerous; the activity must be one in which only adults normally engage. When a child is trespassing at the location of an attractive nuisance, the landowner may have a higher standard of care than that owed to adult trespassers, but the child is not required to conform to a higher standard of care. When a child has an affirmative duty to act, he is not required to conform to a higher standard of care than that of a child of like age, education, intelligence, and experience.

When is a real estate broker entitled to commission?

When there's a ready, willing, & able buyer. This means the buyer is willing to meet the seller's terms. When a seller wouldn't agree to buyer's request for 60 days to back out if she didn't sell her own house, the buyer wasn't ready & willing, so no commission.

What's a death escrow?

When you presently grant property to another, but give it to a 3rd party who will deliver it to the grantee upon the grantor's death. The grantor must place the property outside her control.

Which of the following is correct at common law regarding affirmative duties to act? A One has a general duty to act when it will not result in risk of harm to him B One who acts for the benefit of another has a duty to continue the assistance C A "Good Samaritan" statute excuses any resulting negligence D A physician has a duty to render emergency medical care if it will not result in risk of harm to her

With regard to affirmative duties to act, one who gratuitously acts for the benefit of another is then under a duty to act like an ordinary, prudent, reasonable person and continue the assistance. There is no general duty to act, even if it will not result in risk of harm to the person who would be taking action. As a general matter, no legal duty is imposed on any person to affirmatively act for the benefit of others. Absent a statute changing the common law rule, even physicians are not required to come to the aid of a person needing assistance. A "Good Samaritan" statute does NOT excuse any resulting negligence. Many states have enacted Good Samaritan statutes, which usually exempt doctors, nurses, etc. from liability for ordinary negligence when they voluntarily and gratuitously render emergency treatment. However, liability still exists under most of these statutes for gross negligence.

Auto dealer grants deed of trust in shop to bank, which contains clause requiring him to redeem his loan within 3 months of default. The state "recognizes" deeds of trust. The auto dealer tries to redeem his loan after 3mo have passed. Can he?

Yes. The clause is invalid. A foreclosure sale is required b/c the deed of trust is treated like a mortgage. The equitable right of redemption allows Auto Dealer to redeem his loan anytime before foreclosure. This right can be waived after default, but not before. The clause waived the right before default, so it's invalid.

A shopper was in a large department store that was remodeling its menswear department and had hired a contractor to do the work. A carpenter employed by the contractor was working on the remodeling job. When the carpenter left the store to take a break, she left a carpenter's level projecting out into one of the aisles, unbeknownst to any store employees. Shortly before she returned 15 minutes later, the shopper came down that aisle and tripped over the level. He fell and struck his head on the sharp corner of a display case. The shopper required hospitalization and sued the store for his injuries. Will the shopper prevail in his suit against the store? (A) Yes, because the contractor's employee left the level in the aisle. (B) Yes, because the store's employees had a reasonable time to discover the level before the shopper fell. (C) No, because the store's employees did not leave the level in the aisle. (D) No, because the store's employees were unaware that the level was in the aisle.

Yes, because the contractor's employee left the level in the aisle.

Is The Impact Team guilty of a crime by stealing this data?

Yes, data theft is a crime

Harvey Bangbang owns the Shoot 'M Up Gun Store. He strictly instructs his employees not to load guns before demonstrating them to customers. One employee, Annie Oakleaf, is having a hard time selling a gun to a customer, Long John Silver. She loads a gun and fires at a target on the wall. She accidentally shoots Silver's leg off in the process. Will Harvey be liable for Annie's negligence? _______

Yes, even though Annie had strict instructions not to load the gun. Since the tort occurred within the scope of the employment relationship, and Annie was serving Harvey's objectives (albeit in a prohibited way), Harvey will be liable. To decide otherwise would undermine vicarious liability in general, since employers would almost always escape liability by giving their employees careful instructions. Note: However, an employee's doing what he is expressly told not to do will often be evidence (but non-dispositive) that he was acting outside the scope of employment.

Are there limitations on when public figures & public officials can recover for IIED?

Yes, if it's based on publication, they can recover only if there's actual malice.

Zorro leaves his valuable cape with the cloakroom attendant at a restaurant. When he returns, the attendant wrongfully refuses to hand over the cape, and threatens to burn it. Zorro stays for two hours before he gets the cape back. Can he successfully claim false imprisonment? _______

Yes. A false imprisonment claim requires intentional confinement to a bounded area. The restraint needn't be physical; it can be accomplished by duress. Wrongfully keeping the plaintiff's valuable property is regarded as one type of duress that qualifies.

The Plen-Tee O'Food Company organizes and runs country fairs. For the Lonornament County Fair, Plen-Tee contracts with Circe du Lune, a highly respected holiday-light-show company, to run a laser-guided light show at night. Due to Circe's negligent running of the show, Patron, an audience member, is blinded. A light show of this sort is perfectly safe if proper techniques are used, which they weren't here. Circe is jugment proof. Will Plen-Tee be liable for Patron's blindness? _______

Yes. Although employers are in general not vicariously liable for the torts of their independent contractors, they are liable in a number of special situations. One of those situations is where the work being delegated to the independent contractor poses a "peculiar risk" of physical harm if not properly done. That's the case here. So even though this was not an ultra-hazardous activity (and Circe would be liable only if negligent, as it was), the mere fact that the activity posed a particular risk of harm if not conducted properly means that Plen-Tee is vicariously liable for the negligence of its independent contractor, Circe. Notice that this result occurs even though Circe was apparently well-qualified for the job when picked by Plen-Tee (so that Plen-Tee was not directly negligent in its own behavior regarding the contractor).

Allnever Tell gives his four-year-old son, Willie, a real bow and arrow set for Christmas. Willie takes it outside and fires an arrow at his neighbor, Captain Hook, hitting him in the arm. Will Allnever be liable? _______

Yes. As a general rule, parents are not vicariously liable for their children's torts. However, parents can be directly liable for their children's torts under certain circumstances. One such circumstance exists here: when a parent allows the child to use a dangerous object which the child lacks the maturity and judgment to use safely, the parent will be liable for torts committed with the object. It's clearly unreasonable to give a four-year-old a real bow and arrow. That makes Allnever negligent, and makes him liable for Hook's injuries.

Aggressive, while riding her bicycle on the sidewalk (instead of on the street where she should be), and riding too fast, nearly hits Bystander. To avoid being hit, Bystander throws himself into the street. Bystander is not seriously injured by the impact with the street. However, before he can get up, Careless, who is driving his car too fast and not paying attention, runs over Bystander, causing Bystander's leg to be amputated. Bystander chooses to sue only Aggressive for his injuries. Under the common-law approach, may Bystander recover the full value of his lost leg from Aggressive? _______

Yes. Both Aggressive and Careless were proximate and "but for" causes of the injury to Bystander. Therefore, under the common-law approach they are jointly and severally liable for the damage to Bystander. (The joint-and-several rule applies only where the harm is not capable of apportionment, and a single injury or death is never apportioned.) Because of the joint-and-several liability, Bystander may get a judgment (and collect it) against either Aggressive alone, Careless alone, or both. (However, Bystander may only collect a single time.) See Rest. 2d, §879, and Illustr. 2 thereto. If Bystander collects the full judgment against Aggressive, Aggressive will be entitled to contribution from Careless. Note, however, that the result would be different in many states today: many states have statutorily abolished joint-and-several liability in many or all contexts, and in such a state Aggressive might be liable for only her pro rata share of the damage (e.g., in a comparative-negligence jurisdiction, her percentage of fault).

Cosa Nostra Collectors, Inc. runs a debt collection service. All employees of Cosa are instructed that they should never use violence, or even threats of violence, in attempting to collect a debt. Vincent ("Big Vinny") Testarosa, one of Cosa's collectors, attempts to collect a $10,000 debt owed by Potter to a Cosa client, Carla. Potter refuses to pay even though (as Vincent knows) Potter has the money. In order to soften Potter's resistance, Vincent disregards his employer's instructions, and with an unlicensed pistol fires a slug through Potter's left kneecap, crippling him for life. Potter then pays the money. Potter (after assuming a new identity and state of residence) brings suit against Cosa for battery, under the doctrine of respondeat superior. Can Potter recover? _______

Yes. Even if the tort committed by servant is an intentional one, the master will be held liable for it under the doctrine of respondeat superior, provided that the tort was committed in some sense in furtherance of the employer's business. According to most courts, it does not matter that the method or action used was expressly forbidden by the employer, as long as it was done in furtherance of the employment. Since Vincent, when he fired the slug into Potter's kneecap, was attempting to collect the debt (and indeed the slug helped him succeed), a court would almost certainly find that Vincent was acting in furtherance of his employment with Cosa, so that Cosa would be liable under respondeat superior.

Cruise ship knew a cabin lock was broken. Friend locked P in cabin knowing he was claustrophobic, & P couldn't get out w/ his key b/c broken lock. Is cruise ship liable in negligence for therapy needed b/c nightmares from incident?

Yes. Friend locking in cabin isn't superseding cause b/c P could've accidentally locked himself in--type of harm is foreseeable.

A chiropractor bought a custom table that tilted down. It malfunctioned & the patient wrenched chiropractor's shoulder as he fell. Can chiropractor recover from manufacturer in strict liability?

Yes. He's a proper plaintiff b/c purchasers, users, and bystanders can all recover for injuries caused by a defective product.

Pompeii Canned Goods, Inc., ships cans of Vesuvius Stew to the Volcano Grocery Store. The cans are not properly sealed, and are starting to bulge due to bacteria growth. Volcano doesn't notice and puts them on the shelves anyway. Most of the city comes down with salmonella poisoning as a result of eating the tainted stew. One purchaser who becomes violently ill, Frequentus Regurgitus, sues Volcano on a strict product liability theory, and recovers a $100,000 judgment. Is Volcano likely to be able to recover the entire $100,000 from Pompeii? _______

Yes. Keeping in mind that rules on indemnity vary from state to state, a situation like this is one where indemnity would likely be applied. If the defendant is liable only because he failed to discover another's misconduct, he will normally be entitled to indemnity. A manufacturer who produces defective goods will generally be required to indemnify a retailer who resells the goods and incurs strict liability (as long as the retailer did not know of the defect). Volcano can recover the entire $100,000 from Pompeii. NOTE: Where strict liability is involved, all subsequent suppliers can seek indemnity from those before them in the supply chain, so that the manufacturer — or whoever is responsible for the defect — is ultimately responsible as long as the item was in a defective condition unreasonably dangerous to the user or consumer when it left his control. (Strict liability is liability without fault; that is, liability without regard to how careful the defendant was.")

Grandson moves nextdoor to his elderly grandmother to help her b/c she has a heart condition. He begins banging & yelling threats to abandon her at all hours to make her pay his rent. She has a heart attack due to one of these rants. Is grandson liable for IIED?

Yes. Two general rules would ordinarily prevent recovery, but this falls under exceptions to both. 1. Abusive language is ordinarily not extreme & outrageous. BUT it can be if (1) D is a position of authority or influence over P or (2) P has a known heightened sensitivity. Grandma is elderly, which constitutes a known heightened sensitivity. 2. Unreasonable distress is ordinarily not recoverable. BUT it is if P has a known hypersensitivity. Grandson knew of grandma's heart condition, which is a hypersensitivity.

Caesar and Antony are fighting over possession of an asp which slithered into a street from the woods. As they wrestle over it, Cleopatra walks by, and they accidentally bump into her with the snake. It bites her and she is seriously injured. She sues Caesar (but not Antony) in negligence, and recovers a $100,000 judgment from him. Is Caesar likely to be entitled to contribution from Antony? _______

Yes. Where joint tortfeasors act in concert and their negligence causes harm, and the plaintiff only sues one of the tortfeasors, that tortfeasor can seek contribution (partial reimbursement) from the other joint tortfeasor(s). If the jurisdiction follows comparative negligence, the court will probably apportion the liability between the two in proportion to their fault. In a non-comparative-negligence jurisdiction (and in some comparative-negligence states), the court will split the liability evenly regardless of which tortfeasor was most at fault. Without the doctrine of contribution (which applies in most but not all states), Caesar could not recover anything from Antony, since this is a case of "joint liability": Caesar and Antony acted in concert, and the damages are indivisible. NOTE: Since Cleopatra recovered the entire judgment from Caesar, her claim has been "satisfied," and she can't proceed against Antony. Also, note that Cleopatra could have sued Caesar and Antony in the same lawsuit, recovered a judgment against them, and then proceeded to collect from either one or partially from both — her choice — until her claim was satisfied. Finally, note that the rule of "contribution" is not applicable to intentional torts. RELATED ISSUE: Say Antony and Caesar each had an asp, and each negligently let his asp bite Cleo, injuring her with two separate wounds. The damages would be divisible, and thus joint liability would not apply.

When you make false statements that injure a specific product

You are liable for product disparagement

You manage a small retail store and at closing time, you allow customers to stay inside the store to continue shopping by locking the doors. One of those customers claims to be claustrophobic and tells you to immediately unlock the doors. It takes you a few minutes to unlock the store's doors.

You may be liable for false imprisonment

A company that puts a product into the stream of commerce has a duty to exercise due care in its manufacturing, which would include ______________________

a reasonable inspection

(Fact Pattern) . If a previous employer of the murder suspect had given Vaughan Foods a glowing recommendation that omitted important information about violence on the job or drug activity, that previous employer could be held liable for a. Either fraud or misrepresentation. b. libel. c. slander d. tortious interference with contract e. a, b, and c.

a. Either fraud or misrepresentation.

Comparative negligence means a. If any portion of the damages suffered by the plaintiff were caused by the plaintiff, the plaintiff's damages will be reduced by the amount of damages he caused. b. If any portion of the damages suffered by the plaintiff were caused by the plaintiff, the plaintiff may recover damages for all of his injuries. c. If any portion of the damages suffered by the plaintiff were caused by the plaintiff, the plaintiff may not recover damages for any of his injuries. d. None of the above.

a. If any portion of the damages suffered by the plaintiff were caused by the plaintiff, the plaintiff's damages will be reduced by the amount of damages he caused.

A local liquor store posts the checks of customers that are returned from the bank. The store manager says the public display of the checks is a good way to deter other bad-check writers. The display of the checks is: a. an invasion of privacy. b. defamation. c. protected by a qualified privilege. none of the above

a. an invasion of privacy.

Bill entered a car race knowing that there was a risk of injury or death from a crash. Bill was seriously injured in a crash. Bill sued the owner of the track alleging negligence. The track owner's best defense is: a. assumption of the risk b. contributory negligence c. intervening forces d. all of the above

a. assumption of the risk

The plaintiff must prove that both "but for" causation and "proximate cause" exists in a negligence case in order to recover. TRUE. [Source: Spring 2015 Exam.] The "But for" test: a. is a test for causation. b. is the test for proximate cause. c. has been eliminated as the causation test in negligence cases. d. none of the above

a. is a test for causation.

You practice your twerking moves on the dance floor so you can impersonate your favorite pop singer, Miley Cyrus. Suddenly, you lose your balance and slip and fall, crashing into another patron on the dance floor and as a result, the patron has broken her wrist. You are liable for this tort a. negligence b. strict liability c. defamation d. invasion of privacy NOTE: You had no intent so you are liable for negligence.

a. negligence

Usually established by the "but for" test—an act is the actual cause of an injury when it would not have occurred but for the act

actual cause

Intentional creation by the defendant of a reasonable apprehension of immediate harmful or offensive contact to the plaintiff's person

assault

A direct cause case is one where the facts present an uninterrupted chain of events from the time of the defendant's negligent act to the time of plaintiff's injury. If a particular harmful result was ____________________ from the defendant's negligent conduct, the unusual timing of cause and effect or the unusual manner in which the injury occurred is _______________ to the defendant's liability.

at all foreseeable; not relevant

Under the _____________________________, a child trespasser who is injured by a dangerous artificial condition need not have been attracted onto the property by the condition

attractive nuisance doctrine

A tenant invited a friend over for dinner. On his arrival, the friend stepped on a split board on the front steps and the board broke, causing him to break his ankle. If the friend sues the tenant for his injuries and does not prevail, what is most likely the reason? a) in the lease, the landlord had undertaken the duty to discover and repair dangerous conditions on the premises b) the friend should have noticed the dangerous condition himself c) the friend arrived an hour earlier than his invitation specified d) the tenant had stayed beyond the lease term and she no longer had legal right to occupy the residence

b

if a driver brings a strict liability action against the manufacturer in a jurisdiction that doesn't apply its comparative negligence rules to strict liability, what is the likely result? a) driver will be awarded damages for all injuries incurred as a result of the accident b) driver will be awarded damages for injuries incurred because the turn signal rod was defective, but he will not recover for other injuries incurred in the accident

b

Which of the following constitutes a defamatory statement (assuming the statements are untrue)? a. "I don't think you'll be happy with their work." b. "He was dismissed for embezzlement from his last job." c. "I don't care for him at all." "All people from Massachusetts are stupid."

b. "He was dismissed for embezzlement from his last job."

In the question above, Citibank sued Wells Fargo for $20 billion in compensatory damages and $40 billion in punitive damages. Recent case law noted in your textbook states that the punitive damage award a. can be any amount that the judge or jury feels will adequately punish the wrongdoers. b. cannot exceed the amount of compensatory damages awarded. c. will always equal the amount sought in the complaint. d. can not exceed the amount of damages that the plaintiff can afford to pay

b. cannot exceed the amount of compensatory damages awarded.

Which of the following defenses to negligence serves as a complete bar to recovery? a. comparative negligence b. contributory negligence c. both a and b d. none of the above

b. contributory negligence

A harmful or offensive contact with the plaintiff's person intentionally caused by the defendant

battery

In order to prove negligence a. "but for" causation must be proven. b. Proximate cause must be proven. c. Both a and b. d. None of the above.

c. Both a and b.

(Fact Pattern) Would the victim's heirs recover from Vaughan Foods for damages (including future wages, pain and suffering and so forth) relating to wrongful death (for negligent hiring) in this case? a. Only if Vaughan did a background check. b. Only if Vaughan did not do a background check. c. The heirs would receive damages whether or not Vaughan did a background check. d. The heirs would receive nothing regardless of whether a background check had been done.

d. The heirs would receive nothing regardless of whether a background check had been done.

The defense of shopkeeper's privilege a. Applies to the tort of negligence. b. Applies to the tort of defamation. c. States that shopkeepers may detain individuals for a reasonable time. None of the above.

c. States that shopkeepers may detain

10.A baseball fan sued ESPN, the network's announcers, Major League Baseball and the Yankees for $10 million for defamation after he was filmed sleeping in April 2014 during the fourth inning of Yankees game against the Boston Red Sox. The ESPN broadcasters described him using derogatory words, such as "stupor" and "stupid". What was a valid reason for the court to dismiss the case? a. ESPN was not negligent. b. The fan could not prove that ESPN was telling the truth. c. The fan was sleeping and the other statements were opinions. d. ESPN committed intentional infliction of emotional distress

c. The fan was sleeping and the other statements were opinions.

Jane Mitchell, age 16, was shopping at her neighborhood Osco Drug Store. She had been looking at magazines as she waited for a friend. She decided to purchase a Tiger Beat magazine and then wait for her friend outside the store. She paid for the magazine, but as she headed for the door, the store manager used the store's loudspeaker system to announce, "You, with the green hair and the maroon Doc Marten's on. Yes, you, by the front of the store. I saw you take that magazine. Stop right there. I have a gun pointed at you." The manager's actions: a. were excessive but will not result in any liability. b. are protected by the shopkeeper's privilege. c. constitute defamation. d. none of the above.

c. constitute defamation.

On September 29, 2008, Citigroup announced publicly that it agreed to buy all of Wachovia's banking assets. The companies never signed a definitive merger agreement, but they did sign a two-page term sheet and a legally enforceable "exclusivity agreement" stating that Wachovia can't negotiate a deal with anyone else. Citi provided emergency liquidity and market support to Wachovia since the time the agreement was announced. If Citi did not provide this financing, Wachovia would have collapsed. Wells Fargo then made an offer to buy Wachovia, which Wachovia accepted. Which tort, if any, was committed by Wells Fargo? a. defamation. b. breach of contract. c. strict liability. d. tortious interference with contract.

d. tortious interference with contract.

Although private necessity is a defense to trespass to lands, it does not relieve liability for ________________________

damage done to the property

Jane Mitchell, age 16, was shopping at her neighborhood Osco Drug Store. She had been looking at magazines as she waited for a friend. She decided to purchase a Tiger Beatmagazine and then wait for her friend outside the store. She paid for the magazine, but as she headed for the door, the store manager used the store's loudspeaker system to announce, "You, with the green hair and the maroon Doc Marten's on. Yes, you, by the front of the store. I saw you take that magazine. Stop right there. I have a gun pointed at you." The manager's actions:

constitute defamation

Invasion of privacy includes a. Intrusion into plaintiff's private affairs b. Public disclosure of private facts c. Appropriation of another's name for commercial advantage d. All of the above.

d. All of the above.

The elements of defamation are a. A statement about a business' or person's reputation or honesty that is untrue b. A statement about a discernable business c. Publication to a third party d. All of the above.

d. All of the above.

John Smith is in the sales department in a large high tech company. In a meeting for all of the sales executives in the company, his manager accused him of inflating his sales figures. What does John need to prove in order to recover? a. That the statements were published. b. Malice or reckless disregard for the truthfulness of the statements. c. That John suffered damages. d. Both a and c. e. A, b, and c above.

d. Both a and c.

landowner duty to invitees

duty to reasonably inspect for dangerous conditions

landowner duty to licensees

duty to warn all dangerous artificial and natural conditions that he/she is unlikely to discover

landowner duty to discovered trespasser

duty to warn or make safe of manmade death traps

The tort of contract interference occurs when a. there is a valid contract b. the third party knew of contract c. the plaintiff is injured d. the third party induces one of the parties to breach the original contract. e. all of the above.

e. all of the above.

The statement above was released to the press (TMZ) by Warner Brothers ("WB") when it fired the notorious actor with the "bad boy" image, Charlie Sheen. What does Sheen have to prove to recover? a. that the allegations are true. b. that the allegations are false. c. that the statements were published. d. both a and c are correct. e. both b and c are correct.

e. both b and c are correct.

Which of the following circumstances has no bearing on whether an employer will be vicariously liable for an employee's intentional tort? (a) employee was furthering business of employer (b) friction was generated by the employment (c) employee was negligently supervised by the employer (d) force was authorized in employment

employee was negligently supervised by the employer (that would be direct liability)

for the res ipsa loquitur doctrine to apply, the plaintiff must show that (i) the accident causing his injury is the type that would not normally occur unless ___________________; (ii) the negligence was _________________________; and (iii) the injury was ________________________.

someone was negligent; attributable to the defendant; not attributable to the plaintiff

consent may either be ________________ or ___________________

express; implied

T/F friend passengers of your car and passenger's of an airplane are owed the same duty of care

false

T/F in a direct cause case, the unusual manner in which the injury occurred is relevant

false

t/f generally, a parent is vicariously liable for a child's torts at common law

false

T/F If a defendant can bring the defense of private necessity, he will be relieved of all liability

false (he will still be liable for damage done to the property)

A trainer of homing pigeons brought several of them to a park that he often used for training. He had trained this group of pigeons carefully and was confident that they would readily find their way home. When they were released, one of the pigeons inexplicably turned in the opposite direction from home. Several blocks away at the other end of the park, it collided with a radio-controlled model airplane that its owner had just purchased and was trying out for the first time. The collision sent the airplane out of control; it dipped low across a highway and was struck and run over by a truck. The airplane owner sued the pigeon trainer for the destruction of his airplane. The parties stipulated to the above facts and the airplane owner presented evidence of his damages. The trainer then moved for a directed verdict. Should it be granted? A No, because the trainer's pigeon caused the destruction of the airplane. B No, because the jury could find negligence on the trainer's part under the doctrine of res ipsa loquitur. C Yes, because the truck, rather than the pigeon, was the direct cause of the airplane's destruction. D Yes, because the trainer took reasonable care in training his pigeons.

he court should grant a directed verdict for the trainer because the airplane owner has not shown that the trainer breached any duty that he owed to him. A prima facie case of negligence requires plaintiff to show the following elements: (i) the existence of a duty on the part of the defendant to conform to a specific standard of conduct for the protection of the plaintiff against unreasonable risk of injury, (ii) breach of that duty by the defendant, (iii) that the breach of duty was the actual and proximate cause of plaintiff's injury, and (iv) damage to plaintiff's person or property. Here, it is doubtful that the trainer's releasing his pigeons created any duty to other users of the park. To the extent that it did, the fact that he had taken great care to train them to return directly to their roosts indicates that he did not breach his duty to the airplane owner. Because the airplane owner has offered no other evidence of negligence, nor any reason to impose strict liability on the trainer (as discussed below), the trainer's motion for a directed verdict should be granted. (A) is incorrect because that choice indicates the imposition of a strict liability standard on the trainer. The owner of a domestic or inherently nondangerous animal is not strictly liable for the injuries it causes. The conduct of the trainer's homing pigeon would not make the trainer liable in the absence of some negligence on his part. (B) is incorrect because the doctrine of res ipsa loquitur applies only to situations where the fact that a particular injury occurred itself establishes that defendant breached a duty. If the doctrine is applicable, no directed verdict may be given for defendant because plaintiff has established a prima facie case. However, the accident must be the type that would not normally occur unless someone was negligent. The collision between the trainer's homing pigeon and the model airplane is not that type of accident; by itself, it provides no suggestion that anyone was negligent. (C) is incorrect because the truck is not a superseding force that breaks "the causal connection" between the action of the trainer's pigeon and the airplane's destruction. In indirect cause cases, where a force came into motion after defendant's act and combined with it to cause injury to plaintiff, defendant will still be potentially liable for foreseeable intervening forces that are within the increased risk caused by his acts. Even if the intervening force is independent (i.e., not a natural response or reaction to the situation), it will be foreseeable where defendant's negligence increased the risk that the independent force would cause harm. Hence, if the trainer were negligent in releasing his pigeon, the fact that the destruction of the airplane was directly caused by the truck would not relieve the trainer from liability, because the initial collision with the pigeon caused the airplane to go out of control and created a substantial risk that it would be damaged by an intervening force.

A golfer and her instructor were playing golf in a foursome when the golfer became very annoyed with critical comments made by the instructor. To show the other golfers in the group how annoyed she was with her instructor, the golfer stood a few yards behind him while the instructor was teeing off and swung a club at him. The instructor, who was focusing on his shot, was not within range of the club but unfortunately the club slipped out of the golfer's hands and struck the instructor in the head, injuring him. If the instructor brings a battery action against the golfer, will he recover? A Yes, because the golfer acted intentionally and caused harmful contact to her instructor. B Yes, because the golfer intended to cause the instructor reasonable apprehension of imminent harmful contact. C No, because the golfer did not intend to cause harmful or offensive contact. D No, unless the golfer acted unreasonably in swinging the club at her instructor.

he golfer will not be liable because she did not intend to cause harmful or offensive contact. The prima facie case for battery has the following elements: (i) an act by the defendant that brings about harmful or offensive contact to the plaintiff's person; (ii) intent on the part of the defendant to bring about harmful or offensive contact to the plaintiff's person; and (iii) causation. Here, the golfer did not have the intent to cause harmful or offensive contact. Hence, she will not be guilty of battery. (A) is incorrect because even though the golfer had the intent to swing the club, she did not have the intent required for battery—to cause harmful or offensive contact to another. (B) is incorrect because the facts do not support an intent to cause an assault. Under the transferred intent doctrine, an intent to cause an assault (intent to cause apprehension of imminent harmful or offensive contact) will satisfy the intent requirement for battery when the other elements of battery are present. Here, however, the golfer was standing behind the instructor and was intending only to show the other golfers how annoyed she was. No intent to commit assault is apparent here. (D) is incorrect because it describes a negligence standard. The instructor may be able to recover against the golfer in a negligence cause of action if the golfer acted unreasonably in swinging the club, but this does not establish intent for a battery action.

A man purchased a new power boat with an inboard engine from a boating supply store. The boating supply store properly inspected the boat before delivery, but did not detect a virtually invisible manufacturing defect in the boat's steering mechanism. Later that summer, when the man was driving his boat, the steering mechanism of his boat jammed, and the boat crashed. The man was severely injured. The man brings an action for damages against the boating supply store on a theory of strict liability in tort in a jurisdiction that does not apply its comparative fault rules to strict liability actions. Here, how relevant is the fact that the boating supply store properly inspected the boat to the question of liability?

it is not relevant here because what matters is that there was, in fact, a defect before the boat left the manufacturer

Where D's actions cause peril - even if he was not negligent - he owes a duty to the person harmed to ________________________________

make reasonable efforts to rescue

A mother whose son was riding on a ride walked away to get a snack. She heard a commotion by the ride and saw a crowd gathered. When she came closer, she heard someone close to the scene stay that a young boy had fallen off the ride. Does this count as her perceiving the accident?

no

someone bought a tv that ended up blowing up. if they bring a negligence action against the store, will they win for the fact that the store sold a defective tv?

no (if they sued on a theory of strict liability- yes, but they sued on a theory of negligence. so no)

what effect do foreseeable intervening forces have on the defendant's liability?

none- liability not cut off

in car accident cases, medical malpractice following a driver's negligence is NOT considered a ___________________

superseding cause (therefore, car drivers who cause accidents are generally liable for any resulting medical malpractice)

for a dangerous defect, the product leaving the manufacturer with the dangerous defect is enough to trigger strict liability. but, if the defect isn't necessarily dangerous but instead _________________________________________, a failure to provide sufficient warning can be sufficient to trigger strict products liability

presents an unreasonable risk of injury to users and bystanders

necessity is either ___________________ or ___________________

public; private

a rescuer is a foreseeable plaintiff as long as the rescue is not done _______________

recklessly

A consumer purchased a grass trimmer from a hardware store. He took it out of the box and assembled it according to the instructions. He noticed that there were bolts and screws left over and some joints that could have accepted additional fasteners, but he just discarded the extra hardware. As he was using the trimmer, the housing came apart and a hard piece of plastic flew off. His neighbor, who was standing nearby, was struck in the eye by the piece of plastic and suffered permanent injuries. The neighbor sued the hardware store and the manufacturer of the trimmer in a strict liability action. Through discovery, it was determined that the instructions omitted a critical step in the assembly process that would have used the extra hardware, which is why the housing came apart, and that the manufacturer had received some complaints about the instructions previously. The hardware store had no knowledge of any complaints regarding any of the manufacturer's products. As to the hardware store, the neighbor will:

recover, because the consumer's failure to recognize the improper assembly does not cut off the store's liability

For the doctrine of ___________ to apply, the plaintiff must establish that the accident causing his injury is the type that would not normally occur unless someone was negligent.

res ipsa loquitur

Normally, at common law, contributory negligence is a complete bar to recovery. However, under _____________________, a person can recover despite their own contributory negligence.

the doctrine of last clear chance

Liability arises from supplying a defective product even if the defendant exercised due care and was not negligent

strict liability

conduct that creates liability in the absence of fault

strict liability

something is NOT a ____________________ of the loss if it was within the increased risk caused by the D's negligence

superseding cause

A man purchased a new power boat with an inboard engine from a boating supply store. The boating supply store properly inspected the boat before delivery, but did not detect a virtually invisible manufacturing defect in the boat's steering mechanism. Later that summer, when the man was driving his boat, the steering mechanism of his boat jammed, and the boat crashed. The man was severely injured. The man brings an action for damages against the boating supply store on a theory of strict liability in tort in a jurisdiction that does not apply its comparative fault rules to strict liability actions. Who will prevail?

the man because the steering failed due to a defect present when the boat left the manufacturer

In conversion, the intent involved refers to ___________________, not to the Defendant's desires regarding the ultimate disposition of the property

the physical act

Where a product presents an unreasonable risk of injury to users and bystanders, the fact that ______________________________ may be a dangerous defect hat will invoke strict products liability

there is no sufficient warning of the danger

An intentional act by the defendant that causes an interference with the plaintiff's right of possession in a chattel, resulting in damages

trespass to chattels

An intentional act by the defendant that causes a physical invasion of the plaintiff's real property

trespass to land

elements for attractive nuisance: (i) condition is in a place where a child is likely to _____________________ (ii) the risk is one the owner or occupier _______________________ involves an unreasonable risk of death or serious bodily harm (iii) the risk is one that a child _______________ realize because of his/her youth (iv) the owner/occupier's utility in maintaining the dangerous condition and the burden of eliminating the ____________ are slight compared to the risk to the children

trespass; knows or should know; would not; danger

T/F Res ipsa loquitur requires that the plaintiff present evidence connecting the defendant with the negligence that occurred in order to support a finding of liability.

true

T/F a defendant may be both vicariously liable and directly liable in the same action

true

t/f A manufacturer is held strictly liable when a flaw in the manufacturing process makes a product more dangerous than it was intended

true

t/f Intent will transfer from the intended tort to the committed tort, or from the intended victim to the actual victim

true

t/f battery does not require harm

true

t/f an owner/occupier of land will be liable for their failure to exercise reasonable care to protect trespassing child from physical harm caused by artificial conditions

true (attractive nuisance doctrine)

t/f If the guest is likely to discover a danger himself, landowner does not have to warn

true (duty to warn licensee of dangers he/she is UNLIKELY to discover)

The general rule is that a defendant is not _____________ for the torts of an independent contractor that she hires. However, there are several broad exceptions to that rule which have nothing to do with ____________ in the selection of the contractor (which would make the defendant _____________ rather than vicariously liable)

vicariously liable; negligence; directly liable

Let's say you and your friend went to a play. At the coat check, your friend accidentally takes your coat home, instead of her own. They looked similar, but yours was an expensive designer brand. Your friend did not mean to take your coat. Then, on the street, your friend got mugged so your coat is gone. If you sued your friend for conversion for the price of your coat, will you win?

yes (even though she didn't mean to take the wrong coat, she meant to take the coat she ended up with)

Let's say a physician had consent to perform surgery on a patient's right ear. Then, she realized the patient has the same abnormality in the left ear. The physician went ahead and performed surgery on the left ear as well. It went great. Would the patient have any tort claim? If so, which?

yes, battery

Let's say a car hits a bicyclist. Turns out, the car driver was not negligent. The cyclist broke his leg, and later got pneumonia from laying on the side of the road for too long. Is the car driver liable for anything?

yes, the pneumonia


Conjuntos de estudio relacionados

The General Journal & the G/L (TRUE/FALSE EXERCISE)

View Set

APEC Architect Operations Manual

View Set

Chapter 9: Teaching and Counseling

View Set

Organizational Behavior Chapter 7

View Set

Real Bis: VA Law and Regulations - Chapter 6

View Set

MS Chapter 38 Women Reproductive

View Set

Chapter 20: Heart Failure and Circulatory Shock

View Set

Penny chapters 3 & 4 Biliary tree

View Set

CCNA (200-301) Practice Exam (ENSA v7.0)

View Set

3e031 set A Vol 1 self test questions

View Set